A14 Vectors and 3D Geometry
A14 Vectors and 3D Geometry
& 3D
Geometry
21 February 2024
Revision: 3916
TABLE OF CONTENTS
1.12 Cross Product-II: Component Definition 107
TABLE OF CONTENTS ................................. 2 1.13 Cross Product Revision 114
1. VECTORS .................................................. 3 1.14 Triple Products 117
P a g e 2 | 168
Get all the files at: https://bit.ly/azizhandouts
Aziz Manva (azizmanva@gmail.com)
1. VECTORS
1.1 Basics
A. Introduction
➢ Note that we could 𝑑𝑒𝑓𝑖𝑛𝑒 vectors to be any one of the above interpretations, and we could still work
with them.
➢ In fact, these would lead to equivalent properties, though we will not show this here.
B. Geometrical Definition
We introduce vectors in terms of coordinate geometry. Later on, we show applications in other areas, including
algebra, geometry, and physics. Vectors have deep connections to all of these areas.
1.1: Definition
A vector is a 𝑑𝑖𝑟𝑒𝑐𝑡𝑒𝑑 line segment that has a start point and an
endpoint.
𝑆𝑡𝑎𝑟𝑡 𝑃𝑜𝑖𝑛𝑡 = 𝑇𝑎𝑖𝑙 = (𝑥1 , 𝑦1 )
𝐸𝑛𝑑 𝑃𝑜𝑖𝑛𝑡 = 𝑇𝑖𝑝 = (𝑥2 , 𝑦2 )
Example 1.2
Identify the start points and the endpoints of the vectors drawn
alongside.
(2,1), (3,5)
(5,2), (7,4)
(1,2), (−3, −1)
1.3: Notation
Vectors are often indicated in textbooks using boldface notation.
𝑉𝑒𝑐𝑡𝑜𝑟 𝑎 = 𝒂
Vectors when written by hand are usually indicated with an arrow on top of the letter:
𝑉𝑒𝑐𝑡𝑜𝑟 𝑎 = 𝑎⃗
P a g e 3 | 168
Get all the files at: https://bit.ly/azizhandouts
Aziz Manva (azizmanva@gmail.com)
We will use the notation interchangeably. We might also combine both notations:
𝑉𝑒𝑐𝑡𝑜𝑟 𝑎 = 𝒂⃗⃗
Variables that you have come across so far in Algebra are scalars.
Example 1.4
Decide whether the following quantities are vectors or scalars (based on the notation):
A. 𝑥
B. 𝒙
C. 𝑥⃗
𝑆𝑐𝑎𝑙𝑎𝑟: 𝐴
𝑉𝑒𝑐𝑡𝑜𝑟: 𝐵, 𝐶
C. Translation Vectors
Example 1.6
The diagram alongside shows a vector with a start end, and an
endpoint. It also shows three different paths from the start point to
the end point.
Do these paths represent three different vectors?
𝑁𝑜.
A vector is the translation from its start point to its endpoint.
Example 1.7
Write each vector in the diagram alongside as a translation vector.
(3 − 2,5 − 1) = (1,4)
(7 − 5,4 − 2) = (2,2)
(−3 − 1, −1 − 2) = (−4, −3)
Example 1.8
A. Jane was visiting a town that had a rectangular grid layout. She
imagined a coordinate system with the town’s central square at the origin. Jane is currently at the town
fountain at (3, −2). She asked for directions to the mayor’s office, and was told she needed to apply a
translation vector of (2, −5) to her current position. Identify the coordinates of the mayor’s office.
B. Write a translation of 3 units in the 𝑥 direction, and 2 units in the 𝑦 direction as a translation vector.
C. Identify the translation in the x direction, and the y direction for the translation vector (𝜋, 𝑒).
Part A
(3,
⏟ −2) + (2,
⏟ −5) = (3 + 2, −2 − 5) = (5,
⏟ −7)
𝐶𝑢𝑟𝑟𝑒𝑛𝑡 𝑇𝑟𝑎𝑛𝑠𝑙𝑎𝑡𝑖𝑜𝑛 𝑀𝑎𝑦𝑜𝑟 ′ 𝑠
𝑃𝑜𝑠𝑖𝑡𝑖𝑜𝑛 𝑉𝑒𝑐𝑡𝑜𝑟 𝑂𝑓𝑓𝑖𝑐𝑒
Part B
P a g e 4 | 168
Get all the files at: https://bit.ly/azizhandouts
Aziz Manva (azizmanva@gmail.com)
(3,2)
Part C
𝑥 − 𝑑𝑖𝑟𝑒𝑐𝑡𝑖𝑜𝑛 𝑡𝑟𝑎𝑛𝑠𝑙𝑎𝑡𝑖𝑜𝑛 = 𝜋
𝑦 − 𝑑𝑖𝑟𝑒𝑐𝑡𝑖𝑜𝑛 𝑡𝑟𝑎𝑛𝑠𝑙𝑎𝑡𝑖𝑜𝑛 = 𝑒
Example 1.9
Consider the points:
𝐴 = (1,2), 𝐵 = (3,5), 𝐶 = (5,2), 𝐷 = (7,4), 𝐸 = (1,2), 𝐹 = (−3, −1)
Plot the following vectors, and find their values
⃗⃗⃗⃗⃗⃗⃗
A. 𝑨𝑩
⃗⃗⃗⃗⃗⃗⃗
B. 𝑪𝑫
⃗⃗⃗⃗⃗⃗
C. 𝑬𝑭
Example 1.11
Consider the points:
𝐴 = (1,2), 𝐵 = (3,5), 𝐶 = (5,2), 𝐷 = (7,4), 𝐸 = (1,2), 𝐹 = (−3, −1)
Write the following as position vectors.
A. 𝑨𝑩⃗⃗⃗⃗⃗⃗⃗
B. ⃗⃗⃗⃗⃗⃗⃗
𝑪𝑫
C. ⃗⃗⃗⃗⃗⃗
𝑬𝑭
1.13: 𝒊, 𝒋, 𝒌 notation
A vector can be written in 𝑖̂, 𝑗̂, 𝑘̂ notation as
P a g e 5 | 168
Get all the files at: https://bit.ly/azizhandouts
Aziz Manva (azizmanva@gmail.com)
̂
(𝑥1 , 𝑦1 , 𝑧1 ) = 𝑥1 𝒊̂ + 𝑦1 𝒋̂ + 𝑧1 𝒌
E. Direction
Example 1.15
Consider the points
𝐴 = (1,2), 𝐵 = (3,5), 𝐶 = (5,2), 𝐷 = (7,4), 𝐸 = (1,2), 𝐹 = (−3, −1)
Plot the following vectors, and find their values:
⃗⃗⃗⃗⃗⃗⃗
A. 𝑩𝑨
B. ⃗⃗⃗⃗⃗⃗⃗
𝑫𝑪
C. ⃗⃗⃗⃗⃗⃗
𝑭𝑬
Example 1.16
Consider the points
𝐴 = (1,2), 𝐵 = (3,5)
⃗⃗⃗⃗⃗⃗⃗ ⃗⃗⃗⃗⃗⃗⃗
A. Are vector 𝑨𝑩, and vector 𝑩𝑨 the same or different?
B. If you plot them, will they look the same? If they look different, what will be the difference?
C. What concept in Euclidean Geometry does this connect to?
Part A
The vectors are different.
Part B
The start point and the end point will be the same, but the arrow will point in exactly opposite directions.
Part C
Ray
F. Algebraic Definition
P a g e 6 | 168
Get all the files at: https://bit.ly/azizhandouts
Aziz Manva (azizmanva@gmail.com)
⃗⃗ = (𝑥, 𝑦)
𝒂
𝑥1
(𝑥
⏟ 1 , 𝑦1 ) = (⏟𝑦 )
1
𝑹𝒐𝒘 𝑽𝒆𝒄𝒕𝒐𝒓
𝑭𝒐𝒓𝒎 𝑪𝒐𝒍𝒖𝒎𝒏
𝑽𝒆𝒄𝒕𝒐𝒓
𝑭𝒐𝒓𝒎
G. Physical Interpretation
A vector can be interpreted as a directed line segment that represents:
➢ Displacement
➢ Velocity
➢ Acceleration
➢ Force
Kinematics is one of the most important applications of vectors. The above definitions are crucial to working in
Kinematics.
Example 1.22
Given position vectors ⃗⃗⃗⃗⃗⃗⃗
𝑶𝑨 = (2, −5), ⃗⃗⃗⃗⃗⃗⃗
𝑶𝑩 = (−3,4), find
A. 𝑩𝑨 and interpret it.
B. 𝑨𝑩 and interpret it.
P a g e 7 | 168
Get all the files at: https://bit.ly/azizhandouts
Aziz Manva (azizmanva@gmail.com)
Example 1.25
Decide whether the following are vectors or scalars:
A. The bank balance in a savings account
B. The pull exerted by gravity between the moon and object on the moon
C. The distance travelled by an asteroid orbiting the Sun.
D. A plane takes off from a runaway.
E. The wind conditions during a hot air balloon event.
F. The quantity of sand stored in a balloon before it takes off.
G. The temperature of a hot plate used to cook.
H. The distance travelled by a car owner travelling from home to office.
𝑉𝑒𝑐𝑡𝑜𝑟𝑠: 𝐵, 𝐷, 𝐸
𝑆𝑐𝑎𝑙𝑎𝑟𝑠: 𝐴, 𝐶, 𝐹, 𝐺, 𝐻
Example 1.27
Rory the dog is pulling a cart. He is currently stationed at (−2,5). Rory applies the displacement vector (2, −5)
of the dog cart. Determine the final position of the dog cart.
Example 1.28
Convert the following vectors, if given in row form, into column form, and vice versa.
A. (−2,8)
1
B. ( 2 )
−3
−2
( )
8
1
( , −3)
2
1.2 Properties
A. Magnitude
P a g e 8 | 168
Get all the files at: https://bit.ly/azizhandouts
Aziz Manva (azizmanva@gmail.com)
⃗⃗⃗⃗⃗⃗⃗ = |𝑨𝑩
𝐿𝑒𝑛𝑔𝑡ℎ 𝑜𝑓 𝑨𝑩 ⃗⃗⃗⃗⃗⃗⃗| = √(𝑥2 − 𝑥1 )2 + (𝑦2 − 𝑦1 )2
𝐴 = (𝑥1 , 𝑦1 )
𝐵 = (𝑥2 , 𝑦2 )
Example 1.30
Find the magnitude of the following vectors:
𝐴 = (3,4), 𝐵 = (4,9)
⃗⃗⃗⃗⃗⃗⃗
A. 𝑨𝑩
To determine the nature of the quadrilateral, we need the distance between the points.
This will be given to us by the magnitude:
⃗⃗ = (7, −4,7)
𝒂
⃗𝒃⃗ = (1, −6,10)
⃗⃗ = (−1, −3,4)
𝒄
⃗𝒅⃗ = (5, −1,5)
⃗⃗⃗⃗⃗⃗⃗| ≠ |𝑪𝑫
Since |𝑨𝑩 ⃗⃗⃗⃗⃗⃗⃗|, opposite sides are not equal:
𝑖𝑡 𝑖𝑠 𝑛𝑜𝑡 𝑎 𝑟𝑒𝑐𝑡𝑎𝑛𝑔𝑙𝑒 ⇒ 𝑂𝑝𝑡𝑖𝑜𝑛𝑠 𝐴, 𝐵, 𝐶 𝑎𝑟𝑒 𝑛𝑜𝑡 𝑣𝑎𝑙𝑖𝑑
𝑂𝑝𝑡𝑖𝑜𝑛 𝐷
Example 1.33
⃗⃗ has components 3𝑝 and 1 with respect to rectangular cartesian system. This system is rotated
A vector 𝒂
⃗⃗ has
through a certain angle about the origin in the counter clockwise sense. If, with respect to new system, 𝒂
P a g e 9 | 168
Get all the files at: https://bit.ly/azizhandouts
Aziz Manva (azizmanva@gmail.com)
components, 𝑝 + 1 and √10, then the values that 𝑝 can take are: (JEE Main 2021, 18 March, Shift-I)
The magnitude of the vector is the same before and after rotation.
⃗⃗⃗⃗⃗⃗⃗| = |𝑶𝑩
|𝑶𝑨 ⃗⃗⃗⃗⃗⃗⃗|
2 2
⃗⃗⃗⃗⃗⃗⃗| = |𝑶𝑩
|𝑶𝑨 ⃗⃗⃗⃗⃗⃗⃗|
2
(3𝑝)2 + 1 = (𝑝 + 1)2 + (√10)
9𝑝2 + 1 = 𝑝2 + 2𝑝 + 1 + 10
8𝑝2 − 2𝑝 − 10 = 0
4𝑝2 − 𝑝 − 5 = 0
(𝑝 + 1)(4𝑝 − 5) = 0
5
𝑝 ∈ {−1, }
4
The magnitude of the vector is the same before and after rotation.
⃗⃗⃗⃗⃗⃗⃗| = |𝑶𝑩
2|𝑶𝑨 ⃗⃗⃗⃗⃗⃗⃗|
2|(1, 𝑥, 3)| = |(4, −4𝑥 + 2,2)|
Expand:
40 + 4𝑥 2 = 24 + 16𝑥 2 − 16𝑥
Factor:
(𝑥 − 2)(3𝑥 + 2) = 0
P a g e 10 | 168
Get all the files at: https://bit.ly/azizhandouts
Aziz Manva (azizmanva@gmail.com)
Example 1.36
Determine the magnitude of the following position vectors:
1 3
⃗⃗ = ( , )
A. 𝒂
2 4
1 2 3 2 1 9 13 √13
|𝑎| = √( ) + ( ) = √ + =√ =
2 4 4 16 16 4
⃗⃗ = ⃗⃗⃗⃗⃗⃗⃗
𝒂 𝑶𝑨 + ⃗⃗⃗⃗⃗⃗⃗
𝑶𝑩 = ⃗𝟎⃗
Rotate ⃗⃗⃗⃗⃗⃗⃗
𝑶𝑨 + ⃗⃗⃗⃗⃗⃗⃗
𝑶𝑩 to get
⃗⃗⃗⃗⃗⃗⃗
𝑶𝑪 + 𝑶𝑫 = ⃗𝟎⃗
⃗⃗⃗⃗⃗⃗⃗
Example 1.39
⃗⃗ does not change after being rotated 𝜃°, 𝜃 ≠ 0. Identify 𝒂
𝒂 ⃗⃗.
⃗⃗ = ⃗𝟎⃗
𝒂
P a g e 11 | 168
Get all the files at: https://bit.ly/azizhandouts
Aziz Manva (azizmanva@gmail.com)
Example 1.42
Determine the values of the variables in each case
3 𝑘
A. ( ) + ( ) = 𝟎
𝑗 −4
Think of the second vector as a translation vector. It translates the first vector back to the origin:
3 𝑘 3+𝑘 0
( )+( )= ( )=( )
𝑗 −4 𝑗−4 0
Equating components:
𝑥: 3 + 𝑘 = 0 ⇒ 𝑘 = −3
𝑦: 𝑗 − 4 = 0 ⇒ 𝑗 = 4
Example 1.44
Determine which of the following vectors are equal.
Example 1.46
In the adjoining diagram, we have a circle with its center at the origin, and a radius
̂ , 𝑶𝑩
of 1 unit. Explain why 𝑶𝑨 ̂ 𝑎𝑛𝑑 𝑶𝑪̂ are unit vectors.
Example 1.47
P a g e 12 | 168
Get all the files at: https://bit.ly/azizhandouts
Aziz Manva (azizmanva@gmail.com)
⃗⃗ = (0,1)
C. 𝒄 1
⃗⃗ = ( , 𝑒)
E. 𝒆
3 √7 2
D. ⃗𝒅⃗ = (4 , 4 ) F. ⃗⃗
2
𝒇 = (𝑓, 3)
Back Calculations ⃗⃗⃗ = (2, 𝑔)
G. 𝒈
Determine the value of the variables such that the
vector is a unit vector.
1 2 1 2 2 2 2
⃗⃗ √
|𝒃| = ( ) + ( ) = √ ≠ 1 √𝑓 2 + ( ) = 1
2 2 4 3
4
⃗⃗| = √02 + 12 = 1
|𝒄 𝑓2 + = 1
2
9
3 2 9 7 16 4 5
⃗⃗| = √( ) + (√7) = √ +
|𝒅 =√ =1 𝑓2 = 1 − =
4 4 16 16 16 9 9
⃗⃗, ⃗𝒅⃗
⃗⃗, 𝒄
𝑈𝑛𝑖𝑡𝑠 𝑣𝑒𝑐𝑡𝑜𝑟𝑠 𝑎𝑟𝑒 𝒂 5 √5
𝑓 = ±√ = ±
Back Calculations 9 3
Part E Part G
|𝒆
⃗⃗| = 1 |𝒈
⃗⃗⃗| = 1
1 2 √22
+ 𝑔2 = 1
√( ) + 𝑒 2 = 1
2 4 + 𝑔2 = 1
1 𝑔2 = −3
+ 𝑒2 = 1 𝑁𝑜 𝑟𝑒𝑎𝑙 𝑠𝑜𝑙𝑢𝑡𝑖𝑜𝑛𝑠
4
3 We can also realize that there no solutions since the
𝑒2 = 𝑥 component is greater than 1.
4
3 √3
𝑒 = ±√ = ±
4 2
Example 1.49
Write the following vectors in component form using the standard basis.
A. Position vector ⃗⃗⃗⃗⃗⃗⃗
𝑶𝑨 = (3,4)
B. Position vector ⃗⃗⃗⃗⃗⃗⃗
𝑶𝑩 = (𝑥, 𝑦)
C. Position vector ⃗⃗⃗⃗⃗⃗⃗
𝑶𝑩 with a magnitude of 13 units, and which translates its input 5 units in the 𝑥
direction.
P a g e 13 | 168
Get all the files at: https://bit.ly/azizhandouts
Aziz Manva (azizmanva@gmail.com)
Parts A-B
⃗⃗⃗⃗⃗⃗⃗
𝑶𝑨 = 3𝒊̂ + 4𝒋̂
⃗⃗⃗⃗⃗⃗⃗
𝑶𝑩 = 𝑥𝒊̂ + 𝑦𝒋̂
Parts C
⃗⃗⃗⃗⃗⃗⃗ = 5𝒊̂ + 12𝒋̂
𝑶𝑪
Example 1.50
⃗⃗⃗⃗⃗⃗⃗, 𝑪𝑩
The adjacent diagram shows vectors 𝑪𝑫 ⃗⃗⃗⃗⃗⃗⃗ and 𝑪𝑬
⃗⃗⃗⃗⃗⃗. Write these vectors in
component form.
⃗⃗⃗⃗⃗⃗⃗ 2
𝑪𝑫 = ( )
2
3
⃗⃗⃗⃗⃗⃗⃗ = ( )
𝑪𝑩
4
⃗⃗⃗⃗⃗⃗ = ( )−4
𝑪𝑬
3
Example 1.51
1 √3
⃗⃗ has 𝑥 −component with magnitude | 𝒂
Vector 𝒂 ⃗⃗| , and 𝑦-component with magnitude | 𝒂⃗⃗|. Determine what
2 2
kind of triangle is formed by the vector and its components.
D. Slope of a Vector
The slope of a vector can be used to determine the direction of the vector.
Example 1.53
P a g e 14 | 168
Get all the files at: https://bit.ly/azizhandouts
Aziz Manva (azizmanva@gmail.com)
Part A
Length:
⃗⃗⃗⃗⃗⃗⃗| = √(6 − 3)2 + (8 − 4)2 = √25 = 5
|𝑨𝑩
⃗⃗⃗⃗⃗⃗⃗| = √(4 − 1)2 + (8 − 4)2 = √25 = 5
|𝑪𝑫
⃗⃗⃗⃗⃗⃗| = |𝐶𝐷
|𝐴𝐵 ⃗⃗⃗⃗⃗⃗ |
Direction/Slope
⃗⃗⃗⃗⃗⃗ = 𝑦2 −𝑦1 = 8−4 = 4
𝑆𝑙𝑜𝑝𝑒 𝑜𝑓 𝐴𝐵 𝑥 −𝑥
2 1 6−3 3
𝑦2 − 𝑦1 8 − 4 4
𝑆𝑙𝑜𝑝𝑒 𝑜𝑓 ⃗⃗⃗⃗⃗⃗
𝐶𝐷 = = =
𝑥2 − 𝑥1 4 − 1 3
⃗⃗⃗⃗⃗⃗⃗ = 𝑆𝑙𝑜𝑝𝑒 𝑜𝑓 ⃗⃗⃗⃗⃗⃗⃗
𝑆𝑙𝑜𝑝𝑒 𝑜𝑓 𝑨𝑩 𝑪𝑫
The length and the slope are the same.
The direction is also the same.
Part B
The length and the slope of the two vectors are the same.
But ⃗⃗⃗⃗⃗⃗⃗
𝑫𝑪 is oriented in 180° from ⃗⃗⃗⃗⃗⃗⃗
𝑪𝑫. Hence, it is going in the opposite
direction.
E. Angle between two vectors
Example 1.55
𝜃 is the angle between two vectors. Find the range of 𝜃.
0 ≤ 𝜃 ≤ 180°
Example 1.56
Find the angle between the vectors in each case:
A. The position vectors given by (2,0) and (0, −1) on the
coordinate plane.
⃗⃗⃗⃗⃗⃗⃗ and 𝑶𝑨
B. 𝑶𝑩 ⃗⃗⃗⃗⃗⃗⃗
⃗⃗⃗⃗⃗⃗⃗ and 𝑨𝑪
C. 𝑶𝑨 ⃗⃗⃗⃗⃗⃗
⃗⃗⃗⃗⃗⃗⃗ and 𝑨𝑪
D. 𝑶𝑩 ⃗⃗⃗⃗⃗⃗
P a g e 15 | 168
Get all the files at: https://bit.ly/azizhandouts
Aziz Manva (azizmanva@gmail.com)
Part A
Draw the vectors, and note that we get a right-angled triangle:
𝐴𝑛𝑔𝑙𝑒 𝑏𝑒𝑡𝑤𝑒𝑒𝑛 𝑉𝑒𝑐𝑡𝑜𝑟𝑠 = 90°
Part B
The vectors are already arranged tail to tail. Hence, we can directly get the
answer from the diagram:
𝐴𝑛𝑔𝑙𝑒 𝑏𝑒𝑡𝑤𝑒𝑒𝑛 𝑂𝐵 ⃗⃗⃗⃗⃗⃗ 𝑎𝑛𝑑 𝑂𝐴
⃗⃗⃗⃗⃗⃗ = 125°
Part C
⃗⃗⃗⃗⃗⃗⃗
𝑶𝑨 and ⃗⃗⃗⃗⃗⃗ 𝑨𝑪 are currently arranged tip to tail. To determine the angle
between them, we arrange them tail to tail.
We do this by moving 𝑶𝑨 ⃗⃗⃗⃗⃗⃗⃗ to the right until its tip coincides with the tip
of 𝑨𝑪 ⃗⃗⃗⃗⃗⃗.
Part D
Move 𝑶𝑩⃗⃗⃗⃗⃗⃗⃗ to the right until its tail coincides with the tail of 𝑨𝑪
⃗⃗⃗⃗⃗⃗.
Call the angle between the two vectors
𝑥 = 180 − 55 − 77
Shortcut
If you recognize that the angle between the vectors is the overlap of the two
angles, then you can directly calculate:
𝐴𝑛𝑔𝑙𝑒 𝑏𝑒𝑡𝑤𝑒𝑒𝑛 𝑉𝑒𝑐𝑡𝑜𝑟𝑠 = 125 + 103 − 180 = 48
F. 𝒏-Dimensional Vectors
So far, the vectors we have been working with have been two dimensional. However, the framework that we
have created generalizes easily to more dimensions.
Example 1.58
Write the row vector (𝑥1 , 𝑦1 , 𝑧1 ) as a column vector
𝑥1
(𝑥1 , 𝑦1 , 𝑧1 ) = ( 𝑦1 )
⏟
𝑅𝑜𝑤 𝑉𝑒𝑐𝑡𝑜𝑟 ⏟𝑧1
𝐶𝑜𝑙𝑢𝑚𝑛
𝑉𝑒𝑐𝑡𝑜𝑟
1.60: Magnitude
P a g e 16 | 168
Get all the files at: https://bit.ly/azizhandouts
Aziz Manva (azizmanva@gmail.com)
The formula for the magnitude of an n dimensional vector is a straightforward generalization of the formula in
two dimensions:
⃗⃗| = √𝑥12 + 𝑥22
⃗⃗ = (𝑥1 , 𝑥2 ) ⇒ |𝒂
𝒂
Example 1.61
Find the distance from the origin of the vector in infinite dimensions given below:
1 1 1 1
( , , ,…, 𝑛 ,…)
√2 2 √8 22
1 1
Substitute 𝑎 = 2 , 𝑟 = 2 in the formula for the sum of a geometric series:
1 1
𝑎 2
= = 2 =1
1−𝑟 1−1 1
2 2
G. Probability Vectors
Example 1.63
A.
Identification 2 2
B. (5 , 7)
State, with reasons, whether the vectors below are
Back Calculations
probability vectors.
1 1 1 Can the vectors given below be probability vectors?
A. ( , , ) If so, find the values of the missing variables.
2 3 6
P a g e 17 | 168
Get all the files at: https://bit.ly/azizhandouts
Aziz Manva (azizmanva@gmail.com)
1 3 4
C. (4 , 𝑝) E. (5 , 5 , 𝑥)
D. (2, 𝑐)
Part A 2 2 14 10 24
+ = + = ≠1
Consider the first condition. Each component of the 5 7 35 35 35
vector is between 0 and 1. That is: Not a probability vector
1 Part C
0≤ ≤1 1
2 0≤ ≤1
1 4
0≤ ≤1 Hence, the given vector can be a probability vector.
3
1 1 3
0≤ ≤1 +𝑝 =1⇒𝑝=
6 4 4
Consider the second condition. The sum of the Part D
components is: 2>1
1 1 1 3 2 1 Not a probability vector
+ + = + + =1
2 3 6 6 6 6 Part E
Both the conditions are satisfied. Hence, the vector 3 4 7
is a probability vector. + = >1
5 5 5
Part B Not a probability vector
Example 1.64
An urn that has three green balls, two blue balls, four yellow balls and five purple balls.
A. (𝑊𝑎𝑟𝑚𝑢𝑝) I draw a single ball from the urn. Represent the sample space of the possibilities using a
probability vector.
B. I draw a ball from the urn, replace it, and then draw another ball from the urn. What is the probability
that the two balls have distinct colors?
Part A
3 2 4 5
⃗⃗ = (𝐺𝑟𝑒𝑒𝑛, 𝐵𝑙𝑢𝑒, 𝑌𝑒𝑙𝑙𝑜𝑤, 𝑃𝑢𝑟𝑝𝑙𝑒) =
𝒑 , , ,
⏟ 14
14 ⏟ 14⏟ ⏟
14
𝐺𝑟𝑒𝑒𝑛 𝐵𝑙𝑢𝑒 𝑌𝑒𝑙𝑙𝑜𝑤 𝑃𝑢𝑟𝑝𝑙𝑒
( 𝐵𝑎𝑙𝑙 𝐵𝑎𝑙𝑙 𝐵𝑎𝑙𝑙 𝐵𝑎𝑙𝑙 )
Part B
1 − 𝑃(𝑆𝑎𝑚𝑒 𝐶𝑜𝑙𝑜𝑟)
Without using vectors
1 − 𝑃(𝑆𝑎𝑚𝑒 𝐶𝑜𝑙𝑜𝑟)
3 2 2 2 4 2 5 2
=1− ( ) + ( ) + ( ) + ( )
⏟14 ⏟14 ⏟14 ⏟14
𝑇𝑤𝑜 𝑇𝑤𝑜 𝑇𝑤𝑜 𝑇𝑤𝑜
[𝐺𝑟𝑒𝑒𝑛 𝐵𝑎𝑙𝑙𝑠 𝐵𝑙𝑢𝑒 𝐵𝑎𝑙𝑙𝑠 𝑌𝑒𝑙𝑙𝑜𝑤 𝐵𝑎𝑙𝑙𝑠 𝑃𝑢𝑟𝑝𝑙𝑒 𝐵𝑎𝑙𝑙𝑠]
9 4 16 25 54 27 71
=1−[ + + + ]=1− =1− =
196 196 196 196 196 98 98
Vectors
3 2 2 2 4 2 5 2 71
1− |𝒑|2 =1− ( ) + ( ) + ( ) + ( ) =
⏟14 ⏟14 ⏟14 ⏟14 98
𝑇𝑤𝑜 𝑇𝑤𝑜 𝑇𝑤𝑜 𝑇𝑤𝑜
[𝐺𝑟𝑒𝑒𝑛 𝐵𝑎𝑙𝑙𝑠 𝐵𝑙𝑢𝑒 𝐵𝑎𝑙𝑙𝑠 𝑌𝑒𝑙𝑙𝑜𝑤 𝐵𝑎𝑙𝑙𝑠 𝑃𝑢𝑟𝑝𝑙𝑒 𝐵𝑎𝑙𝑙𝑠]
P a g e 18 | 168
Get all the files at: https://bit.ly/azizhandouts
Aziz Manva (azizmanva@gmail.com)
Example 1.65
1 3 1
Bill walks 2 mile south, then 4 mile east, and finally 2 mile south. How many miles is he, in a direct line, from his
starting point? (AMC 8 2005/7)
The diagram shows three vectors (one for each movement). Note
that
➢ the starting point of 𝒃 is placed precisely where 𝒂 ends.
➢ the starting point of 𝒄 is placed precisely where 𝒃 ends.
(This approach, which can be done intuitively to get Bill’s movement is exactly the definition of vector addition.)
Example 1.66
While walking on a plane surface, a traveler first headed 18 miles north, then 11 miles west, then 6 miles south
and finally 6 miles east. How many miles from the starting point was the traveler after these four legs of the
journey? (MathCounts 1996 Warm-Up 8)
1.67: Orthogonal
An angle of 90° is called orthogonal.
P a g e 19 | 168
Get all the files at: https://bit.ly/azizhandouts
Aziz Manva (azizmanva@gmail.com)
Example 1.68
Minneapolis-St. Paul International Airport is 8 miles southwest of downtown St. Paul and 10 miles southeast of
downtown Minneapolis. Which integer is closest to the number of miles between downtown St. Paul and
downtown Minneapolis? (AMC 10B 2004/8)
Example 1.69
Point 𝐵 is due east of point 𝐴. Point 𝐶 is due north of point 𝐵. The distance between points 𝐴 and 𝐶 is 10√2, and
∠𝐵𝐴𝐶 = 45∘ . Point 𝐷 is 20 meters due north of point 𝐶. The distance 𝐴𝐷 is between which two integers? (AMC
10B 2012/12)
10√2
𝐴𝐵 = = 10
√2
𝐴𝐷 = √102 + 302 = √1000
31 < 𝐴𝐷 < 32
B. Algebraic Method
➢ When vectors are given in component form, they can be added or subtracted by adding/subtracting
their individual components.
➢ This is one of the most useful properties in applications of vectors.
Example 1.71
1 2
Add the vectors ⃗⃗⃗⃗⃗⃗⃗
𝑶𝑷 = (1, 2) 𝑎𝑛𝑑 ⃗⃗⃗⃗⃗⃗⃗
𝑶𝑸 = (3 , 2) where 𝑂 is the origin.
P a g e 20 | 168
Get all the files at: https://bit.ly/azizhandouts
Aziz Manva (azizmanva@gmail.com)
2 5
1+
⃗⃗⃗⃗⃗⃗⃗
𝑶𝑷 + ⃗⃗⃗⃗⃗⃗⃗
𝑶𝑸 = ( 3) = (3)
1 5
+2
2 2
Example 1.73
Given that 𝒂⃗⃗ = 2𝒊̂ + 3𝒋̂, ⃗𝒃⃗ = 5𝒊̂ − 2𝒋̂ find
A. 𝒂⃗⃗ + ⃗𝒃⃗
B. 𝒂⃗⃗ − ⃗𝒃⃗
𝒙 ⃗⃗ − 𝒂
⃗⃗ = 𝒃 ⃗⃗
C. Geometric Method: Addition
Example 1.78
Add the vectors
P a g e 21 | 168
Get all the files at: https://bit.ly/azizhandouts
Aziz Manva (azizmanva@gmail.com)
If you draw the vectors, you will get the diagram on the
left.
To add the vectors, pick any vector and move it to the tip of
the other vector.
Suppose, we choose to move ⃗⃗⃗⃗⃗⃗
𝑂𝐴 and put it at the tip of ⃗⃗⃗⃗⃗⃗
𝑂𝐵 .
We then get the diagram to the right.
And
⃗⃗⃗⃗⃗⃗
𝑂𝐴 + ⃗⃗⃗⃗⃗⃗
𝑂𝐵 = (3,7)
Geometric Method
We get an equilateral triangle, as shown alongside. The angle between
the line segments is 60°, but this is not the angle between the vectors,
because the vectors are arranged 𝑡𝑖𝑝 𝑡𝑜 𝑡𝑎𝑖𝑙.
To find the angle between the vectors, we move ⃗𝒃⃗ to the right until its tail
⃗⃗.
coincides with the tail of 𝒂
P a g e 22 | 168
Get all the files at: https://bit.ly/azizhandouts
Aziz Manva (azizmanva@gmail.com)
The vectors are now arranged tail to tail, and the angle is:
180 − 60 = 120°
Example 1.81
Identify the sum of the following vectors in the vector polygon drawn
alongside.
We want to find:
⃗⃗⃗⃗⃗⃗⃗ + ⃗⃗⃗⃗⃗⃗⃗
𝑨𝑩 𝑩𝑪 + ⃗⃗⃗⃗⃗⃗⃗ 𝑫𝑬 + ⃗⃗⃗⃗⃗⃗
𝑪𝑫 + ⃗⃗⃗⃗⃗⃗⃗ 𝑬𝑨
This looks complicated until we realize we can use the tip to tail approach, and we end up exactly where we
began and hence:
⃗⃗⃗⃗⃗⃗⃗ + 𝑩𝑪
𝑨𝑩 ⃗⃗⃗⃗⃗⃗⃗ + 𝑪𝑫
⃗⃗⃗⃗⃗⃗⃗ + 𝑫𝑬 ⃗⃗⃗⃗⃗⃗ = 𝑨𝑩
⃗⃗⃗⃗⃗⃗⃗ + 𝑬𝑨 ⃗⃗⃗⃗⃗⃗⃗ = 0
Example 1.83
Simplify:
A. 𝑨𝑩⃗⃗⃗⃗⃗⃗⃗ + 𝑩𝑪
⃗⃗⃗⃗⃗⃗⃗ + 𝑪𝑫
⃗⃗⃗⃗⃗⃗⃗
We do not know the details of the vectors involved. But, using the tip to tail approach (for any diagram that we
draw), we get:
⃗⃗⃗⃗⃗⃗⃗ + ⃗⃗⃗⃗⃗⃗⃗
𝑨𝑩 𝑩𝑪 + ⃗⃗⃗⃗⃗⃗⃗
𝑪𝑫 = ⃗⃗⃗⃗⃗⃗
𝑨𝑪 + ⃗⃗⃗⃗⃗⃗⃗
𝑪𝑫 = ⃗⃗⃗⃗⃗⃗⃗
𝑨𝑫
Example 1.84
𝑇𝑟𝑢𝑒 𝑜𝑟 𝐹𝑎𝑙𝑠𝑒
State whether the following statement is true or false. If it is false, correct it.
The converse of the statement “If three vectors can be arranged to form the sides of a triangle, the sum of three
vectors is zero.” is true.
The converse is: “If the sum of three vectors is zero, then the three vectors can be arranged to form the sides of a
triangle.”
P a g e 23 | 168
Get all the files at: https://bit.ly/azizhandouts
Aziz Manva (azizmanva@gmail.com)
We need to incorporate these two exceptions to make the statement true. The corrected version is:
“If the sum of three non-zero, non-collinear vectors is zero, then the three vectors form the sides of a triangle.”
Example 1.86
The cube alongside consists of unit vectors.
A. Simplify ⃗⃗⃗⃗⃗⃗
𝑬𝑨 + ⃗⃗⃗⃗⃗⃗⃗
𝑨𝑫 + ⃗⃗⃗⃗⃗⃗⃗
𝑫𝑪 + ⃗⃗⃗⃗⃗⃗⃗
𝑪𝑩
⃗⃗⃗⃗⃗⃗⃗ + 𝑩𝑪
B. √|𝑨𝑩 ⃗⃗⃗⃗⃗⃗⃗ + 𝑪𝑮
⃗⃗⃗⃗⃗⃗ + 𝑮𝑯
⃗⃗⃗⃗⃗⃗⃗|
3
⃗⃗⃗⃗⃗⃗⃗ + 𝑫𝑯
C. √|𝑨𝑫 ⃗⃗⃗⃗⃗⃗⃗ + 𝑬𝑭
⃗⃗⃗⃗⃗⃗ + 𝑯𝑬 ⃗⃗⃗⃗⃗⃗|
⃗⃗⃗⃗⃗⃗⃗ + 𝑭𝑮
Part A
⃗⃗⃗⃗⃗⃗⃗ + 𝑪𝑩
⃗⃗⃗⃗⃗⃗⃗ + 𝑫𝑪
𝑬𝑫 ⃗⃗⃗⃗⃗⃗⃗ = 𝑬𝑪
⃗⃗⃗⃗⃗⃗ + 𝑪𝑩
⃗⃗⃗⃗⃗⃗⃗ = 𝑬𝑩
⃗⃗⃗⃗⃗⃗⃗
Part B
⃗⃗⃗⃗⃗⃗ + 𝑪𝑮
√|𝑨𝑪 ⃗⃗⃗⃗⃗⃗ + ⃗⃗⃗⃗⃗⃗⃗ ⃗⃗⃗⃗⃗⃗ + ⃗⃗⃗⃗⃗⃗⃗
𝑮𝑯| = √|𝑨𝑮 ⃗⃗⃗⃗⃗⃗⃗| = √√2 = 4√2
𝑮𝑯| = √|𝑨𝑯
Part C
3 3 3 3 3
⃗⃗⃗⃗⃗⃗⃗ + ⃗⃗⃗⃗⃗⃗⃗
√|𝑨𝑯 𝑬𝑭 + ⃗⃗⃗⃗⃗⃗
𝑯𝑬 + ⃗⃗⃗⃗⃗⃗ ⃗⃗⃗⃗⃗⃗ + ⃗⃗⃗⃗⃗⃗
𝑭𝑮| = √|𝑨𝑬 𝑬𝑭 + ⃗⃗⃗⃗⃗⃗ ⃗⃗⃗⃗⃗⃗ + ⃗⃗⃗⃗⃗⃗
𝑭𝑮| = √|𝑨𝑭 ⃗⃗⃗⃗⃗⃗| = √√3 = 6√3
𝑭𝑮| = √|𝑨𝑮
Example 1.87
⃗⃗⃗⃗⃗⃗⃗ can be expressed as sum of three vectors with start and end points on
The vector 𝑫𝑭
the vertices of the cube. Write all the ways in which this can be done. If order of the
vectors does matter, then write all the ways in which it can be done if:
A. All vectors used are unit vectors
Part A
(𝑈𝑝, 𝑅𝑖𝑔ℎ𝑡, 𝐹𝑟𝑜𝑛𝑡) = (𝑢, 𝑟, 𝑓)
These can be arranged in:
3×2×1=6
𝑫𝑪
⏟ + 𝑪𝑮
⏟ + 𝑮𝑭
⏟
𝑹𝒊𝒈𝒉𝒕 𝑭𝒐𝒓𝒘𝒂𝒓𝒅 𝑼𝒑
P a g e 24 | 168
Get all the files at: https://bit.ly/azizhandouts
Aziz Manva (azizmanva@gmail.com)
⃗⃗⃗⃗⃗⃗⃗
𝑫𝑪
⏟ + 𝑪𝑩 ⏟+ 𝑩𝑭
⏟
𝑹𝒊𝒈𝒉𝒕 𝑼𝒑 𝑭𝒐𝒓𝒘𝒂𝒓𝒅
𝑫𝑨 + 𝑨𝑬 + 𝑬𝑭
𝑫𝑨 + 𝑨𝑩 + 𝑩𝑭
𝑫𝑯 + 𝑯𝑮 + 𝑮𝑭
𝑫𝑯 + 𝑯𝑬 + 𝑬𝑭
⃗⃗⃗⃗⃗⃗⃗ = 𝒂
𝑷𝑹 ⃗⃗
⃗⃗ + 𝒃
𝑸𝑺 = ⃗𝒃⃗ + ⃗𝒄⃗
⃗⃗⃗⃗⃗⃗
⃗⃗⃗⃗⃗⃗ = 𝑷𝑹
𝑷𝑺 ⃗⃗⃗⃗⃗⃗ = (𝒂
⃗⃗⃗⃗⃗⃗⃗ + 𝑹𝑺 ⃗⃗ + ⃗𝒃⃗) + 𝒄⃗⃗
⃗⃗⃗⃗⃗⃗ = 𝑷𝑸
𝑷𝑺 ⃗⃗⃗⃗⃗⃗⃗ + 𝑸𝑺
⃗⃗⃗⃗⃗⃗ = 𝒂 ⃗⃗ + 𝒄
⃗⃗ + (𝒃 ⃗⃗)
From the above two statements:
(𝒂 ⃗⃗) + 𝒄
⃗⃗ + 𝒃 ⃗⃗ = 𝒂 ⃗⃗ + 𝒄
⃗⃗ + (𝒃 ⃗⃗)
Example 1.90
Consider pentagon 𝐴𝐵𝐶𝐷𝐸. Find 𝒙 ⃗⃗ in each case below (answer separately):
⃗⃗⃗⃗⃗⃗⃗ ⃗⃗⃗⃗⃗⃗⃗
A. 𝑨𝑩 + 𝑩𝑪 = 𝒙 ⃗⃗
⃗⃗⃗⃗⃗⃗⃗ ⃗⃗⃗⃗⃗⃗
B. 𝑨𝑩 + 𝑬𝑨 + 𝑫𝑬 = 𝒙 ⃗⃗⃗⃗⃗⃗⃗ ⃗⃗
⃗⃗⃗⃗⃗⃗⃗ ⃗⃗⃗⃗⃗⃗⃗ ⃗⃗⃗⃗⃗⃗⃗
C. 𝑨𝑩 + 𝑩𝑪 + 𝑪𝑫 + 𝑫𝑬 + 𝒙 ⃗⃗⃗⃗⃗⃗⃗ ⃗⃗ = 0
⃗⃗ = 𝑨𝑩
𝒙 ⃗⃗⃗⃗⃗⃗⃗ + ⃗⃗⃗⃗⃗⃗⃗
𝑩𝑪 = ⃗⃗⃗⃗⃗⃗
𝑨𝑪
⃗⃗⃗⃗⃗⃗⃗ ⃗⃗⃗⃗⃗⃗
⃗⃗ = 𝑨𝑩 + 𝑬𝑨 + 𝑫𝑬 = 𝑫𝑩
𝒙 ⃗⃗⃗⃗⃗⃗⃗ ⃗⃗⃗⃗⃗⃗⃗
⃗⃗⃗⃗⃗⃗⃗
𝑨𝑩 + ⃗⃗⃗⃗⃗⃗⃗
𝑩𝑪 + ⃗⃗⃗⃗⃗⃗⃗
𝑪𝑫 + ⃗⃗⃗⃗⃗⃗⃗ 𝑫𝑬 + ⃗⃗⃗⃗⃗⃗
𝑬𝑨 = 0 ⇒ 𝒙 ⃗⃗ = ⃗⃗⃗⃗⃗⃗
𝑬𝑨
Example 1.92
Simplify:
A. ⃗𝒃⃗ + (−𝒃
⃗⃗)
⃗𝒃⃗ + (−𝒃
⃗⃗) = 𝟎
P a g e 25 | 168
Get all the files at: https://bit.ly/azizhandouts
Aziz Manva (azizmanva@gmail.com)
Example 1.93
Consider Regular Hexagon 𝐴𝐵𝐶𝐷𝐸𝐹. Find
A. the sum of the six vectors originating from the center of the hexagon and terminating at the vertices.
⃗⃗⃗⃗⃗⃗ + 𝑪𝑨
B. 𝑬𝑪 ⃗⃗⃗⃗⃗⃗ + 𝑪𝑫
⃗⃗⃗⃗⃗⃗⃗
Part A
⃗⃗⃗⃗⃗⃗⃗ + 𝑶𝑫
𝑶𝑨 ⃗⃗⃗⃗⃗⃗⃗ = 𝟎
⃗⃗
⃗⃗⃗⃗⃗⃗⃗ + 𝑶𝑬
𝑶𝑩 ⃗⃗⃗⃗⃗⃗⃗ = 𝟎
⃗⃗
⃗⃗⃗⃗⃗⃗⃗ + 𝑶𝑪
𝑶𝑭 ⃗⃗⃗⃗⃗⃗⃗ = 𝟎
⃗⃗
Example 1.94
Consider regular pentagon 𝐴𝐵𝐶𝐷𝐸. Find the sum of the five vectors originating from the center of the pentagon
and terminating at the vertices.
Rotate the vectors so that each vector moves over to its next vertex
counterclockwise.
For example,
𝑅𝑜𝑡𝑎𝑡𝑒 ⃗⃗⃗⃗⃗⃗⃗
𝑶𝑨 𝑠𝑜 𝑡ℎ𝑎𝑡 𝑖𝑡 𝑏𝑒𝑐𝑜𝑚𝑒𝑠 ⃗⃗⃗⃗⃗⃗⃗
𝑶𝑩
𝑅𝑜𝑡𝑎𝑡𝑒 𝑶𝑩 𝑠𝑜 𝑡ℎ𝑎𝑡 𝑖𝑡 𝑏𝑒𝑐𝑜𝑚𝑒𝑠 ⃗⃗⃗⃗⃗⃗⃗
⃗⃗⃗⃗⃗⃗⃗ 𝑶𝑪
.
.
𝑅𝑜𝑡𝑎𝑡𝑒 𝑶𝑬 𝑠𝑜 𝑡ℎ𝑎𝑡 𝑖𝑡 𝑏𝑒𝑐𝑜𝑚𝑒𝑠 ⃗⃗⃗⃗⃗⃗⃗
⃗⃗⃗⃗⃗⃗⃗ 𝑶𝑨
The sum of the vectors did not change under a rotation which was not 360°. Hence, the vector must be the zero
vector.
P a g e 26 | 168
Get all the files at: https://bit.ly/azizhandouts
Aziz Manva (azizmanva@gmail.com)
Example 1.97
In the diagram
⃗⃗⃗⃗⃗⃗⃗
𝑩𝑪 = 𝒂 𝑪𝑨 = ⃗𝒃⃗, 𝑨𝑩
⃗⃗, ⃗⃗⃗⃗⃗⃗ ⃗⃗⃗⃗⃗⃗⃗ = 𝒄
⃗⃗, ⃗⃗⃗⃗⃗⃗⃗
𝑩𝑿 = 𝒙 ⃗⃗, ⃗⃗⃗⃗⃗⃗
𝑪𝒀 = 𝒚 ⃗⃗, ⃗⃗⃗⃗⃗⃗
𝑨𝒁 = 𝒛 ⃗⃗
Express the vectors below in terms of the vectors above:
A. 𝒂⃗⃗ + ⃗𝒃⃗
B. ⃗⃗⃗⃗⃗⃗⃗
𝑨𝑿
C. ⃗⃗⃗⃗⃗⃗⃗
𝑩𝒀
D. ⃗⃗⃗⃗⃗⃗
𝑪𝒁
⃗⃗⃗⃗⃗⃗
E. 𝑪𝑿
⃗⃗⃗⃗⃗⃗
F. 𝑿𝑪
Part A
⃗⃗ + ⃗𝒃⃗ = −𝒄
𝒂 ⃗⃗
Part B
⃗⃗⃗⃗⃗⃗⃗ = 𝑨𝑩
𝑨𝑿 ⃗⃗⃗⃗⃗⃗⃗ + 𝑩𝑿
⃗⃗⃗⃗⃗⃗⃗ = 𝒄 ⃗⃗ + 𝒙
⃗⃗
⃗⃗⃗⃗⃗⃗⃗ = 𝑨𝑪
𝑨𝑿 ⃗⃗⃗⃗⃗⃗ + 𝑪𝑿
⃗⃗⃗⃗⃗⃗ = −𝒃 ⃗⃗ + 𝑪𝑿⃗⃗⃗⃗⃗⃗ = 𝑪𝑿 ⃗⃗
⃗⃗⃗⃗⃗⃗ − 𝒃
Part C
⃗⃗⃗⃗⃗⃗⃗
𝑩𝒀 = 𝒂 ⃗⃗ + 𝒚
⃗⃗
⃗⃗⃗⃗⃗⃗⃗
𝑩𝒀 = −𝒄 ⃗⃗⃗⃗⃗⃗
⃗⃗ + 𝑨𝒀
Part D
𝑪𝒁 ⃗⃗ + 𝒛
⃗⃗⃗⃗⃗⃗ = 𝒃 ⃗⃗
⃗⃗⃗⃗⃗⃗ = −𝒂
𝑪𝒁 ⃗⃗ + 𝒛⃗⃗
Part E
⃗⃗⃗⃗⃗⃗ = ⃗⃗⃗⃗⃗⃗⃗
𝑪𝑿 𝑪𝑩 + ⃗⃗⃗⃗⃗⃗⃗
𝑩𝑿 = −𝒂 ⃗⃗ + 𝒙
⃗⃗
⃗⃗⃗⃗⃗⃗ ⃗
𝑪𝑿 = 𝒃 + 𝒄⃗ ⃗⃗ + 𝒙
⃗⃗
Part F
⃗⃗ = 𝑪𝑨
𝒃 ⃗⃗⃗⃗⃗⃗
⃗⃗ = 𝑨𝑪
−𝒃 ⃗⃗⃗⃗⃗⃗
⃗⃗⃗⃗⃗⃗ = −𝑪𝑿
𝑿𝑪 ⃗⃗⃗⃗⃗⃗ = −(−𝒂 ⃗⃗ + 𝒙
⃗⃗) = 𝒂 ⃗⃗ − 𝒙
⃗⃗
⃗⃗⃗⃗⃗⃗ = 𝑿𝑨
𝑿𝑪 ⃗⃗⃗⃗⃗⃗⃗ + 𝑨𝑪
⃗⃗⃗⃗⃗⃗ = 𝑿𝑨 ⃗⃗
⃗⃗⃗⃗⃗⃗⃗ − 𝒃
Example 1.98
⃗⃗⃗⃗⃗⃗⃗ = 𝒖
𝐴𝐵𝐶𝐷𝐸𝐹 is a regular hexagon with 𝑨𝑩 ⃗⃗ and ⃗⃗⃗⃗⃗⃗⃗
𝑩𝑪 = 𝒗 ⃗⃗. Find ⃗⃗⃗⃗⃗⃗⃗
𝑪𝑫, ⃗⃗⃗⃗⃗⃗⃗
𝑫𝑬, ⃗⃗⃗⃗⃗⃗ ⃗⃗⃗⃗⃗⃗ in terms of 𝒖
𝑬𝑭 and 𝑭𝑨 ⃗⃗ and 𝒗
⃗⃗.
P a g e 27 | 168
Get all the files at: https://bit.ly/azizhandouts
Aziz Manva (azizmanva@gmail.com)
⃗⃗⃗⃗⃗⃗⃗ = −𝒖
𝑬𝑫 ⃗⃗
⃗⃗⃗⃗⃗⃗ = −𝒗
𝑬𝑭 ⃗⃗
Method I
Draw a triangle
⃗⃗⃗⃗⃗⃗ = 𝒖
𝑭𝑨 ⃗⃗ − 𝒗
⃗⃗
⃗⃗⃗⃗⃗⃗⃗
𝑪𝑫 = −𝑭𝑨 = 𝒗 ⃗⃗⃗⃗⃗⃗ ⃗⃗ − 𝒖
⃗⃗
Method II
⃗⃗⃗⃗⃗⃗⃗ + ⃗⃗⃗⃗⃗⃗⃗
𝑨𝑩 𝑩𝑪 + ⃗⃗⃗⃗⃗⃗⃗
𝑪𝑫 = ⃗⃗⃗⃗⃗⃗⃗
𝑨𝑫
⃗⃗ + 𝒗
𝒖 ⃗⃗⃗⃗⃗⃗⃗
⃗⃗ + 𝑪𝑫 = 2𝒗 ⃗⃗
⃗⃗⃗⃗⃗⃗⃗
𝑪𝑫 = 𝒗 ⃗⃗ − 𝒖 ⃗⃗
Example 1.99
The sum of two unit vectors is itself a unit vector.
A. Find the angle between the vectors.
B. Find the magnitude of the difference of the two vectors.
Part A
̂.
̂ and 𝒃
Let the unit vectors be 𝒂
̂+𝒃
𝒂 ̂ = 𝒄̂
|𝒂 ̂| = |𝒄̂| = 1
̂ | = |𝒃
Draw a diagram, and note that the triangle formed by the three vectors is
equilateral.
Part B
̂ and 𝒃
̂, 𝒃
Note that the triangle formed by 𝒂 ̂−𝒂
̂ is isosceles.
Hence, the angle shown is:
𝟏𝟖𝟎 − 𝟏𝟐𝟎 𝟔𝟎
= = 𝟑𝟎°
𝟐 𝟐
In a 30 − 60 − 90 triangle:
P a g e 28 | 168
Get all the files at: https://bit.ly/azizhandouts
Aziz Manva (azizmanva@gmail.com)
√3 √3 √3
𝑆𝑖𝑑𝑒 𝑜𝑝𝑝 60° = × 𝐻𝑦𝑝 = ×1=
2 2 2
Similarly, the other half is also:
√3
2
Hence, the magnitude of the vector we want is:
√3 √3
+ = √3
2 2
Example 1.100
̂,𝒊̂ + 𝒌
If 𝒊̂ + 𝒋̂,𝒋̂ + 𝒌 ̂ are the position vectors of the vertices of a triangle 𝐴𝐵𝐶 taken in order, then ∠𝐴 is equal to
(BITSAT 2018)
Let
⃗⃗⃗⃗⃗⃗⃗ = 𝒊̂ + 𝒋̂, 𝑶𝑪
𝑶𝑨 ̂, 𝑶𝑩
⃗⃗⃗⃗⃗⃗⃗ = 𝒊̂ + 𝒌 ̂
⃗⃗⃗⃗⃗⃗⃗ = 𝒋̂ + 𝒌
Calculate the length of the sides of the triangle by calculating the magnitude of the vectors that make up the
sides of the triangle:
⃗⃗⃗⃗⃗⃗⃗| = |𝑩𝑪
|𝑨𝑩 ⃗⃗⃗⃗⃗⃗⃗| = ⃗⃗⃗⃗⃗⃗
𝑪𝑨 = √12 + 12 = √2
Since the sides of the triangle are equal:
Δ𝐴𝐵𝐶 𝑖𝑠 𝑒𝑞𝑢𝑖𝑙𝑎𝑡𝑒𝑟𝑎𝑙 ⇒ ∠𝐴 = 60°
Example 1.101
𝑀𝑎𝑟𝑘 𝑎𝑙𝑙 𝑐𝑜𝑟𝑟𝑒𝑐𝑡 𝑜𝑝𝑡𝑖𝑜𝑛𝑠
̂, −𝒊̂ + 6𝒋̂ + 6𝒌
If the position vectors of the vertices 𝐴, 𝐵, 𝐶 of a triangle 𝐴𝐵𝐶 are 7𝒋̂ + 10𝒌 ̂ and −4𝒊̂ + 9𝒋̂ + 6𝒌
̂
respectively, the triangle is:
A. Equilateral
B. Isosceles
C. Scalene
D. Right Angled
E. Acute Angled
F. Obtuse Angled (BITSAT 2008, Adapted)
⃗⃗⃗⃗⃗⃗⃗ = 𝑶𝑩
𝑨𝑩 ⃗⃗⃗⃗⃗⃗⃗ − 𝑶𝑨 ̂ ⇒ |𝑨𝑩
⃗⃗⃗⃗⃗⃗⃗ = −𝒊̂ − 𝒋̂ − 𝟒𝒌 ⃗⃗⃗⃗⃗⃗⃗| = √12 + 12 + 42 = √18
⃗⃗⃗⃗⃗⃗⃗
𝑩𝑪 = ⃗⃗⃗⃗⃗⃗⃗ 𝑶𝑪 − ⃗⃗⃗⃗⃗⃗⃗
𝑶𝑩 = −3𝒊̂ + 𝟑𝒋̂ ⇒ |𝑩𝑪 ⃗⃗⃗⃗⃗⃗⃗| = √32 + 32 = √18
⃗⃗⃗⃗⃗⃗
𝑪𝑨 = ⃗⃗⃗⃗⃗⃗⃗
𝑶𝑨 − ⃗⃗⃗⃗⃗⃗⃗ ̂ ⇒ |𝑪𝑨
𝑶𝑪 = 4𝒊̂ − 2𝒋̂ + 4𝒌 ⃗⃗⃗⃗⃗⃗| = √42 + 22 + 42 = √36 = 6
P a g e 29 | 168
Get all the files at: https://bit.ly/azizhandouts
Aziz Manva (azizmanva@gmail.com)
Since
⃗⃗⃗⃗⃗⃗⃗| = |𝑩𝑪
|𝑨𝑩 ⃗⃗⃗⃗⃗⃗⃗| = √18 ⇒ Δ𝐴𝐵𝐶 𝑖𝑠 𝑖𝑠𝑜𝑠𝑐𝑒𝑙𝑒𝑠
Check whether the triangle is right-angled by checking whether is satisfies Pythagoras Theorem
2 𝟐 𝟐
⃗⃗⃗⃗⃗⃗⃗| + |𝑩𝑪
|𝑨𝑩 ⃗⃗⃗⃗⃗⃗⃗| = 18 + 18 = 36 = |𝑶𝑪
⃗⃗⃗⃗⃗⃗⃗| ⇒ Δ𝐴𝐵𝐶 𝑖𝑠 𝑟𝑖𝑔ℎ𝑡 − 𝑎𝑛𝑔𝑙𝑒𝑑
𝑂𝑝𝑡𝑖𝑜𝑛𝑠 𝐵 𝑎𝑛𝑑 𝐷
E. Physics Applications
Example 1.104
A trolley being dragged along the ground is subjected to a force of 5𝑁 in the upward direction, and 7N in the
negative 𝑥 direction.
A. Draw a diagram showing the trolley, the applied forces, the resultant force
B. Calculate the magnitude of the resultant force.
Magnitude of Resultant
= √72 + 52 = √74𝑁
Example 1.105
Shannon is dragging his luggage when going to vacation in the Bahamas with a force of 12N. The handle of the
luggage is at an angle of 30° to the ground. When, he gets tired, his mother pulls the luggage, with the same
force as Shannon, but at an angle of 60° to the ground (since she is taller).
A. Draw a diagram for the force vectors for Shannon and his mother.
B. Determine the components of each force vector.
C. Who will pull the luggage faster? Shannon or his mother?
Part B
𝑥
𝑺𝒉𝒂𝒏𝒏𝒐𝒏 = (𝑦) = (6√3)
⃗⃗⃗⃗⃗⃗⃗⃗⃗⃗⃗⃗⃗⃗⃗⃗⃗⃗⃗⃗⃗⃗
6
𝑥 6
⃗⃗⃗⃗⃗⃗⃗⃗⃗⃗⃗⃗⃗⃗⃗⃗⃗⃗
𝑴𝒐𝒕𝒉𝒆𝒓 = (𝑦) = ( )
6√3
Part C
Shannon will pull the luggage faster because the force in the horizontal
direction is greater for him.
P a g e 30 | 168
Get all the files at: https://bit.ly/azizhandouts
Aziz Manva (azizmanva@gmail.com)
1.106: F
𝑊𝑜𝑟𝑘 = 𝑁𝑒𝑡 𝑐ℎ𝑎𝑛𝑔𝑒 𝑖𝑛 𝑒𝑛𝑒𝑟𝑔𝑦 = Δ𝐸
𝑃𝑜𝑡𝑒𝑛𝑡𝑖𝑎𝑙 𝐸𝑛𝑒𝑟𝑔𝑦
𝐾𝑖𝑛𝑒𝑡𝑖𝑐 𝐸𝑛𝑒𝑟𝑔𝑦
𝑇𝑀𝐸 = 𝐾𝐸 + 𝑃𝑀𝐸
Example 1.107
𝑚𝑖𝑙𝑒𝑠
Ignore friction. Shannon drags his ⏟
𝑡𝑟𝑜𝑙𝑙𝑒𝑦 𝑓𝑟𝑜𝑚 𝑟𝑒𝑠𝑡 until it has a speed
⏟ of 10 ℎ𝑜𝑢𝑟 .
𝑆𝑡𝑎𝑡𝑒 𝐴 𝑆𝑡𝑎𝑡𝑒 𝐵
A. Is there a change in energy for the trolley from State A to State B?
B. What caused it?
Part A
𝑆𝑡𝑎𝑡𝑒 𝐴: 𝑇𝑟𝑜𝑙𝑙𝑒𝑦 ℎ𝑎𝑠 𝑛𝑜 𝑒𝑛𝑒𝑟𝑔𝑦
𝑆𝑡𝑎𝑡𝑒 𝐵: 𝑇𝑟𝑜𝑙𝑙𝑒𝑦 ℎ𝑎𝑠 𝐾𝑖𝑛𝑒𝑡𝑖𝑐 𝐸𝑛𝑒𝑟𝑔𝑦
Part B
𝑊𝑜𝑟𝑘 𝑑𝑜𝑛𝑒 𝑜𝑛 𝑡ℎ𝑒 𝑡𝑟𝑜𝑙𝑙𝑒𝑦 𝑖𝑛𝑐𝑟𝑒𝑎𝑠𝑒𝑑 𝑡ℎ𝑒 𝑘𝑖𝑛𝑒𝑡𝑖𝑐 𝑒𝑛𝑒𝑟𝑔𝑦
𝑊𝑜𝑟𝑘 = Δ𝐸
F. Triangle Inequality
In the 2-dimensional case, the above statement is equal to the statement from geometry:
𝑇ℎ𝑒 𝑙𝑒𝑛𝑔𝑡ℎ 𝑜𝑓𝑡ℎ𝑒 𝑡ℎ𝑖𝑟𝑑 𝑠𝑖𝑑𝑒 𝑜𝑓 𝑎 𝑡𝑟𝑖𝑎𝑛𝑔𝑙𝑒 𝑖𝑠 𝑙𝑒𝑠𝑠 𝑡ℎ𝑎𝑛 𝑡ℎ𝑒 𝑠𝑢𝑚 𝑜𝑓 𝑡ℎ𝑒 𝑜𝑡ℎ𝑒𝑟 𝑡𝑤𝑜 𝑠𝑖𝑑𝑒𝑠.
We can demonstrate by drawing a vector triangle.
Note that equality is achieved exactly when the two vectors both point in the same direction.
Assumptions
Let
⃗⃗ = (𝑎1 , 𝑎2 , … , 𝑎𝑛 ),
𝒙 ⃗⃗ = (𝑏1 , 𝑏2 , … , 𝑏𝑛 )
𝒚
We can add the two vectors above to get:
⃗⃗ = (𝑎1 + 𝑏1 , 𝑎2 + 𝑏2 , … , 𝑎𝑛 + 𝑏𝑛 )
⃗⃗ + 𝒚
𝒙
Left Hand Side
Using the definition of magnitude, the LHS is:
|𝒙 ⃗⃗| = √(𝑎1 + 𝑏1 )2 + (𝑎2 + 𝑏2 )2 + ⋯ + (𝑎𝑛 + 𝑏𝑛 )2
⃗⃗ + 𝒚
Square both sides of the above:
(|𝒙 ⃗⃗|)2 = (𝑎1 + 𝑏1 )2 + (𝑎2 + 𝑏2 )2 + ⋯ + (𝑎𝑛 + 𝑏𝑛 )2
⃗⃗ + 𝒚
Expand the RHS to get:
= (𝑎12 + 2𝑎1 𝑏1 + 𝑏12 ) + (𝑎22 + 2𝑎2 𝑏2 + 𝑏22 ) + ⋯ + (𝑎𝑛2 + 2𝑎𝑛 𝑏𝑛 + 𝑏𝑛2 )
P a g e 31 | 168
Get all the files at: https://bit.ly/azizhandouts
Aziz Manva (azizmanva@gmail.com)
Rearrange to get:
2 2 2 2
⏟ 1 + 𝑎2 + ⋯ + 𝑎𝑛2 ) + (𝑏1 + 𝑏2 + ⋯ + 𝑏𝑛2 ) + (2𝑎1 𝑏1 + 2𝑎2 𝑏2 + ⋯ + 2𝑎𝑛 𝑏𝑛 )
= (𝑎
𝑪𝒂𝒍𝒍 𝒕𝒉𝒊𝒔 𝑿
Right Hand Side
The square of the right-hand side is:
⃗⃗|)2 = |𝒙
⃗⃗| + |𝒚
(|𝒙 ⃗⃗|2 + |𝒚
⃗⃗|2 + 2|𝒙
⃗⃗||𝒚
⃗⃗|
Substitute using the definition of magnitude:
2 2 2 2
⏟ 1 + 𝑎2 + ⋯ + 𝑎𝑛2 ) + (𝑏1 + 𝑏2 + ⋯ + 𝑏𝑛2 ) + 2√𝑎12 + 𝑎22 + ⋯ + 𝑎𝑛2 √𝑏12 + 𝑏22 + ⋯ + 𝑏𝑛2
= (𝑎
𝑪𝒂𝒍𝒍 𝒕𝒉𝒊𝒔 𝑿
Bringing it together
Comparing the two above, we see that X is common to both. Hence, we need to show inequality in the remaining
terms. Start with the Cauchy-Schwarz inequality:
(𝑎1 𝑏1 + 𝑎2 𝑏2 + ⋯ + 𝑎𝑛 𝑏𝑛 )2 ≤ (𝑎12 + 𝑎22 + ⋯ + 𝑎𝑛2 )(𝑏12 + 𝑏22 + ⋯ + 𝑏𝑛2 )
Take the square root both sides and multiply by 2:
2(𝑎1 𝑏1 + 𝑎2 𝑏2 + ⋯ + 𝑎𝑛 𝑏𝑛 ) ≤ 2√𝑎12 + 𝑎22 + ⋯ + 𝑎𝑛2 √𝑏12 + 𝑏22 + ⋯ + 𝑏𝑛2
And the above is exactly what we need.
However, when we come to vector multiplication, there are different kinds of multiplication.
⃗⃗ → 𝑆𝑐𝑎𝑙𝑎𝑟 𝑀𝑢𝑙𝑡𝑖𝑝𝑙𝑖𝑐𝑎𝑡𝑖𝑜𝑛
𝜆𝒂
For scalar multiplication, the input is a scalar and a vector, and the output is a vector.
B. Basics
Scalar multiplication:
➢ Is achieved by multiplying each component of the vector by 𝜆.
➢ Changes the magnitude of a vector (𝜆 ≠ 1).
P a g e 32 | 168
Get all the files at: https://bit.ly/azizhandouts
Aziz Manva (azizmanva@gmail.com)
1 2 3 4
𝜆𝒂 = (2,3,4) = ( , , )
5 5 5 5
𝜆𝒂 ⃗⃗ = 𝒄
⃗⃗ + 𝜇𝒃 ⃗⃗
2 3 4
𝜆( )+𝜇( ) = ( )
3 4 5
2𝜆 3𝜇 4
( )+( ) = ( )
3𝜆 4𝜇 5
2𝜆 + 3𝜇 4
( )=( )
3𝜆 + 4𝜇 5
Example 1.113
The position vectors of point A, B and C are:
⃗⃗⃗⃗⃗⃗⃗ = (4,7),
𝑶𝑨 ⃗⃗⃗⃗⃗⃗⃗ = (−2,3),
𝑶𝑩 ⃗⃗⃗⃗⃗⃗⃗ = (6, −4)
𝑶𝑪
A. Vector 𝒂 is parallel to 𝑶𝑨 and has magnitude twice of 𝑂𝐴. Find the position vector of 𝒂.
B. Find the vector 𝒃 which is parallel to BC and has thrice the magnitude of BC.
Part A
⃗⃗ = 2(𝑶𝑨) = 2(4,7) = (8,14)
𝒂
Part B
6 −2 8
𝑩𝑪 = 𝑶𝑪 − 𝑶𝑩 = ( )−( ) = ( )
−4 3 −7
8 24
𝒃 = 3𝑩𝑪 = 3 ( ) = ( )
−7 −21
1 Unit Vector is a very important concept, and gets used in many places going forward.
P a g e 33 | 168
Get all the files at: https://bit.ly/azizhandouts
Aziz Manva (azizmanva@gmail.com)
➢ Multiplying a vector with the reciprocal of its 𝑙𝑒𝑛𝑔𝑡ℎ scales the vector, and gives us a unit vector.
Example 1.115
Find the unit vector that is in the same direction as the vector from 𝐴(6,8) to 𝐵(14,23).
⃗⃗⃗⃗⃗⃗⃗ is:
The unit vector in the direction of 𝑨𝑩
⃗⃗⃗⃗⃗⃗⃗
𝑨𝑩 (8,15) 8 15
= =( , )
⃗⃗⃗⃗⃗⃗⃗|
|𝑨𝑩 17 17 17
Example 1.117
2 7
The position vectors of points X and Y are ( ) and ( ) respectively.
−4 3
⃗⃗⃗⃗⃗⃗|.
A. Find |𝑿𝒀
⃗⃗⃗⃗⃗⃗)| ∈ ℕ
B. Find the smallest value of 𝜇 > 1 such that |𝜇(𝑿𝒀
⃗⃗⃗⃗⃗⃗.
C. Find the unit vectors parallel and antiparallel to 𝑿𝒀
Part A
⃗⃗⃗⃗⃗⃗ 7 2 5
𝑿𝒀 = ( ) − ( ) = ( )
3 −4 7
|𝑿𝒀| = √52 + 72 = √25 + 49 = √74
Part B
𝜇 = √74
Part C
⃗⃗⃗⃗⃗⃗ is:
Unit vector parallel to 𝑿𝒀
15
( )
√74 7
⃗⃗⃗⃗⃗⃗ is:
Unit vector antiparallel to 𝑿𝒀
1 5
− ( )
√74 7
P a g e 34 | 168
Get all the files at: https://bit.ly/azizhandouts
Aziz Manva (azizmanva@gmail.com)
Part A
The length of the vector is:
√52 + 122 = √169 = 13 (𝑃𝑦𝑡ℎ𝑎𝑔𝑜𝑟𝑒𝑎𝑛 𝑇𝑟𝑖𝑝𝑙𝑒𝑡: 5,12,13)
Then:
1 5 12
⃗⃗ = (13) (
𝒂 ) (5,12) = (13)
⏟ ( , )
13 ⏟13 13
𝑳𝒆𝒏𝒈𝒕𝒉
𝑼𝒏𝒊𝒕 𝑽𝒆𝒄𝒕𝒐𝒓
Part B
1 3 4
⃗𝒃⃗ = (3,4) = (5) ( ) (3,4) = (5) ( , )
5 5 5
Part C
8 15
⃗⃗ = (17) (
𝒄 , )
17 17
Part D
√42 + 52 4 5
⃗𝒅⃗ = ( ) (4,5) = √41 ( , )
√42 + 52 √41 √41
Example 1.121
⃗⃗ makes an angle of 34° with the positive direction of the 𝑥 −axis. Find the angle made by −𝒂
Position vector 𝒂 ⃗⃗
with the positive direction of 𝑥 − 𝑎𝑥𝑖𝑠.
C. Parallel Vectors
P a g e 35 | 168
Get all the files at: https://bit.ly/azizhandouts
Aziz Manva (azizmanva@gmail.com)
Example 1.124
The non-zero vectors 𝒂 ⃗⃗ and 𝒄
⃗⃗, 𝒃 ⃗⃗ are related by 𝒂 ⃗⃗, and 𝒄
⃗⃗ = 8𝒃 ⃗⃗. Then, the angle between 𝒂
⃗⃗ = −7𝒃 ⃗⃗ and 𝒄
⃗⃗ is: (JEE
Main 2008)
⃗⃗, 𝒂
⃗⃗ is a scalar multiple of 𝒃
Since 𝒂 ⃗⃗.
⃗⃗ is parallel to 𝒃
⃗⃗
⃗⃗ 𝑔𝑜𝑒𝑠 𝑖𝑛 𝑡ℎ𝑒 𝑠𝑎𝑚𝑒 𝑑𝑖𝑟𝑒𝑐𝑡𝑖𝑜𝑛 𝑎𝑠 𝒃
𝒂
⃗⃗ and 𝒄
So, angle between 𝒂 ⃗⃗ is 𝜋.
Example 1.125
If vectors ⃗⃗⃗⃗⃗ ̂ and ⃗⃗⃗⃗⃗
𝒂𝟏 = 𝑥𝒊̂ − 𝒋̂ + 𝒌 ̂ are collinear, then a possible unit vector parallel to the vector
𝒂𝟐 = 𝒊̂ + 𝑦𝒋̂ + 𝑧𝒌
𝑥𝒊̂ + 𝑦𝒋̂ + 𝑧𝒌̂ is:
1
A. (−𝒋̂ + 𝒌̂)
√2
1
B. (𝒊̂ − 𝒋̂)
√2
1
C. (𝒊̂ ̂)
+ 𝒋̂ − 𝒌
√3
1
D. (𝒊̂ ̂) (JEE Main 2021, 26 Feb, Shift-II)
− 𝒋̂ + 𝒌
√3
Since the two vectors are collinear, one must a non-zero scalar multiple of the other. Let
⃗⃗⃗⃗⃗ ⃗⃗⃗⃗⃗𝟐
𝒂𝟏 = 𝜆𝒂
(𝑥, −1,1) = 𝜆(1, 𝑦, 𝑧)
(𝑥, −1,1) = (𝜆, 𝜆𝑦, 𝜆𝑧)
1 1
Equate components to get 𝑥 = 𝜆, 𝑦 = − 𝜆 , 𝑧 = 𝜆 and substitute in 𝑥𝒊̂ + 𝑦𝒋̂ + 𝑧𝒌̂ to get:
1 1 1
̂ = (𝜆2 𝒊̂ − 𝒋̂ + 𝒌
= 𝜆𝒊̂ + (− ) 𝒋̂ + ( ) 𝒌 ̂)
𝜆 𝜆 𝜆
̂ is:
The magnitude of 𝑥𝒊̂ + 𝑦𝒋̂ + 𝑧𝒌
1 2 1 2 1 1 𝜆4 + 2 1 4
√𝜆2 + (− ) + ( ) = √𝜆2 + + = √ = √𝜆 + 2
𝜆 𝜆 𝜆2 𝜆2 𝜆2 𝜆
̂ is:
A unit vector in the direction of 𝑥𝒊̂ + 𝑦𝒋̂ + 𝑧𝒌
P a g e 36 | 168
Get all the files at: https://bit.ly/azizhandouts
Aziz Manva (azizmanva@gmail.com)
1 2 ̂) (𝜆2 𝒊̂ − 𝒋̂ + 𝒌
𝒊̂ − 𝒋̂ + 𝒌 ̂)
𝜆 (𝜆 =
1 4 √𝜆4 + 2
+2
𝜆 √𝜆
We do not know the value of the parameter 𝜆. Any non-zero value will work. Try:
̂ 𝒊̂ − 𝒋̂ + 𝒌
𝒊̂ − 𝒋̂ + 𝒌 ̂
𝜆=1⇒ = ⇒ 𝑂𝑝𝑡𝑖𝑜𝑛 𝐷
√1 + 2 √3
Example 1.126
A. Let 𝒂 ⃗⃗ and 𝒄
⃗⃗, 𝒃 ⃗⃗ be three non-zero vectors which are pairwise non-collinear. If 𝒂 ⃗⃗ + 3𝒃⃗⃗ is collinear with 𝒄 ⃗⃗
and 𝒃 ⃗⃗ + 2𝒄 ⃗⃗ is collinear with 𝒂 ⃗⃗, then 𝒂 ⃗⃗ + 6𝒄
⃗⃗ + 3𝒃 ⃗⃗ is: (JEE Main 2011)
B. 𝒂 ⃗⃗ and 𝒄
⃗⃗, 𝒃 ⃗⃗ are three non-zero vectors such that no two of these are collinear. If the vector 𝒂 ⃗⃗ + 2𝒃⃗⃗ is
collinear with ⃗𝒄⃗ and ⃗𝒃⃗ + 3𝒄 ⃗⃗ is collinear with ⃗𝒂⃗ (𝜆 being some non-zero scalar), then ⃗𝒂⃗ + 2𝒃⃗⃗ + 6𝒄⃗⃗ is: (JEE
Main 2004)
Part A
From the collinearity conditions:
𝒂 ⃗⃗ = 𝜆𝒄
⃗⃗ + 3𝒃 ⃗⃗ ⇒ 𝒂
⃗⏟ ⃗⃗ + 6𝒄
⃗ + 3𝒃 ⃗⃗ = (𝜆 + 6)𝒄
⃗⃗
𝑬𝒒𝒖𝒂𝒕𝒊𝒐𝒏 𝑰
⃗𝒃⃗ + 2𝒄
⃗⃗ = 𝜇𝒂 ⃗⃗ + 6𝒄
⃗⃗ ⇒ 3𝒃 ⃗⃗ = 3𝜇𝒂
⃗⃗ ⇒ ⏟ ⃗⃗ + 6𝒄
⃗⃗ + 3𝒃
𝒂 ⃗⃗ = (1 + 3𝜇)𝒂
⃗⃗
𝑬𝒒𝒖𝒂𝒕𝒊𝒐𝒏 𝑰𝑰
From Equation I and II:
(𝜆 + 6)𝒄 ⃗⃗ = (1 + 3𝜇)𝒂 ⃗⃗
But the vectors are non-collinear and non-zero. Hence:
𝜆 + 6 = 1 + 3𝜇 = 0
⃗⃗
⃗⃗ + 3𝒃 + 6𝒄
𝒂 ⃗⃗ = (𝜆 + 6)𝒄 ⃗⃗ = 0𝒄 ⃗⃗ = 0
Part B
From the collinearity conditions:
⃗⃗ + 2𝒃
𝒂 ⃗⃗ = 𝜆𝒄 ⃗⃗ ⇒ ⏟⃗⃗ + 2𝒃
𝒂 ⃗⃗ + 6𝒄⃗⃗ = (𝜆 + 6)𝒄⃗⃗
𝑬𝒒𝒖𝒂𝒕𝒊𝒐𝒏 𝑰
⃗𝒃⃗ + 3𝒄
⃗⃗ = 𝜇𝒂 ⃗⃗ + 6𝒄
⃗⃗ ⇒ 2𝒃 ⃗⃗ = 2𝜇𝒂
⃗⃗ ⇒ ⏟ ⃗⃗ + 6𝒄
⃗⃗ + 2𝒃
𝒂 ⃗⃗ = (1 + 2𝜇)𝒂
⃗⃗
𝑬𝒒𝒖𝒂𝒕𝒊𝒐𝒏 𝑰𝑰
From Equation I and II:
(𝜆 + 6)𝒄 ⃗⃗ = (1 + 2𝜇)𝒂⃗⃗
But the vectors are non-collinear and non-zero. Hence:
𝜆 + 6 = 1 + 2𝜇 = 0
⃗⃗
⃗⃗ + 3𝒃 + 6𝒄
𝒂 ⃗⃗ = (𝜆 + 6)𝒄⃗⃗ = 0𝒄⃗⃗ = 0
P a g e 37 | 168
Get all the files at: https://bit.ly/azizhandouts
Aziz Manva (azizmanva@gmail.com)
D. Midpoint Formula
Example 1.127
⃗⃗⃗⃗⃗⃗⃗ = (3, −2), ⃗⃗⃗⃗⃗⃗
Given that 𝑨𝑩 ⃗⃗⃗⃗⃗⃗, where X is the midpoint of BC.
𝑨𝑪 = (−2,4), determine 𝑪𝑿
𝟏 𝟏 𝟏
⃗⃗⃗⃗⃗⃗ = −𝑩𝑿
𝑪𝑿 ⃗⃗⃗⃗⃗⃗⃗ + 𝑨𝑪
⃗⃗⃗⃗⃗⃗⃗ = − (𝑩𝑨 ⃗⃗⃗⃗⃗⃗) = − (−𝑨𝑩
⃗⃗⃗⃗⃗⃗⃗ + 𝑨𝑪
⃗⃗⃗⃗⃗⃗) = (𝑨𝑩
⃗⃗⃗⃗⃗⃗⃗ − 𝑨𝑪
⃗⃗⃗⃗⃗⃗)
𝟐 𝟐 𝟐
1 3 1 5 5
−2
= [( ) − ( )] = [( )] = ( 2 )
2 −2 4 2 −6
−3
Using Vectors
From the triangle:
⃗⃗⃗⃗⃗⃗⃗ = 𝑨𝑪
𝑨𝑩 ⃗⃗⃗⃗⃗⃗ + 𝑪𝑩 ⃗⃗ + 𝒂
⃗⃗⃗⃗⃗⃗⃗ = −𝒃 ⃗⃗ = 𝒂 ⃗⃗
⃗⃗ − 𝒃
And since 𝑋 is the midpoint of 𝐴𝐵:
1 1
⃗⃗⃗⃗⃗⃗⃗
𝑨𝑿 = ⃗⃗⃗⃗⃗⃗⃗ ⃗⃗ − ⃗𝒃⃗)
𝑨𝑩 = (𝒂
2 2
Finally, the expression that we want is:
1 1 𝟏 1 1 ⃗⃗ + ⃗𝒃⃗
𝒂
⃗⃗⃗⃗⃗⃗ = ⃗⃗⃗⃗⃗⃗
𝑪𝑿 𝑨𝑿 = ⃗𝒃⃗ + (𝒂
𝑪𝑨 + ⃗⃗⃗⃗⃗⃗⃗ ⃗⃗ − ⃗𝒃⃗) = ⃗𝒃⃗ + 𝒂⃗⃗ − ⃗𝒃⃗ = ⃗𝒃⃗ + 𝒂 ⃗⃗ =
2 2 𝟐 2 2 2
Coordinate Geometry
Introduce an origin at 𝐶 = (0,0). Let the position vectors of A and B be:
𝐴(𝑥1 , 𝑦1 ) 𝑎𝑛𝑑 𝐵(𝑥2 , 𝑦2 )
By the midpoint formula:
𝑥1 + 𝑥2 𝑦1 + 𝑦2 1 1 1 1
𝑋=( , ) = (𝑥1 + 𝑥2 , 𝑦1 + 𝑦2 ) = [(𝑥1 , 𝑦1 ) + (𝑥2 , 𝑦2 )] = [𝑪𝑨 ⃗⃗⃗⃗⃗⃗ + 𝑪𝑩 ⃗⃗ + 𝒂
⃗⃗⃗⃗⃗⃗⃗] = [𝒃 ⃗⃗]
2 2 2 2 2 2
P a g e 38 | 168
Get all the files at: https://bit.ly/azizhandouts
Aziz Manva (azizmanva@gmail.com)
Example 1.130
X is the midpoint of CD in pentagon 𝐴𝐵𝐶𝐷𝐸. Using the midpoint formula,
write the vectors below in terms of vectors that have start and end points
on consecutive vertices of the pentagon:
A. ⃗⃗⃗⃗⃗⃗⃗
𝑨𝑿 (without using ⃗⃗⃗⃗⃗⃗⃗
𝑪𝑫)
B. 𝑬𝑿 (without using ⃗⃗⃗⃗⃗⃗⃗
⃗⃗⃗⃗⃗⃗⃗ 𝑪𝑫)
⃗⃗⃗⃗⃗⃗⃗
C. 𝑩𝑿(without using 𝑪𝑫) ⃗⃗⃗⃗⃗⃗⃗
𝟏
⃗⃗⃗⃗⃗⃗ + ⃗⃗⃗⃗⃗⃗⃗
D. Simplify (𝑪𝑨 𝑨𝑫)
𝟐
Part A
1 1
⃗⃗⃗⃗⃗⃗⃗
𝑨𝑿 = (𝑨𝑪 ⃗⃗⃗⃗⃗⃗ + ⃗⃗⃗⃗⃗⃗⃗
𝑨𝑫) = (𝑨𝑩 ⃗⃗⃗⃗⃗⃗⃗ + ⃗⃗⃗⃗⃗⃗⃗
𝑩𝑪 + ⃗⃗⃗⃗⃗⃗
𝑨𝑬 + ⃗⃗⃗⃗⃗⃗⃗
𝑬𝑫)
2 2
Part B
𝟏 𝟏
⃗⃗⃗⃗⃗⃗⃗
𝑬𝑿 = ⃗⃗⃗⃗⃗⃗ + ⃗⃗⃗⃗⃗⃗⃗
(𝑬𝑪 𝑬𝑫) = (𝑬𝑨 ⃗⃗⃗⃗⃗⃗ + 𝑨𝑩
⃗⃗⃗⃗⃗⃗⃗ + ⃗⃗⃗⃗⃗⃗⃗
𝑩𝑪 + ⃗⃗⃗⃗⃗⃗⃗
𝑬𝑫)
𝟐 𝟐
Part C
𝟏 𝟏
⃗⃗⃗⃗⃗⃗⃗
𝑩𝑿 = ⃗⃗⃗⃗⃗⃗⃗ + 𝑩𝑫
(𝑩𝑪 ⃗⃗⃗⃗⃗⃗⃗ + 𝑩𝑨
⃗⃗⃗⃗⃗⃗⃗) = (𝑩𝑪 ⃗⃗⃗⃗⃗⃗⃗ + ⃗⃗⃗⃗⃗⃗
𝑨𝑬 + ⃗⃗⃗⃗⃗⃗⃗
𝑬𝑫)
𝟐 𝟐
Part D
𝟏 𝟏
⃗⃗⃗⃗⃗⃗ + ⃗⃗⃗⃗⃗⃗⃗
(𝑪𝑨 𝑨𝑫) = (𝑪𝑫 ⃗⃗⃗⃗⃗⃗⃗) = 𝑪𝑿
⃗⃗⃗⃗⃗⃗
𝟐 𝟐
Example 1.131
̂ and 𝑨𝑪
⃗⃗⃗⃗⃗⃗⃗ = 3𝒊̂ + 4𝒌
If the vectors 𝑨𝑩 ̂ are the sides of a Δ𝐴𝐵𝐶, then the length of the median
⃗⃗⃗⃗⃗⃗ = 5𝒊̂ − 2𝒋̂ + 4𝒌
through 𝐴 is: (JEE Main 2003, 2013)
⃗⃗⃗⃗⃗⃗⃗ + 𝑨𝑪
𝑨𝑩 ⃗⃗⃗⃗⃗⃗ (3,0,4) + (5, −2,4)
𝑨𝑴 = = =, (4, −1,4)
2 2
|𝑨𝑴| = √42 + (−1)2 + 42 = √33
Example 1.132
Show, using the midpoint formula, that the sum of the medians of a triangle, with the vertices as the start point
for each median, is zero.
P a g e 39 | 168
Get all the files at: https://bit.ly/azizhandouts
Aziz Manva (azizmanva@gmail.com)
1.133: Force
⃗𝑭⃗ = 𝑚𝒂
⃗⃗
Force is a vector quantity given by the product of mass and acceleration where:
➢ Mass is a scalar quantity (with unit, for example, 𝑘𝑔)
𝑚
➢ Acceleration is a vector quantity (with unit, for example, 𝑠2 )
Example 1.134
𝑚
A. On a frictionless surface, an object of mass 3 kg is accelerated by 2 𝑠2 . Calculate the force in Newtons.
B. A force of 72 Newtons accelerates an object with an integer number of kg in its mass on a frictionless
surface an integer number of meters per second squared. Calculate the product of the possible values of
the mass of the object.
C. A spaceship with a mass of 107 kg fires a thruster that pushes the spaceship with a force of 72 Newtons.
𝑚
Calculate the acceleration of the spaceship in 𝑠2 . Write your answer in scientific notation.
Part A
⃗⃗ = 𝑚𝒂
𝑭 ⃗⃗ = (3)(2) = 6𝑁
Part B
{1 × 72,2 × 36,3 × 24,4 × 18,6 × 12, 8 × 9}
726 = (23 × 32 )6 = 218 × 312
You can also do this using the formula for product of factors of a number. The product of factors of 72
𝜏(72) 𝜏(23 ×32) (3+1)(2+1)
= 72 2 = 72 2 = 72 2 = 726 = 218 × 312
Part C
72 = 107 × 𝒂
⃗⃗
72 𝑚
⃗⃗ =
𝒂 7
= 72 × 10−7 = 7.2 × 10−6 2
10 𝑠
2 This works because the equations are cyclical. Adding them gives a zero.
P a g e 40 | 168
Get all the files at: https://bit.ly/azizhandouts
Aziz Manva (azizmanva@gmail.com)
Example 1.135
2 3
Relative to an origin 𝑂, the position vectors of points 𝑋 and 𝑌 are given by ( ) and ( ). 𝑋, 𝑌 and 𝑍 lie on a
3 2
straight line such that the length of 𝑿𝒁 ⃗⃗⃗⃗⃗⃗ is four times the length of 𝑿𝒀
⃗⃗⃗⃗⃗⃗. The sum of the possible values of the
⃗⃗⃗⃗⃗⃗⃗ can be written as √𝑎 + √𝑏. Find the sum of the digits of 𝑎 + 𝑏.
length of 𝑶𝒁
There are two possible locations for Z, based on which direction we go in,
giving us two cases:
⃗⃗⃗⃗⃗⃗ = ±4 [(3) − (2)] = ±𝟒 ( 1 ) = ± ( 4 )
⃗⃗⃗⃗⃗⃗ = ±4𝑿𝒀
𝑿𝒁
⏟ 2 3 −1 −4
𝑬𝒒𝒖𝒂𝒕𝒊𝒐𝒏 𝑰𝑰
⃗⃗⃗⃗⃗⃗⃗ are
The possible values for the length of 𝑶𝒁
{√37, √53}
P a g e 41 | 168
Get all the files at: https://bit.ly/azizhandouts
Aziz Manva (azizmanva@gmail.com)
Draw Δ𝑋𝑌𝑍.
Let 𝑃 be the midpoint of 𝑋𝑌, and 𝑄 be the midpoint of 𝑋𝑍. Then:
⃗⃗⃗⃗⃗⃗ = 𝑷𝑿
𝒀𝑷 ⃗⃗⃗⃗⃗⃗⃗ = 𝒂
⃗⃗ (𝑠𝑎𝑦), ⃗⃗⃗⃗⃗⃗⃗ = 𝑸𝒁
𝑿𝑸 ⃗⃗ (𝑠𝑎𝑦),
⃗⃗⃗⃗⃗⃗⃗ = 𝒃 ⃗⃗⃗⃗⃗⃗⃗ = 𝒄
𝑷𝑸 ⃗⃗
Because
𝑌𝑃 = 𝑃𝑋 𝑠𝑖𝑛𝑐𝑒 𝑃 𝑖𝑠 𝑡ℎ𝑒 𝑚𝑖𝑑𝑝𝑜𝑖𝑛𝑡 ⇒ 𝑆𝑎𝑚𝑒 𝑀𝑎𝑔𝑛𝑖𝑡𝑢𝑑𝑒
𝑌, 𝑃, 𝑋 𝑎𝑟𝑒 𝑐𝑜𝑙𝑙𝑖𝑛𝑒𝑎𝑟 ⇒ 𝑆𝑎𝑚𝑒 𝐷𝑖𝑟𝑒𝑐𝑡𝑖𝑜𝑛
Note that:
⃗⃗⃗⃗⃗⃗⃗ = 𝒄
𝑷𝑸 ⃗⃗ = 𝒂⃗⃗ + 𝒃⃗⃗
⃗⃗⃗⃗⃗⃗ = 𝒀𝑿
𝒀𝒁 ⃗⃗⃗⃗⃗⃗ + 𝑿𝒁
⃗⃗⃗⃗⃗⃗ = 2𝒂 ⃗⃗ = 2(𝒂
⃗⃗ + 2𝒃 ⃗⃗) = 2𝒄
⃗⃗ + 𝒃 ⃗⃗
Hence,
𝑌𝑍 𝑖𝑠 𝑑𝑜𝑢𝑏𝑙𝑒 𝑜𝑓 𝑃𝑄
𝑌𝑍 𝑖𝑠 𝑝𝑎𝑟𝑎𝑙𝑙𝑒𝑙 𝑡𝑜 𝑃𝑄
Example 1.138
Show, without using the midpoint theorem, that the quadrilateral formed by the midpoints of a convex
quadrilateral, taken in order, is a parallelogram.
Hint: A quadrilateral is a parallelogram if a pair of its opposite sides is parallel and congruent.
P a g e 42 | 168
Get all the files at: https://bit.ly/azizhandouts
Aziz Manva (azizmanva@gmail.com)
𝜆𝒂 ⃗⃗ = 𝒄
⃗⃗ + 𝜇𝒃 ⃗⃗
➢ It is called a linear combination since there are no powers (squares, cubes, etc) involved.
Example 1.140
Find the linear combination of the first two vectors that results in the third vector:
1 2 8
A. 𝒂⃗⃗ = ( ) , ⃗𝒃⃗ = ( ) , 𝒄
⃗⃗ = ( )
3 5 21
Consider Option G:
𝜆 = 𝜇 = 0 ⇒ 𝑉𝑎𝑙𝑖𝑑 𝑆𝑜𝑙𝑢𝑡𝑖𝑜𝑛 ⇒ 𝐺 𝑖𝑠 𝑛𝑜𝑡 𝑐𝑜𝑟𝑟𝑒𝑐𝑡
𝐶𝑜𝑟𝑟𝑒𝑐𝑡 𝑂𝑝𝑡𝑖𝑜𝑛𝑠 𝐶, 𝐷, 𝐻, 𝐼
Example 1.143
⃗⃗ = 𝑝𝒂
⃗⃗ + 𝑛𝒃
What can we conclude about the coefficients if 𝑚𝒂 ⃗⃗?
⃗⃗ + 𝑞𝒃
Since the vectors on both sides are equal, the coefficients must also be the same.
𝑚=𝑝
𝑛=𝑞
Example 1.144
⃗⃗⃗ = (𝜆 − 2)𝒂
Let 𝜶 ⃗⃗ and 𝜷
⃗⃗ + 𝒃 ⃗⃗⃗ = (4𝜆 − 2)𝒂 ⃗⃗ be two given vectors where 𝒂
⃗⃗ + 3𝒃 ⃗⃗ are non-collinear. The value
⃗⃗ and 𝒃
P a g e 43 | 168
Get all the files at: https://bit.ly/azizhandouts
Aziz Manva (azizmanva@gmail.com)
Since the vectors on both sides are equal, the coefficients must also be the same.
1
1 = 3𝜇 ⇒ 𝜇 =
3
𝜆 − 2 = 𝜇(4𝜆 − 2)
1
𝜆 − 2 = (4𝜆 − 2)
3
3𝜆 − 6 = 4𝜆 − 2
𝜆 = −4
H. Linear Combination: Geometric
Example 1.145
Show that the sum of the medians of a triangle, taken with the vertices as the start point, is zero.
Example 1.146
Show that the diagonals of a quadrilateral bisect each other if and only if the quadrilateral is a parallelogram.
P a g e 44 | 168
Get all the files at: https://bit.ly/azizhandouts
Aziz Manva (azizmanva@gmail.com)
𝑴𝑪 = 𝑨𝑴
⏟ 𝑨𝑪 = 𝒂 + 𝒃 ⇒ ⏟
𝑨𝑴 = 𝜆(𝒂 + 𝒃)
𝑬𝒒𝒖𝒂𝒕𝒊𝒐𝒏 𝟐 𝑬𝒒𝒖𝒂𝒕𝒊𝒐𝒏 𝑰𝑰
Example 1.147
In Δ𝑋𝑌𝑍, lines 𝑍𝑃 and 𝑍𝑄 trisect 𝑋𝑌. The points 𝑌, 𝑃, 𝑄, 𝑋 are collinear, in that order. The point of intersection of
𝑍𝑃 with the median 𝑋𝑀 is point A. Find the ratio in which point 𝐴 divides 𝑋𝑀, and also the ratio in which point
𝐴 divides 𝑍𝑃.
Substitute:
= 𝒂 + 𝜆(3𝒃 − 𝒂) = 𝒂 + 3𝜆𝒃 − 𝜆𝒂 = 𝒂(1
⏟ − 𝜆) + 3𝜆𝒃
𝑬𝒙𝒑𝒓𝒆𝒔𝒔𝒊𝒐𝒏 𝑰
P a g e 45 | 168
Get all the files at: https://bit.ly/azizhandouts
Aziz Manva (azizmanva@gmail.com)
1 − 𝜆 = 2𝜇 ⇒ 𝜆 = 1 − 2𝜇 ⇒ 3𝜆 = 3 − 6𝜇
Equating coefficients for 𝑏:
2
3𝜆 = 1 − 𝜇 ⇒ 3 − 6𝜇 = 1 − 𝜇 ⇒ 𝜇 =
5
2 1
𝜆 = 1 − 2𝜇 = 1 − ( ) =
5 5
Example 1.148
A. (𝐿𝑒𝑓𝑡 𝐷𝑖𝑎𝑔𝑟𝑎𝑚) In Δ𝐴𝐵𝐶, point 𝐹
divides side 𝐴𝐶 in the ratio 1:2. Let 𝐸 be
the point of intersection of side 𝐵𝐶 and
𝐴𝐺 where 𝐺 is the midpoint of 𝐵𝐹. The
point 𝐸 divides side 𝐵𝐶 in the ratio ____
(AHSME 1971/26)
B. (𝑅𝑖𝑔ℎ𝑡 𝐷𝑖𝑎𝑔𝑟𝑎𝑚) In triangle 𝐴𝐵𝐶, point
𝐷 divides side 𝐴𝐶 so that 𝐴𝐷: 𝐷𝐶 = 1: 2. Let 𝐸 be the midpoint of 𝐵𝐷 and let 𝐹 be the point of
intersection of line 𝐵𝐶 and line 𝐴𝐸. Given that the area of △ 𝐴𝐵𝐶 is 360, what is the area of △ 𝐸𝐵𝐹?
(AMC 8 2019/24)
Part A 1: 3
Let Part B
⃗⃗⃗⃗⃗⃗ = ⃗𝒂⃗ ⇒ ⃗⃗⃗⃗⃗⃗
𝑭𝑨 𝑪𝑭 = ⃗⃗⃗⃗⃗⃗
𝟐𝒂
⃗⃗⃗⃗⃗⃗⃗ ⃗⃗⃗⃗⃗⃗
𝑩𝑮 = 𝑮𝑭 = 𝒃 ⃗⃗
Then:
⃗⃗⃗⃗⃗⃗ = 𝑮𝑭
𝑮𝑨 ⃗⃗⃗⃗⃗⃗ + 𝑭𝑨
⃗⃗⃗⃗⃗⃗ = 𝒂 ⃗⃗ ⇒ 𝑬𝑮
⃗⃗ + 𝒃 ⃗⃗⃗⃗⃗⃗ = 𝜇(𝒂 ⃗⃗)
⃗⃗ + 𝒃
⃗⃗⃗⃗⃗⃗⃗ = 𝑩𝑭
𝑩𝑪 ⃗⃗⃗⃗⃗⃗ = 2𝒃
⃗⃗⃗⃗⃗⃗⃗ + 𝑭𝑪 ⃗⃗ − 2𝒂 ⃗⃗ ⇒ 𝑩𝑬 ⃗⃗ − 2𝒂
⃗⃗⃗⃗⃗⃗⃗ = 𝜆(2𝒃 ⃗⃗)
Finally:
⃗⃗⃗⃗⃗⃗⃗ = 𝒃
𝑩𝑮 ⃗⃗
Note that we have the same triangle as in Part A
⃗⃗⃗⃗⃗⃗⃗ + 𝑬𝑮
𝑩𝑬 ⃗⃗⃗⃗⃗⃗ = 𝒃⃗⃗ (except for names of vertices).
𝜆(2𝒃 ⃗⃗ − 2𝒂 ⃗⃗) + 𝜇(𝒂 ⃗⃗ + 𝒃 ⃗⃗) = 𝒃⃗⃗ We will use the ratios from Part A:
⃗⃗ + ⃗𝒃⃗
𝑎(𝜇 − 2𝜆) + 𝑏(𝜇 + 2𝜆) = 0𝒂 1 1
𝐵𝐹 = 𝐵𝐶, 𝐸𝐹 = 𝐴𝐹
Equating coefficients: 4 3
𝜇 − 2𝜆 = 0 , 𝜇 + 2𝜆 = 1 Using theorems on area of triangles:
⏟ ⏟
1 1
𝐸𝑞𝑢𝑎𝑡𝑖𝑜𝑛 𝐼 𝐸𝑞𝑢𝑎𝑡𝑖𝑜𝑛 𝐼𝐼 𝐴(Δ𝐸𝐵𝐹) = 𝐴(Δ𝐴𝐵𝐶) × ×
Add Equations I and II: ⏟
4 ⏟
3
𝑅𝑎𝑡𝑖𝑜 𝑜𝑓 𝑅𝑎𝑡𝑖𝑜 𝑜𝑓
1 𝐵𝑎𝑠𝑒𝑠 𝐻𝑒𝑖𝑔ℎ𝑡𝑠
2𝜇 = 1 ⇒ 𝜇 =
2 Simplify:
1
Substitute 𝜇 = in Equation II: 1 1
2 𝐴(Δ𝐸𝐵𝐹) = 360 × × = 30
1 1 4 3
+ 2𝜆 = 1 ⇒ 𝜆 =
2 4
E divides BC in the ratio
P a g e 46 | 168
Example 1.149
In Δ𝐴𝐵𝐶, points 𝐷 and 𝐸 lie on 𝐵𝐶 and 𝐴𝐶, respectively. If 𝐴𝐷 and 𝐵𝐸 intersect at 𝑇
𝐴𝑇 𝐵𝑇 𝐶𝐷
so that 𝐷𝑇 = 3 and 𝐸𝑇 = 4, what is 𝐵𝐷? (AMC 10B 2004/20)
Let:
𝐸𝑇 = ⃗𝒂⃗ ⇒ 𝑇𝐵 = 3𝒂
⃗⃗
𝑇𝐷 = ⃗𝒃⃗ ⇒ 𝐴𝑇 = 3𝒃
⃗⃗
⃗⃗⃗⃗⃗⃗⃗
𝑪𝑫
Let ⃗⃗⃗⃗⃗⃗⃗ = 𝜇:
𝑫𝑩
⃗⃗⃗⃗⃗⃗⃗ = 𝜇𝑫𝑩
𝑪𝑫 ⃗⃗⃗⃗⃗⃗⃗ = 𝜇(𝑻𝑩
⃗⃗⃗⃗⃗⃗ − 𝑻𝑫
⃗⃗⃗⃗⃗⃗⃗) = 𝜇(4𝒂 ⃗⃗)
⃗⃗ − 𝒃
⃗⃗⃗⃗⃗⃗
𝑪𝑬
Let ⃗⃗⃗⃗⃗⃗ = 𝜆:
𝑨𝑬
⃗⃗⃗⃗⃗⃗ = 𝜆𝑨𝑬
𝑪𝑬 ⃗⃗⃗⃗⃗⃗ = 𝜆(𝑨𝑻
⃗⃗⃗⃗⃗⃗ − 𝑻𝑬 ⃗⃗ − 𝒂
⃗⃗⃗⃗⃗⃗) = 𝜆(3𝒃 ⃗⃗)
Calculate ⃗⃗⃗⃗⃗⃗⃗
𝑬𝑫 in two different ways in Quadrilateral 𝐸𝑇𝐶𝐷:
⃗⃗⃗⃗⃗⃗⃗
𝑬𝑫 = 𝒂 ⃗⃗ + ⃗𝒃⃗
⃗⃗⃗⃗⃗⃗⃗
𝑬𝑫 = 𝜆(3𝒃 ⃗⃗ − 𝒂
⃗⃗) + 𝜇(4𝒂⃗⃗ − ⃗𝒃⃗) = 𝒂
⃗⃗(4𝜇 − 𝜆) + ⃗𝒃⃗(−𝜇 + 3𝜆)
Equate coefficients:
4𝜇 − 𝜆 = 1 ⇒ ⏟ 12𝜇 − 3𝜆 = 3 , −𝜇 + 3𝜆 = 1
⏟
𝐸𝑞𝑢𝑎𝑡𝑖𝑜𝑛 𝐼 𝐸𝑞𝑢𝑎𝑡𝑖𝑜𝑛 𝐼𝐼
Add Equations I and II:
4
11𝜇 = 4 ⇒ 𝜇 =
11
I. Centroid
Example 1.150
Centroid of a triangle is 2/3 of the way along the median (from the vertices)
Example 1.151
Centroid of a triangle is the average of the position vectors of the vertices of the triangle.
Pending
J. Section Formula
Example 1.152
Find, using first principles.
Example 1.154
Get all the files at: https://bit.ly/azizhandouts
Aziz Manva (azizmanva@gmail.com)
We have also seen the geometric form of a vector where the vector is represented as a directed line segment.
There is one more form of a vector, where it is specified using the magnitude and angle.
➢ If the tail of the vector is at the origin, then the polar form of a vector gives the position of the tip in
polar coordinates. Hence, the two are related.
➢ Angle is considered:
✓ positive in the counterclockwise direction
✓ negative in the clockwise direction
Example 1.157
Plot the vectors given in polar form on the coordinate plane. Write them in component form. State the quadrant
in which the vectors lie.
A. 𝒂⃗⃗ = (2,30°)
B. ⃗𝒃⃗ = (5,45°)
C. 𝒄⃗⃗ = (√3, −60°)
𝜋
D. ⃗𝒅⃗ = (3, )3
𝜋
E. ⃗⃗ = (4, )
𝒂 6
F. ⃗⃗ = (7, − 𝜋)
𝒃 3
9𝜋
G. ⃗⃗ = (2, )
𝒄 4
P a g e 48 | 168
Get all the files at: https://bit.ly/azizhandouts
Aziz Manva (azizmanva@gmail.com)
Example 1.158
Let a vector 𝛼𝒊 + 𝛽𝒋 be obtained by rotating the vector √3𝒊 + 𝒋 by an angle 45° about the origin in counter
clockwise direction in the first quadrant. Then, the area of the triangle having vertices (𝛼, 𝛽), (0, 𝛽) and (0,0) is
equal to: (JEE Main 2021, 16 March, Shift-I)
Rotate the vector by 45° counterclockwise (middle diagram) and note that the magnitude remains 2.
The components are
𝛼 = 2 cos 75° , 𝛽 = 2 sin 75°
Draw the required triangle (rightmost diagram), and hence, the area of that triangle=
1 𝛼𝛽 (2 cos 75°)(2 sin 75°)
= ℎ𝑏 = =
2 2 2
Simplify:
= 2 cos 75° sin 75°
Use the double angle identity for sin 𝜃:
1
= sin 150° = sin 30° =
2
P a g e 49 | 168
Get all the files at: https://bit.ly/azizhandouts
Aziz Manva (azizmanva@gmail.com)
Example 1.160
Write the following position vectors in standard polar form. If the angles are given in radians, convert into
degrees. Also, state the quadrant in which they lie.
A. 𝒂 ⃗⃗ = (2, −30°)
B. 𝒃 ⃗⃗ = (5, −210°)
C. 𝒄 ⃗⃗ = (9,410°)
D. 𝒅 ⃗⃗ = (1, 𝜋)
2
𝜋
⃗⃗ = (10, − )
E. 𝒆 6
Example 1.161
A vector has a magnitude of 11, and makes an angle of 30° with the horizontal in the right direction. Determine
the exact value of its horizontal and vertical components.
𝐵𝐶 1 11
sin 30° = ⇒ 𝐵𝐶 = 𝐴𝐵 sin 30° = 𝐵𝐶 × =
𝐴𝐵 2 2
𝐴𝐶 √3 11√3
cos 30° = ⇒ 𝐴𝐶 = 𝐴𝐵 × cos 30° = 11 × =
𝐴𝐵 2 2
11√3 11
( , )
2 2
Example 1.162
𝑚𝑖𝑙𝑒𝑠
A. Rory the dog is running towards his favorite ball on the beach, at a velocity of 34 at an angle of 𝛼 to
ℎ𝑜𝑢𝑟
the positive direction of 𝑥 axis. Find his speed in the horizontal and vertical direction.
B. The 𝑥 component of 𝒂 ⃗⃗ is −3. 𝒂
⃗⃗ makes an angle 𝛾 (in the counterclockwise direction) with the positive
direction of the 𝑥 axis. Given that 𝒂⃗⃗ lies in the second quadrant, determine the magnitude of 𝒂 ⃗⃗, and the
vector which is the 𝑦 component of 𝒂 ⃗⃗.
C. Find 𝑥 given that 𝒂⃗⃗ = 𝑥𝒊̂ + 𝟒𝒋̂ (lying in the first quadrant) and 𝒂
⃗⃗ makes an angle 𝛾 (in the clockwise
direction) with the positive direction of the 𝑦 axis.
Part A
𝑉𝑒𝑟𝑡𝑖𝑐𝑎𝑙 𝑆𝑝𝑒𝑒𝑑 = 34 sin 𝛼
𝐻𝑜𝑟𝑖𝑧𝑜𝑛𝑡𝑎𝑙 𝑆𝑝𝑒𝑒𝑑 = 34 cos 𝛼
Part B
3 3 3
cos(180 − 𝛾) = − ⇒ |𝒂⃗⃗| = − =
|𝒂
⃗⃗| cos(180 − 𝛾) cos 𝛾
|𝒚|
tan(180 − 𝛾) = ⇒ |𝒚| = 3 tan 𝛾 ⇒ 𝒚 = 3 tan 𝛾 𝒋̂
−3
P a g e 50 | 168
Get all the files at: https://bit.ly/azizhandouts
Aziz Manva (azizmanva@gmail.com)
Part C
4
𝑎 cos 𝛾 = 4 ⇒ 𝑎 = = 4 sec 𝛾
cos 𝛾
𝑥
tan 𝛾 = ⇒ 𝑥 = 4 tan 𝛾
4
𝑎 + 𝑥 = 4(sec 𝛾 + tan 𝛾)
𝑂𝑝𝑡𝑖𝑜𝑛 𝐵
Quadrant II Vector
𝑦 𝑦
< 0 ⇒ 𝜃 = tan−1 ( ) < 0 ⇒ 𝜃 ∈ 𝑄𝐼𝑉
𝑥 𝑥
⃗⃗ is in Quadrant II, then you need to add 180° to get the angle in QIII.
But if 𝒓
P a g e 51 | 168
Get all the files at: https://bit.ly/azizhandouts
Aziz Manva (azizmanva@gmail.com)
𝑦
180° + tan−1 ( )
𝑥
To generalize:
𝑦
➢ For vectors in QI and QIV use tan−1 (𝑥 )
𝑦
➢ For vectors in QII and QIII use 180° + tan−1 ( )
𝑥
Example 1.167
The vectors below are given in component form. Convert into polar form using an exact expression.
A. 𝒂⃗⃗ = (3,4)
B. 𝒃⃗⃗ = (−3, −4)
Part A
⃗⃗| = √x 2 + y 2 = √32 + 42 = 5
|𝒂
4 4
tan 𝜃 = ⇒ 𝜃 = tan−1 ( )
3 3
−1
4
⃗⃗ = (5, tan ( ))
𝒂
3
Part B
⃗⃗| has the same magnitude as |𝒂
|𝒃 ⃗⃗|, but it goes in the opposite direction.
|𝒃⃗⃗| = √𝑥 2 + 𝑦 2 = √32 + 42 = 5
4 4
tan 𝜃 = ⇒ 𝜃 = 180° + tan−1 ( )
3 3
4
⃗⃗ = (5,180° + tan−1 ( ))
𝒃
3
Example 1.168
A vector starts at the origin, and ends at (3,5). Find its
A. 𝑥 component
B. 𝑦 component
C. Magnitude
D. Direction
5
𝐷𝑖𝑟𝑒𝑐𝑡𝑖𝑜𝑛 = (tan−1 ) ° 𝑤𝑖𝑡ℎ 𝑡ℎ𝑒 ℎ𝑜𝑟𝑖𝑧𝑜𝑛𝑡𝑎𝑙
3
P a g e 52 | 168
Get all the files at: https://bit.ly/azizhandouts
Aziz Manva (azizmanva@gmail.com)
Example 1.170
𝜋 𝜋
⃗⃗ = (2, ) ,
𝒂 ⃗⃗ = (3, )
𝒃
6 4
The vectors above are given in polar form. Find each part below. State your answer in polar form. Also state the
quadrant they lie in.
A. 𝒂⃗⃗ + 𝒃⃗⃗
B. ⃗𝒂⃗ − ⃗𝒃⃗
Part A
𝜋
⃗⃗ = (2, ) = (√3
𝒂 ⏟ , 1)
6
𝜋 3√2 3√2
⃗⃗ = (3, ) = (
𝒃 , )
4 2 2
Find the vectors in component form:
P a g e 53 | 168
Get all the files at: https://bit.ly/azizhandouts
Aziz Manva (azizmanva@gmail.com)
Example 1.173
An airplane takes off from Point A, travels on a circular path with radius 𝑟 around an island, and lands at point
3
B. The length of the path travelled by the plane is 4 𝜋𝑟. Find, using vector methods only, the distance between
point A and point B in terms of 𝑟.
⃗⃗ = 𝒂
𝒂 ⃗⃗⃗⃗⃗𝒙 + ⃗⃗⃗⃗⃗
𝒂𝒚 = −𝑟𝒊̂ + 𝟎 = −𝑟𝒋̂
√2 √2
⃗𝒃⃗ = ⃗⃗⃗⃗⃗
𝒃𝒙 + 𝒃⃗⃗⃗⃗⃗𝒚 = 𝑟 cos 135° 𝒊̂ + 𝑟 sin 135° 𝒋̂ = 𝑟 (− ) 𝒊̂ + 𝑟 ( ) 𝒋̂
2 2
√2 √2 √2 √2
⃗𝒂⃗ + ⃗𝒃⃗ = −𝑟𝒊̂ + 𝑟 (− ) 𝒊̂ + 𝑟 ( ) 𝒋̂ = −𝑟 (1 + ) 𝒊̂ + 𝑟 ( ) 𝒋̂
2 2 2 2
Example 1.174
Two forces of 30𝑁 and 40𝑁 act on an object in the 𝑥-direction and the 𝑦-direction, respectively.
A. Find the angle between the two forces
B. Find the magnitude and direction of the resultant force
C. Find the angle that the resultant makes with the larger force.
Part A
Part B
𝑀𝑎𝑔𝑛𝑖𝑡𝑢𝑑𝑒 = √302 + 402 = 50
𝐵𝐶 40 4
sin 𝜃 = ⇒ 𝜃 = sin−1 = sin−1
𝐴𝐵 50 5
Part B
𝐴𝐶 30 3
sin 𝜃 = ⇒ 𝜃 = sin−1 = sin−1
𝐴𝐵 50 5
D. Finding Angles
P a g e 54 | 168
Get all the files at: https://bit.ly/azizhandouts
Aziz Manva (azizmanva@gmail.com)
Example 1.177
An airplane takes off from Point A, travels on a circular path with radius 𝑟 around an island, and lands at point
3
B. The length of the path travelled by the plane is 4 𝜋𝑟. Find, using vector methods only, the distance between
point A and point B in terms of 𝑟.
𝐷 = 𝑟√2 + √2
By Law of Cosines:
|𝑢 + 𝑣|2 = 𝑢2 + 𝑣 2 − 2 𝑢𝑣 cos 𝜃
Substituting and taking square roots:
|𝑢 + 𝑣| = √542 + 432 − 2(54)(43)(cos 30) = 27.26
E. Physics
We know that:
⃗⃗ = 𝑚𝒂
𝑭 ⃗⃗
Write each vector in component form:
P a g e 55 | 168
Get all the files at: https://bit.ly/azizhandouts
Aziz Manva (azizmanva@gmail.com)
(𝐹𝑥 , 𝐹𝑦 , 𝐹𝑧 ) = 𝑚(𝑎𝑥 , 𝑎𝑦 , 𝑎𝑧 )
Carry out the scalar multiplication:
(𝐹𝑥 , 𝐹𝑦 , 𝐹𝑧 ) = (𝑚𝑎𝑥 , 𝑚𝑎𝑦 , 𝑚𝑎𝑧 )
If two vectors are equal, then their individual components must be equal.
Equating components gives us the result that we want:
𝐹𝑥 = 𝑚𝑎𝑥 , 𝐹𝑦 = 𝑚𝑎𝑦 , 𝐹𝑧 = 𝑚𝑎𝑧
Example 1.180
A. Find the force in each direction given mass and acceleration.
P a g e 56 | 168
Get all the files at: https://bit.ly/azizhandouts
Aziz Manva (azizmanva@gmail.com)
⃗⃗ = (50,150),
𝒂 ⃗⃗ = (50√3, 60)
𝒃
And we can write the same vectors in component form as:
√3 1
⃗⃗ = (−𝑎 𝑐𝑜𝑠 30°, 𝑎 𝑠𝑖𝑛 30°) = (50 ×
𝒂 , 50 × ) = (−25√3, 25)
2 2
1 √3
⃗⃗ = (𝑏 𝑐𝑜𝑠 60°, 𝑏 𝑠𝑖𝑛 60°) = (50√3 × , 50√3 × ) = (25√3, 75)
𝒃
2 2
P a g e 57 | 168
Get all the files at: https://bit.ly/azizhandouts
Aziz Manva (azizmanva@gmail.com)
1
⃗⃗ = (𝑏 𝑐𝑜𝑠 60°, 𝑏 𝑠𝑖𝑛 60°) = ((100√3 − 100) , (100√3 − 100) √3) = (50√3 − 50,150 − 50√3)
𝒃
2 2
P a g e 58 | 168
Get all the files at: https://bit.ly/azizhandouts
Aziz Manva (azizmanva@gmail.com)
𝐷𝑖𝑠𝑡𝑎𝑛𝑐𝑒 10 𝑐𝑚 𝑐𝑚
𝑉𝑒𝑟𝑡𝑖𝑐𝑎𝑙 𝑆𝑝𝑒𝑒𝑑 𝑅𝑒𝑞𝑢𝑖𝑟𝑒𝑑 = = =5
𝑇𝑖𝑚𝑒 (𝑡 + 2) − 𝑡 𝑠𝑒𝑐𝑜𝑛𝑑
𝑖𝑛𝑐ℎ𝑒𝑠
𝐴𝑐𝑡𝑢𝑎𝑙 𝑉𝑒𝑟𝑡𝑖𝑐𝑎𝑙 𝑆𝑝𝑒𝑒𝑑 = 𝑣 cos 𝜃
𝑚𝑖𝑛𝑢𝑡𝑒
𝑖𝑛𝑐ℎ𝑒𝑠 𝑐𝑚
𝑣 cos 𝜃 =5
𝑚𝑖𝑛𝑢𝑡𝑒 𝑠𝑒𝑐𝑜𝑛𝑑
2.54 𝑐𝑚 𝑐𝑚
𝑣 cos 𝜃 =5
60 𝑠𝑒𝑐𝑜𝑛𝑑𝑠 𝑠𝑒𝑐𝑜𝑛𝑑
300 30000 15000
𝑣= = =
2.54 cos 𝜃 254 cos 𝜃 127 cos 𝜃
𝐷𝑖𝑠𝑡𝑎𝑛𝑐𝑒 4.5 𝑐𝑚 𝑐𝑚
𝐻𝑜𝑟𝑖𝑧𝑜𝑛𝑡𝑎𝑙 𝑆𝑝𝑒𝑒𝑑 𝑅𝑒𝑞𝑢𝑖𝑟𝑒𝑑 = = = 2.25
𝑇𝑖𝑚𝑒 (𝑡 + 2) − 𝑡 𝑠𝑒𝑐𝑜𝑛𝑑
𝑖𝑛𝑐ℎ𝑒𝑠
𝐴𝑐𝑡𝑢𝑎𝑙 𝐻𝑜𝑟𝑖𝑧𝑜𝑛𝑡𝑎𝑙 𝑆𝑝𝑒𝑒𝑑 = 𝑣 sin 𝜃
𝑚𝑖𝑛𝑢𝑡𝑒
𝑖𝑛𝑐ℎ𝑒𝑠 𝑐𝑚
𝑣 sin 𝜃 = 2.25
𝑚𝑖𝑛𝑢𝑡𝑒 𝑠𝑒𝑐𝑜𝑛𝑑
2.54 𝑐𝑚 𝑐𝑚
𝑣 sin 𝜃 = 2.25
60 𝑠𝑒𝑐𝑜𝑛𝑑𝑠 𝑠𝑒𝑐𝑜𝑛𝑑
135
𝑣=
2.54 sin 𝜃
To escape the slipper, the cockroach can escape either horizontally, or vertically. Hence, the final answer is:
15000 135
𝑀𝑖𝑛𝑖𝑚𝑢𝑚 ( , )
127 cos 𝜃 2.54 sin 𝜃
Example 1.185
A man on a wheelchair is being pushed up a hospital ramp by an attendant. The man weighs 70 kg, his
2
wheelchair 15 kg, and the attendant weighs 60 kg. 3 𝑟𝑑 of the way up the ramp, the attendant stops briefly. At
this point, what are the components of the system’s mass parallel and perpendicular to the ramp if the ramp
makes an angle of 20° with the horizontal.
Consider the man, his wheelchair, and the attendant to be a point mass.
P a g e 59 | 168
Get all the files at: https://bit.ly/azizhandouts
Aziz Manva (azizmanva@gmail.com)
Since 𝐹𝐻 ⊥ 𝐹𝐺
∠𝐺𝐹𝐻 = 90° ⇒ ∠𝐺𝐹𝐼 = 90 − ∠𝐼𝐹𝐻 = 90 − 70 = 20°
Step II: Focus on the Vector
𝑜𝑝𝑝 𝐵𝐶
sin 𝐴 = ⇒ sin 20° = ⇒ 𝐵𝐶 = sin 20 ° × 145 = 45.59 𝑘𝑔
ℎ𝑦𝑝 145
𝑎𝑑𝑗 𝐴𝐵
cos 𝐴 = ⇒ cos 20° = ⇒ 𝐴𝐵 = cos 20 ° × 145 = 136.26 𝑘𝑔
ℎ𝑦𝑝 145
1.6 Projections
A. Introduction
We now look at multiplication in the context of vectors, beginning with the concept of projection of vectors to
introduce the idea.
In terms of multiplication with real numbers, recall that:
3
⏟ ∙4 = 3×4
⏟ = 12
𝐷𝑜𝑡 𝑃𝑟𝑜𝑑𝑢𝑐𝑡 𝐶𝑟𝑜𝑠𝑠 𝑃𝑟𝑜𝑑𝑢𝑐𝑡
➢ |𝒓
⃗⃗| = 𝑟
➢ 𝜃 is the angle made by the vector with the positive direction of the
𝑥 − 𝑎𝑥𝑖𝑠.
𝑜𝑝𝑝
sin 𝜃 = ⇒ 𝑜𝑝𝑝 = (sin 𝜃)(ℎ𝑦𝑝) = 𝑟 sin 𝜃
ℎ𝑦𝑝
𝑎𝑑𝑗
cos 𝜃 = ⇒ 𝑎𝑑𝑗 = (cos 𝜃)(ℎ𝑦𝑝) = 𝑟 cos 𝜃
ℎ𝑦𝑝
|𝒂
⏟⃗⃗| ⏟
cos 𝜃 = ⏟
𝑥
𝑆𝑐𝑎𝑙𝑎𝑟 𝑆𝑐𝑎𝑙𝑎𝑟 𝑆𝑐𝑎𝑙𝑎𝑟
As each term on the left is a scalar, the answer on the right is also a
scalar.
➢ The length of the projection of 𝒂 ⃗⃗ is the length of the blue line in the diagram.
⃗⃗ along 𝒃
➢ Imagine a light shining somewhere to the left of vector 𝒂 ⃗⃗. The projection of vector 𝒂 ̂ is the
⃗⃗ along 𝒃
shadow cast by 𝒂 ̂
⃗⃗ upon 𝒃.
P a g e 60 | 168
Get all the files at: https://bit.ly/azizhandouts
Aziz Manva (azizmanva@gmail.com)
Example 1.188
𝟏 𝟏
⃗⃗ = (𝟎, 𝟐) along 𝒚
A. Find the length of the projection of 𝒙 ⃗⃗ = ( , )
√𝟐 √𝟐
𝟏 𝟐
⃗⃗ = (𝟑, 𝟎) along 𝒚
B. Find the length of the projection of 𝒙 ⃗⃗ = ( ,− )
√𝟓 √𝟓
Part A
Note that:
𝜃 = 45°
Hence, the projection is:
1
⃗⃗ ∙ 𝒚
𝒙 ⃗⃗| cos 𝜃 = √22 + 02 cos 45° = 2 ×
̂ = |𝒙 = √2
√2
Part B
1
𝑎𝑑𝑗 1
cos 𝜃 = = √5 =
ℎ𝑦𝑝 1 √5
Hence, the projection is:
1 3
⃗⃗ ∙ 𝒚
𝒙 ⃗⃗| cos 𝜃 = √32 + 02 ×
̂ = |𝒙 =
√5 √5
Example 1.190
5
⃗⃗ = (1.5, −4)
⃗⃗ = (0, − ) along 𝒚
Find the length of the projection of 𝒙 2
P a g e 61 | 168
Get all the files at: https://bit.ly/azizhandouts
Aziz Manva (azizmanva@gmail.com)
Example 1.192
⃗⃗⃗⃗⃗⃗⃗| = 3, |𝑪𝑨
A. In Δ𝐴𝐵𝐶, if |𝑩𝑪 ⃗⃗⃗⃗⃗⃗| = 5 and |𝑩𝑨 ⃗⃗⃗⃗⃗⃗⃗| = 7, then the projection
of the vector 𝑩𝑨⃗⃗⃗⃗⃗⃗⃗ on 𝑩𝑪
⃗⃗⃗⃗⃗⃗⃗ is equal to: (JEE Main 2021, 20 July, Shift-II)
B. In a triangle 𝐴𝐵𝐶, if |𝑩𝑪 ⃗⃗⃗⃗⃗⃗⃗| = 8, |𝑪𝑨
⃗⃗⃗⃗⃗⃗| = 7 and |𝑨𝑩 ⃗⃗⃗⃗⃗⃗⃗| = 10, then the projection of the vector 𝑨𝑩
⃗⃗⃗⃗⃗⃗⃗ on 𝑨𝑪
⃗⃗⃗⃗⃗⃗ is
equal to: (JEE Main 2021, 18 March, Shift-II)
Part A
72 + 32 − 52 49 + 9 − 25 33 11
cos ∠𝐴𝐵𝐶 = = = =
2(7)(3) 42 42 14
⃗⃗⃗⃗⃗⃗⃗ on ⃗⃗⃗⃗⃗⃗⃗
Projection of the vector 𝑩𝑨 𝑩𝑪
11 11
⃗⃗⃗⃗⃗⃗⃗| cos ∠𝐴𝐵𝐶 = 7 ∙
= |𝑩𝑨 =
14 2
Part B
⃗⃗⃗⃗⃗⃗⃗ on 𝑨𝑪
Projection of the vector 𝑨𝑩 ⃗⃗⃗⃗⃗⃗ is equal to:
102 + 72 − 82 85
⃗⃗⃗⃗⃗⃗⃗ cos 𝜃 = 10 (
𝑨𝑩 )=
2(7)(10) 14
⃗⃗ ∙ ⃗𝒃⃗
𝒂
𝒂 ̂=
⃗⃗ ∙ 𝒃 |𝒂
=⏟⃗⃗| cos 𝜃
⃗⃗|
|𝒃
C. Vector Projection
Example 1.195
Compare the formula for a vector projection with the formula for the length of the projection, and determine
the relation between the two:
⃗𝒃⃗
(|𝒂
⃗⃗| cos 𝜃) ( ) , |𝒂
⏟⃗⃗| cos 𝜃
⏟ |𝒃⃗⃗| 𝐿𝑒𝑛𝑔𝑡ℎ 𝑜𝑓 𝑃𝑟𝑜𝑗𝑒𝑐𝑡𝑖𝑜𝑛
𝑉𝑒𝑐𝑡𝑜𝑟 𝑃𝑟𝑜𝑗𝑒𝑐𝑡𝑖𝑜𝑛
P a g e 62 | 168
Get all the files at: https://bit.ly/azizhandouts
Aziz Manva (azizmanva@gmail.com)
Example 1.196
Find the projection vector for the following projections. Also, find the length of the projection.
A. 𝒂 ⃗⃗ = (3,3)
⃗⃗ = (0,2) along 𝒃
1 √3
B. 𝒂 ⃗⃗ = (−
⃗⃗ = (0, −3) along 𝒃 , )
√7 √7
Part A
Length of the projection
√2
= |𝒂
⃗⃗| cos 𝜃 = 2 cos 45° = 2 ( ) = √2
2
Example 1.198
Find the projection vector for the following projections. Also, find the length of the projection.
The ratio of the components of ⃗𝒃⃗ is the same as the ratio of the legs of a 30 − 60 − 90
triangle:
1 √3
− : = −1: √3
√7 √7
We can
√3
𝜃 = 60° + 90° = 150° ⇒ cos 150° = cos(180 − 30) = − cos 30° = −
2
√3 3√3
|𝒂
⃗⃗| cos 𝜃 = (3) (− ) = −
2 2
The length of the projection is
3√3 1 √3
𝑖𝑛 𝑡ℎ𝑒 𝑑𝑖𝑟𝑒𝑐𝑡𝑖𝑜𝑛 𝑜𝑝𝑝𝑜𝑠𝑖𝑡𝑒 𝑡𝑜 ⃗𝒃⃗ = (− , )
2 √7 √7
We can now find the unit vector in the direction of the vector that we are taking the projection along:
1 √3
|(−1, √3)| = 2 ⇒ |(− , )| = 1
2 2
The projection vector is:
⃗⃗
𝒃 3√3 1 √3 3√3 9
|𝒂⃗⃗| cos 𝜃 ( ) = − (− , ) = ( ,− )
⃗⃗|
|𝒃 2 2 2 4 4
P a g e 63 | 168
Get all the files at: https://bit.ly/azizhandouts
Aziz Manva (azizmanva@gmail.com)
Let
⃗⃗⃗⃗⃗⃗⃗ = 𝒃, 𝑨𝑫
𝑨𝑩 ⃗⃗⃗⃗⃗⃗⃗ = 𝒂
Projection of 𝒃 upon 𝒂 is:
⃗⃗| cos 𝜃
|𝒃
⃗⃗ as:
⃗⃗ and 𝒃
Then, we define the dot product of the vectors 𝒂
⃗⃗ = |𝒂
⃗⃗ ∙ 𝒃
𝒂 ⃗⃗| cos 𝜃
⃗⃗||𝒃
Notes:
➢ Scalar multiplication has a vector multiplied with a scalar. The magnitude changes, but the direction is
unchanged.
➢ The dot product (also called scalar product) of two vectors has a scalar output.
➢ Since the output is a scalar, it is also called a scalar product.
➢ The scalar product is in contrast to the cross product which has a vector for an output. We will learn the
cross product later.
Example 1.201
Consider the same diagram as above.
A. What is the domain of cos 𝜃 in this context? What quadrants does it correspond to on the unit circle?
B. What is the range of cos 𝜃 in this context?
C. In which quadrants is cos 𝜃 positive? In which quadrants is it negative?
𝜋
The domain is given by the possible values for 𝜃. Note that 𝜃 can be obtuse( < 𝜃 < 𝜋), but not reflex.
2
0≤𝜃≤𝜋
𝑄𝑢𝑎𝑑𝑟𝑎𝑛𝑡 𝐼 𝑎𝑛𝑑 𝐼𝐼
−1 ≤ cos 𝜃 ≤ 1
P a g e 64 | 168
Get all the files at: https://bit.ly/azizhandouts
Aziz Manva (azizmanva@gmail.com)
Example 1.202
A. Determine the sign of |𝒂 ⃗⃗|
⃗⃗|, the sign of |𝒃
B. Hence, for non-zero vectors 𝒂 ⃗⃗, what does the sign of 𝒂
⃗⃗ and 𝒃 ⃗⃗ depend on?
⃗⃗ ∙ 𝒃
C. Determine the sign of cos 𝜃
Part A
The magnitude of a vector is the length of the vector, which is always a nonnegative quantity:
|𝒂
⃗⃗| ≥ 0
⃗⃗| ≥ 0
|𝒃
Part B
⃗⃗ ∙ ⃗𝒃⃗ = |𝒂
𝒂 ⏟ ⏟⃗⃗| cos 𝜃
⃗⃗| |𝒃
+𝑣𝑒 +𝑣𝑒
⃗⃗ ∙ ⃗𝒃⃗ depends on the sign of cos 𝜃
Since the first two terms are positive, the value of 𝒂
Part C
Example 1.203
⃗⃗, in terms of the magnitude of the vectors, if the angle
⃗⃗ and 𝒃
Find the dot product in each case below between 𝒂
between the two vectors is:
A. zero
𝜋
B.
2
C. 𝜋
|𝒂 ⃗⃗| × 1 = |𝒂
⃗⃗||𝒃 ⃗⃗|
⃗⃗||𝒃
|𝒂 ⃗⃗| × 0 = 0
⃗⃗||𝒃
|𝒂 ⃗⃗| × (−1) = −|𝒂
⃗⃗||𝒃 ⃗⃗|
⃗⃗||𝒃
P a g e 65 | 168
Get all the files at: https://bit.ly/azizhandouts
Aziz Manva (azizmanva@gmail.com)
B. Work
Example 1.206
The moon orbits the Earth. For this question, assume that the orbit is circular. Explain why the work done by
the Earth on the moon is zero:
A. Mathematically
B. Using the concept of KE
Example 1.207
Decide whether work done is positive, negative, or zero:
A. The net force on an object increases its velocity
B. The net force on an object decreases its velocity
C. The net force on an object does not change its velocity.
P a g e 66 | 168
Get all the files at: https://bit.ly/azizhandouts
Aziz Manva (azizmanva@gmail.com)
Example 1.209
Decide whether work done is positive, negative, or zero:
A. The force on an object increases its velocity
B. The force on an object decreases its velocity
C. The force on an object does not change its velocity.
1.210: Work
Work done also equals change in energy, not just kinetic energy.
Suppose you move an object ℎ feet higher then where it was. The increase in potential energy of the object
comes from:
➢ The height h the object was moved up. The greater the height, the more the energy.
➢ The mass of the object. The greater the mass, the more the energy
➢ The resistance provided by gravity. Earth has higher gravity, compared to the moon, for instance.
Example 1.212
Work done by gravity in left/right movement
Example 1.213
Resolution into components for dragging a trolley, and then ignoring the vertical component
1.214: Work in 3D
⃗⃗ ∙ 𝒔
𝑊=𝑭 ⃗⃗ = 𝐹𝑠 cos 𝜃
➢ We have already seen that a force perpendicular to an object does zero work.
➢ Hence, for a two-dimensional force, we only want the component of the force along the direction of
movement.
Example 1.215
̂ and 3𝒊̂ + 𝒋̂ − 𝒌
A particle is acted upon by constant forces 4𝒊̂ + 𝒋̂ − 3𝒌 ̂ which displace it from a point 𝒊̂ + 2𝒋̂ + 3𝒌
̂
̂
to the point 5𝒊̂ + 4𝒋̂ + 𝒌. The work done in standard units by the forces is given by: (JEE Main 2004)
P a g e 67 | 168
Get all the files at: https://bit.ly/azizhandouts
Aziz Manva (azizmanva@gmail.com)
Calculate:
𝐹𝑜𝑟𝑐𝑒 = ⃗𝑭⃗ = (4,1, −3) + (3,1, −1) = (7,2, −4)
𝐷𝑖𝑠𝑝𝑙𝑎𝑐𝑒𝑚𝑒𝑛𝑡 = 𝒔 ⃗⃗ = (5,4,1)
⏟ − (1,2,3)
⏟ = (4,2, −2)
𝑬𝒏𝒅 𝑰𝒏𝒊𝒕𝒊𝒂𝒍
𝑷𝒐𝒊𝒏𝒕 𝑷𝒐𝒊𝒏𝒕
Example 1.217
Find the dot product for each part below:
A. 𝒂 ⃗⃗ has a length of 5 units. The angle between them is 120°.
⃗⃗ has a length of 3 units, 𝒃
B. 𝒂 ⃗⃗ has a length of 2 units. The angle between them is 45°.
⃗⃗ has a length of 7 units, 𝒃
C. 𝒂 = √3,|𝒃| = 2 and angle between a and b is 60. (CBSE 2011)
1 15
⃗⃗ ∙ ⃗𝒃⃗ = |𝒂
𝒂 ⃗⃗| cos 𝜃 = (3)(5) (− ) = −
⃗⃗||𝒃
2 2
⃗⃗ ∙ ⃗𝒃⃗ = |𝒂
𝒂 ⃗⃗| cos 𝜃 = (7)(2) (√2) = 7√2
⃗⃗||𝒃
2
1
⃗⃗ ∙ 𝒃
𝒂 ⃗⃗ = |𝒂 ⃗⃗| cos 𝜃 = (√3)(2) ( ) = √3
⃗⃗||𝒃
2
Example 1.218
The dot product of two vectors is 12. The magnitude of one of the vectors is 6. The magnitude of the other
vector is 3. Then, the angle between the two vectors, in degrees, is between:
A. 0 𝑡𝑜 30
B. 30 𝑡𝑜 60
C. 60 𝑡𝑜 90
D. 90 𝑡𝑜 180
E. 270 𝑡𝑜 360
F. More than one of the above
G. Cannot be determined
P a g e 68 | 168
Get all the files at: https://bit.ly/azizhandouts
Aziz Manva (azizmanva@gmail.com)
|𝒂 ⃗⃗| cos 𝜃 = 12
⃗⃗||𝒃
(6)(3) cos 𝜃 = 12
2
cos 𝜃 = = 0.66 ⇒ 𝐼𝑛 𝑄𝑢𝑎𝑑𝑟𝑎𝑛𝑡 𝐼
3
1 √3 1.71
cos 60 = = 0.5, cos 30 = ≈ = 0.855 ⇒ 𝑂𝑝𝑡𝑖𝑜𝑛 𝐵
2 2 2
Example 1.219
Find the dot product for each part below:
A. The magnitude of 𝒂 ⃗⃗ is 10, the magnitude of ⃗𝒃⃗ is 7, and the larger angle between them is 225°.
B. 𝒂⃗⃗ = (3,0), ⃗𝒃⃗ = (2,2)
C. 𝒂⃗⃗ = (4,4), ⃗𝒃⃗ = ( )
Part A
𝜃 = 360 − 225 = 135°
√2
cos 135° = cos(180 − 45°) = − cos 45° = −
2
⃗⃗ = |𝒂 ⃗⃗| cos 𝜃 = (10)(7) (cos − √2
⃗⃗ ∙ 𝒃
𝒂 ⃗⃗||𝒃 ) = −35√2
2
Part B
√2
𝜃 = 45° ⇒ cos 𝜃 =
2
√2
𝒂 ⃗⃗ = |𝒂
⃗⃗ ∙ 𝒃 ⃗⃗| cos 𝜃 = 3(2√2)
⃗⃗||𝒃 =6
2
Example 1.220
⃗⃗ and ⃗𝒃⃗ are vectors each with a magnitude of 2√2, and ⃗𝒃⃗ − 𝒂
A. 𝒂 ⃗⃗ has a magnitude of 4. Find the dot product
of the two vectors.
B. Find the dot product of vectors 𝒂 ⃗⃗ and ⃗𝒃⃗ given that 𝒂
⃗⃗ is a unit vector, ⃗𝒃⃗ has a magnitude of 2 units, and
⃗𝒃⃗ − 𝒂
⃗⃗ has a magnitude of√3.
Part A
Draw a diagram. Note that the difference of the vectors gives the third side of a
triangle (if they form one).
2
(2√2) = 4 × 2 = 8
2√2 × √2 = 4
Since the magnitude of the vectors 𝒂⃗⃗ and ⃗𝒃⃗ when multiplied by √2 gives the
magnitude of the third side of the triangle formed by them, the triangle is a 45 −
45 − 90 triangle.
⃗⃗ is 90°:
⃗⃗ and 𝒃
Hence, the angle between 𝒂
𝐷𝑜𝑡 𝑝𝑟𝑜𝑑𝑢𝑐𝑡 = 0
P a g e 69 | 168
Get all the files at: https://bit.ly/azizhandouts
Aziz Manva (azizmanva@gmail.com)
Part B
We know that the vectors 𝒂 ⃗⃗, ⃗𝒃⃗, ⃗𝒃⃗ − 𝒂
⃗⃗ form a triangle. The ratios of the
magnitudes are:
1
⃗⃗ − 𝒂
|𝒂|: | 𝒃 ⃗⃗| = 1: √3: 2 = : √3 : 1
⃗⃗|: |𝒃
2 2
But, these are the ratios of a 30 − 60 − 90 𝑡𝑟𝑖𝑎𝑛𝑔𝑙𝑒 (see diagram). Hence:
Therefore, the angle between the two vectors is 60°.
Dot product:
1
= |𝒂 ⃗⃗| cos 𝜃 = (1)(2) cos 60° = 2 ×
⃗⃗||𝒃 =1
2
Example 1.221
⃗⃗, having the same magnitude such that the angle between
⃗⃗ and 𝒃
Find the magnitude of each of the two vectors 𝒂
9
them is 60, and their scalar product is 2. (CBSE 2018)
⃗⃗ ∙ ⃗𝒃⃗ = |𝒂
𝒂 ⃗⃗| cos 𝜃
⃗⃗||𝒃
9
|𝒂
⃗⃗||𝒂
⃗⃗| cos 60° =
2
1 2 9
|𝒂⃗⃗| =
2 2
⃗⃗|2 = 9
|𝒂
|𝒂
⃗⃗| = 3
Example 1.222
⃗⃗ and 𝒃
Given two vectors 𝒂 ⃗⃗, determine
A. the maximum and the minimum value of their dot product as 𝜃 varies. Answer in terms of 𝒂 ⃗⃗.
⃗⃗, 𝒃
B. the angle 𝜃, if the dot product of the two vectors has the maximum possible value? the minimum
possible value.
Part A
The dot product is given by:
⃗⃗ ∙ ⃗𝒃⃗ = |𝒂
𝒂 ⃗⃗| cos 𝜃
⃗⃗||𝒃
We know that:
−1 ≤ cos 𝜃 ≤ 1
P a g e 70 | 168
Get all the files at: https://bit.ly/azizhandouts
Aziz Manva (azizmanva@gmail.com)
Example 1.223
Determine whether the following is possible. If not possible, explain why.
⃗⃗ and ⃗𝒃⃗ are position vectors such that 𝒂
A. 𝒂 ⃗⃗ ∙ ⃗𝒃⃗ is nonnegative, the tip of 𝒂
⃗⃗ lies in Quadrant II, and the tip of
⃗𝒃⃗ lies in Quadrant IV.
Example 1.224
The dot product of two vectors is 12. The magnitude of one of the vectors is √2. Find the magnitude of the other
vector if the angle between the vectors is 135°.
Method I
The angle between the two vectors is obtuse
𝜃 = 135° ⇒ cos 𝜃 < 0 ⇒ 𝐷𝑜𝑡 𝑃𝑟𝑜𝑑𝑢𝑐𝑡 < 0 ⇒ 𝐶𝑜𝑛𝑡𝑟𝑎𝑑𝑖𝑐𝑡𝑖𝑜𝑛 ⇒ 𝑁𝑜 𝑠𝑢𝑐ℎ 𝑣𝑒𝑐𝑡𝑜𝑟
Method II
|𝒂 ⃗⃗| cos 𝜃 = 12
⃗⃗||𝒃
⃗⃗| (− √2) = 12
√2|𝒃
2
⃗⃗| = −12
|𝒃
Since magnitude is never negative, there is no such vector.
D. Projection
⃗⃗ ∙ ⃗𝒃⃗ |𝒂
𝒂 ⃗⃗| cos 𝜃
⃗⃗||𝒃
𝐿𝐻𝑆 = = = |𝒂
⃗⃗| cos 𝜃 = 𝑅𝐻𝑆
⃗⃗|
|𝒃 ⃗⃗|
|𝒃
E. Perpendicularity
Example 1.227
⃗⃗ ∙ 𝒂
If 𝒂 ⃗⃗ = 0 and 𝒂 ⃗⃗ = 0, then what can be concluded about the vector 𝒃
⃗⃗ ∙ 𝒃 ⃗⃗? (CBSE 2011)
⃗⃗ ∙ 𝒂
𝒂 ⃗⃗ = 0
|𝒂
⃗⃗||𝒂⃗⃗| cos 0 = 0
|𝒂
⃗⃗||𝒂⃗⃗| = 0
P a g e 71 | 168
Get all the files at: https://bit.ly/azizhandouts
Aziz Manva (azizmanva@gmail.com)
|𝒂
⃗⃗| = 0
⃗⃗| cos 𝜃) = 0
(0)(|𝒃
Nothing can be concluded about ⃗𝒃⃗.
𝜋
𝒊̂ ∙ 𝒋̂ = |𝒊̂||𝒋̂| cos 𝜃 = |𝒊̂||𝒋̂| cos ( ) = |𝒊̂||𝒋̂|(0) = 0
2
Example 1.230
⃗⃗ = 5𝒊̂ and ⃗𝒃⃗ = 7𝒋̂ and 𝒄
Given that 𝒂 ̂ , find 𝒂
⃗⃗ = 𝟒𝒌 ⃗⃗ ∙ ⃗𝒃⃗ + ⃗𝒃⃗ ∙ 𝒄
⃗⃗ + 𝒄
⃗⃗ ∙ 𝒂
⃗⃗
⃗⃗ ∙ ⃗𝒃⃗ + ⃗𝒃⃗ ∙ 𝒄
Note that there are three terms in the expression 𝒂 ⃗⃗ + 𝒄
⃗⃗ ∙ 𝒂
⃗⃗ and each term requires find the dot
product of two standard basis vectors which are not in the same direction.
For example:
⃗⃗ ∙ ⃗𝒃⃗ = 5𝒊̂ ∙ 7𝒋̂ = 35𝒊̂ ∙ 𝒋̂ = (35)(0) = 0
𝒂
Similarly, the other terms are also zero, and the final answer is:
0+0+0=0
Example 1.232
P a g e 72 | 168
Get all the files at: https://bit.ly/azizhandouts
Aziz Manva (azizmanva@gmail.com)
Two vectors have magnitudes 4 and 5 respectively. Their dot product is zero. Find the magnitude of the
A. Difference of the two vectors.
B. Sum of the two vectors
|𝒂 − 𝒃| = √42 + 52 = √41
|𝒂 + 𝒃| = √42 + 52 = √41
⃗⃗ ∙ ⃗𝒃⃗ = |𝒂
𝒂 ⃗⃗| cos 𝜃
⃗⃗||𝒃
Divide both sides by |𝒂 ⃗⃗|:
⃗⃗||𝒃
⃗𝒂⃗ ∙ ⃗𝒃⃗
cos 𝜃 =
|𝒂⃗⃗||𝒃⃗⃗|
Take the cos inverse of both sides:
⃗⃗ ∙ ⃗𝒃⃗
𝒂
𝜃 = cos−1 ( )
|𝒂
⃗⃗||𝒃⃗⃗|
Example 1.234
9
The dot product of two vectors, both with magnitude 3, is . Find the length of the difference of the two vectors.
2
9 9 1
|𝒂 ⃗⃗| cos 𝜃 =
⃗⃗||𝒃 ⇒ (3)(3) cos 𝜃 = ⇒ cos 𝜃 = ⇒ 𝜃 = 60°
2 2 2
⃗⃗ − ⃗𝒃⃗| is the third side of the triangle, and since the triangle is equilateral,
|𝒂
the
𝐿𝑒𝑛𝑔𝑡ℎ 𝑜𝑓 𝑡ℎ𝑖𝑟𝑑 𝑠𝑖𝑑𝑒 = 3
P a g e 73 | 168
Get all the files at: https://bit.ly/azizhandouts
Aziz Manva (azizmanva@gmail.com)
Example 1.236
Let 𝑉 be the set of standard basis vectors.
A. Write V
B. 𝑊 is the Cartesian product of the set 𝑉 with itself. Write W. Find the cardinality of 𝑊.
C. Find the sum of the dot product of all elements of 𝑊.
D. 𝑋 is the set of elements formed by evaluating the dot product of each pair in 𝑊. Write 𝑋.
Hint: The Cartesian product of two sets is the set of all ordered pairs such that the first element belongs to the
first set, and the second element belongs to the second set.
Part A
̂}
𝑉 = {𝒊̂, 𝒋̂, 𝒌
Part B
̂∙𝒌
𝒊̂ ∙ 𝒊̂ = 𝒋̂ ∙ 𝒋̂ = 𝒌 ̂=1
All other elements involve vectors which are perpendicular to each other, and hence their dot product is zero.
Part C
Sum of dot product
= 1+1+1+0+0+0+0+0+0 = 3
Part D
𝑋 = {0,1}
𝒂 ⃗⃗ = |𝒂
⃗⃗ ∙ 𝒂 ⃗⃗||𝒂
⃗⃗| cos 𝜃
But the angle that a vector makes with itself is 𝜃 = 0:
= |𝒂 ⃗⃗| cos 0 = |𝒂
⃗⃗||𝒂 ⃗⃗|2
⃗⃗| = |𝒂
⃗⃗||𝒂
Example 1.238
⃗⃗ = (3, −4), find 𝒙
A. Given that 𝒙 ⃗⃗ ∙ 𝒙
⃗⃗
⃗⃗ ∙ 𝒙
𝒙 ⃗⃗|2 = 32 + (−42 ) = 9 + 16 = 25
⃗⃗ = |𝒙
⃗⃗ = |𝒂
⃗⃗ ∙ 𝒃
𝒂 ⃗⃗| cos 𝜃 = |𝒃
⃗⃗||𝒃 ⃗⃗||𝒂 ⃗⃗ ∙ 𝒂
⃗⃗| cos 𝜃 = 𝒃 ⃗⃗
P a g e 74 | 168
Get all the files at: https://bit.ly/azizhandouts
Aziz Manva (azizmanva@gmail.com)
⃗⃗ ∙ 𝒄
Let 𝒃 ⃗⃗ = 𝑥:
𝒂 ⃗⃗ ∙ 𝒄
⃗⃗ ∙ 𝒃 ⃗⃗ = 𝒂
⃗⃗ ∙ 𝑥 = ⃗⃗
⏟
𝒂 ∙ ⏟
𝑥 ⇒ 𝑁𝑜𝑡 𝐷𝑒𝑓𝑖𝑛𝑒𝑑
𝑉𝑒𝑐𝑡𝑜𝑟 𝑆𝑐𝑎𝑙𝑎𝑟
Example 1.241
You will later learn about the “triple product” of vectors, where three vectors are multiplied. Explain why
̂ is not a valid triple product.
𝒊̂ ∙ 𝒋̂ ∙ 𝒌
⃗⃗ + ⃗𝒃⃗) = 𝑃𝑟𝑜𝑗(𝒂
Use the property that 𝑃𝑟𝑜𝑗(𝒂 ⃗⃗):
⃗⃗) + 𝑃𝑟𝑜𝑗(𝒃
|𝒄
⃗⃗| [𝑃𝑟𝑜𝑗|𝒂⃗⃗+𝒃⃗⃗| 𝑢𝑝𝑜𝑛 ⃗𝒄⃗ ]
Convert the second term using the definition of projection:
|𝒄
⃗⃗|[|𝒂⃗⃗ + ⃗𝒃⃗| cos 𝜃3 ]
But note the above is exactly a dot product, using the definition:
=𝒄 ⃗⃗ + ⃗𝒃⃗) = 𝐿𝐻𝑆
⃗⃗ ∙ (𝒂
Note
➢ We proved this for two dimensional vectors. However, it is true for vectors of any dimension.
Example 1.243
Two vectors are such that the magnitude of their sum is also the magnitude of their difference. What condition
must these vectors meet? Justify.
P a g e 75 | 168
Get all the files at: https://bit.ly/azizhandouts
Aziz Manva (azizmanva@gmail.com)
Example 1.245
⃗⃗ and 𝒂
Let 𝒂 ⃗⃗ be two unit vectors. If the vectors 𝒄 ⃗⃗ = 𝒂 ⃗⃗ and ⃗𝒅⃗ = 5𝒂
⃗⃗ + 2𝒃 ⃗⃗ are perpendicular to each other,
⃗⃗ − 4𝒃
then the angle between 𝒂 ⃗⃗ and ⃗𝒃⃗ (in radians) is: (JEE Main 2014)
⃗⃗) ∙ (5𝒂
⃗⃗ + 2𝒃
(𝒂 ⃗⃗) = 0
⃗⃗ − 4𝒃
2
⃗⃗|2 + 10𝒂
5|𝒂 ⃗⃗ − 𝟒𝒂
⃗⃗ ∙ 𝒃 ⃗⃗ − 𝟖|𝒃
⃗⃗ ∙ 𝒃 ⃗⃗| = 0
2
⃗⃗|2 + 6𝒂
5|𝒂 ⃗⃗ − 𝟖|𝒃
⃗⃗ ∙ 𝒃 ⃗⃗| = 0
2
⃗⃗| = 1:
⃗⃗|2 = |𝒃
Substitute |𝒂
⃗⃗ − 𝟖 = 0
⃗⃗ ∙ 𝒃
5 + 6𝒂
⃗⃗ = 3
⃗⃗ ∙ 𝒃
6𝒂
Example 1.246
If (𝒂 ⃗⃗) is perpendicular to (7𝒂
⃗⃗ + 3𝒃 ⃗⃗) and (𝒂
⃗⃗ − 5𝒃 ⃗⃗) is perpendicular to (7𝒂
⃗⃗ − 4𝒃 ⃗⃗), then the angle between 𝒂
⃗⃗ − 2𝒃 ⃗⃗
⃗⃗
and 𝒃 (in degrees) is: (JEE Main 2021, 25 July, Shift-II)
P a g e 76 | 168
Get all the files at: https://bit.ly/azizhandouts
Aziz Manva (azizmanva@gmail.com)
⃗⃗) ∙ (7𝒂
⃗⃗ − 4𝒃
(𝒂 ⃗⃗) = 0
⃗⃗ − 2𝒃
2
⃗⃗|2 − 2𝒂
7|𝒂 ⃗⃗ − 28𝒂
⃗⃗ ∙ 𝒃 ⃗⃗ + 8|𝒃
⃗⃗ ∙ 𝒃 ⃗⃗| = 0
2
2
⏟ ⃗⃗| − 30𝒂
7|𝒂 ⃗⃗ + 8|𝒃
⃗⃗ ∙ 𝒃 ⃗⃗| = 0
𝑬𝒒𝒖𝒂𝒕𝒊𝒐𝒏 𝑰𝑰
There are more than two variables here. So, first eliminate the dot product 𝒂 ⃗⃗
⃗⃗ ∙ 𝒃
Multiply Equation I by 15, and Equation II by 8:
2
105|𝒂 ⃗⃗|2 + 240𝒂 ⃗⃗ − 225|𝒃
⃗⃗ ∙ 𝒃 ⃗⃗| = 0
2
⃗⃗|2 − 240𝒂
56|𝒂 ⃗⃗ + 64|𝒃
⃗⃗ ∙ 𝒃 ⃗⃗| = 0
B. Square of a Sum
Dot product of a vector with itself is the square of its magnitude. That is 𝒙 ⃗⃗ ∙ 𝒙 ⃗⃗|2 . Apply the property to the
⃗⃗ = |𝒙
LHS of the above:
2
⃗⃗ + 𝒃
𝑳𝑯𝑺 = |𝒂 ⃗⃗| = (𝒂 ⃗⃗) ∙ (𝒂
⃗⃗ + 𝒃 ⃗⃗)
⃗⃗ + 𝒃
Using the distributive property of vectors:
⃗⃗ ∙ 𝒂
𝒂 ⃗⃗ + 𝒂 ⃗⃗ + 𝒃
⃗⃗ ∙ 𝒃 ⃗⃗ ∙ 𝒂 ⃗⃗ ∙ 𝒃
⃗⃗ + 𝒃 ⃗⃗
⃗⃗ ∙ 𝒂
Substitute 𝒂 ⃗⃗ = ⃗⃗|2
|𝒂 :
2
⃗⃗|2 + 𝒂
|𝒂 ⃗⃗ + 𝒃
⃗⃗ ∙ 𝒃 ⃗⃗ ∙ 𝒂
⃗⃗ + |𝒃⃗⃗|
Using the commutative property of dot product:
2
⃗⃗|2 + 2𝒂
|𝒂 ⃗⃗ ∙ ⃗𝒃⃗ + |𝒃⃗⃗|
Example 1.248
⃗⃗ and ⃗𝒃⃗ are perpendicular vectors, |𝒂
If 𝒂 ⃗⃗ + ⃗𝒃⃗| = 13 and |𝒂 ⃗⃗|. (CBSE 2014, ISC 2019)
⃗⃗| = 5, then find the value of |𝒃
⃗⃗ + ⃗𝒃⃗| = 13:
Square both sides of |𝒂
2
⃗⃗ + ⃗𝒃⃗| = 169
|𝒂
P a g e 77 | 168
Get all the files at: https://bit.ly/azizhandouts
Aziz Manva (azizmanva@gmail.com)
2 2
⃗⃗ + ⃗𝒃⃗| = |𝒂
Use the property that |𝒂 ⃗⃗ ∙ ⃗𝒃⃗ + |𝒃
⃗⃗|2 + 2𝒂 ⃗⃗| :
2
⃗⃗ ∙ ⃗𝒃⃗ + |𝒃
⃗⃗|2 + 2𝒂
|𝒂 ⃗⃗| = 169
⃗⃗|2 = 52 = 25, 𝒂
Substitute |𝒂 ⃗⃗ = 0:
⃗⃗ ∙ 𝒃
2
⃗⃗| = 169
25 + |𝒃
2
⃗⃗| = 144
|𝒃
⃗⃗| = 12
|𝒃
Example 1.249
⃗⃗ are two unit vectors such that |𝒂
⃗⃗ and 𝒃
If 𝒂 ⃗⃗| is also a unit vector, then find the angle between 𝒂
⃗⃗ + 𝒃 ⃗⃗. (CBSE
⃗⃗ and 𝒃
2014)
Geometric Method
We get an equilateral triangle, as shown alongside.
To find the angle between the vectors, we arrange the vectors tail to tail,
getting an angle of
180 − 60 = 120°
Algebraic Method
⃗⃗ + ⃗𝒃⃗| = 1:
Square both sides of |𝒂
2
⃗⃗ ∙ ⃗𝒃⃗ + |𝒃
⃗⃗|2 + 2𝒂
|𝒂 ⃗⃗| = 1
1 + 2𝒂 ⃗⃗ ∙ ⃗𝒃⃗ + 1 = 1
1
⃗⃗ = −
⃗⃗ ∙ 𝒃
𝒂
2
1
|𝒂 ⃗⃗| cos 𝜃 = −
⃗⃗||𝒃
2
1
cos 𝜃 = −
2
𝜃 = 120°
Example 1.250
⃗⃗ are two vectors such that |𝒂
⃗⃗ and 𝒃
If 𝒂 ⃗⃗| = |𝒂
⃗⃗ + 𝒃 ⃗⃗|, then prove that vector 2𝒂 ⃗⃗ is perpendicular to vector 𝒃
⃗⃗ + 𝒃 ⃗⃗.
(CBSE 2013)
⃗⃗ + ⃗𝒃⃗| = |𝒂
Square both sides of |𝒂 ⃗⃗|:
2
⃗⃗ ∙ ⃗𝒃⃗ + |𝒃
⃗⃗|2 + 2𝒂
|𝒂 ⃗⃗| = |𝒂
⃗⃗|2
2
⃗⃗| = ⃗𝒃⃗ ∙ ⃗𝒃⃗
⃗⃗|2 from both sides and substitute |𝒃
Subtract |𝒂
2𝒂⃗⃗ ∙ ⃗𝒃⃗ + ⃗𝒃⃗ ∙ ⃗𝒃⃗ = 0
⃗𝒃⃗ ∙ (2𝒂 ⃗⃗ + ⃗𝒃⃗) = 0
Since the dot product is zero, the two vectors are perpendicular.
Example 1.251
⃗⃗ + 𝒄
⃗⃗ + 𝒃
If 𝒂 ⃗⃗ = 0, and |𝒂 ⃗⃗| = 6, and |𝒄
⃗⃗| = 5, |𝒃 ⃗⃗| = 9, then find the angle between 𝒂 ⃗⃗. (CBSE 2018)
⃗⃗ and 𝒃
𝒂 ⃗⃗ = −𝒄
⃗⃗ + 𝒃 ⃗⃗
P a g e 78 | 168
Get all the files at: https://bit.ly/azizhandouts
Aziz Manva (azizmanva@gmail.com)
Example 1.252
If ⃗𝒙⃗ and ⃗𝒚⃗ be two non-zero vectors such that |𝒙
⃗⃗ + ⃗𝒚⃗| = |𝒙
⃗⃗| and 2𝒙
⃗⃗ + 𝜆𝒚
⃗⃗ is perpendicular to ⃗𝒚⃗, then the value of 𝜆
is: (JEE Main 2020, 6 Sep, Shift-II)
1=𝜆
Example 1.253
1
⃗⃗ and ⃗𝒃⃗ be two vectors such that |2𝒂
Let 𝒂 ⃗⃗| = |3𝒂
⃗⃗ + 3𝒃 ⃗⃗ + ⃗𝒃⃗| and the angle between 𝒂
⃗⃗ and ⃗𝒃⃗ is 60°. If 𝒂⃗⃗ is a unit
8
⃗⃗| is equal to: (JEE Main 2021, 31 Aug, Shift-I)
vector, then |𝒃
⃗⃗| = |3𝒂
⃗⃗ + 3𝒃
|2𝒂 ⃗⃗|
⃗⃗ + 𝒃
Square both sides:
2 2
⃗⃗ ∙ ⃗𝒃⃗ + 9|𝒃
⃗⃗|2 + 12𝒂
4|𝒂 ⃗⃗| = 9|𝒂 ⃗⃗ ∙ ⃗𝒃⃗ + |𝒃
⃗⃗|2 + 6𝒂 ⃗⃗|
P a g e 79 | 168
Get all the files at: https://bit.ly/azizhandouts
Aziz Manva (azizmanva@gmail.com)
1
⃗⃗ ∙ ⃗𝒃⃗ = 6|𝒂
6𝒂 ⃗⃗| cos 𝜃 = 6 ∙ 8|𝒃
⃗⃗||𝒃 ⃗⃗| cos 60° = 6 ∙ 8|𝒃
⃗⃗| ( ) = 24|𝒃
⃗⃗|
2
C. Square of a Difference
Dot product of a vector with itself is the square of its magnitude. That is 𝒙 ⃗⃗ ∙ 𝒙 ⃗⃗|2 . Apply the property to the
⃗⃗ = |𝒙
LHS of the above:
2
⃗⃗ − 𝒃
𝑳𝑯𝑺 = |𝒂 ⃗⃗| = (𝒂 ⃗⃗) ∙ (𝒂
⃗⃗ − 𝒃 ⃗⃗)
⃗⃗ − 𝒃
Using the distributive property of vectors:
⃗⃗ ∙ 𝒂
𝒂 ⃗⃗ ∙ ⃗𝒃⃗ − ⃗𝒃⃗ ∙ 𝒂
⃗⃗ − 𝒂 ⃗⃗ + ⃗𝒃⃗ ∙ ⃗𝒃⃗
Cancel common terms on both sides:
2
⃗⃗|2 − 𝒂
|𝒂 ⃗⃗ − 𝒃
⃗⃗ ∙ 𝒃 ⃗⃗ ∙ 𝒂
⃗⃗ + |𝒃⃗⃗|
Using the commutative property of dot product:
2
⃗⃗|2 − 2𝒂
|𝒂 ⃗⃗ ∙ ⃗𝒃⃗ + |𝒃⃗⃗|
Example 1.255
⃗⃗ and ⃗𝒃⃗ are unit vectors, then find the angle between 𝒂
If 𝒂 ⃗⃗ and ⃗𝒃⃗ given that (√3𝒂
⃗⃗ − ⃗𝒃⃗) is a unit vector.
⃗⃗| = 1
⃗⃗ − 𝒃
|√3𝒂
Square both sides:
⃗⃗| = 1
⃗⃗ − 𝒃
|√3𝒂
2 2
⃗⃗| = |𝒂
⃗⃗ − 𝒃
Use the property that |𝒂 ⃗⃗|2 − 2𝒂 ⃗⃗ + |𝒃
⃗⃗ ∙ 𝒃 ⃗⃗| :
2 2
⃗⃗)|2 − (2)(√3𝒂
(√3) |(𝒂 ⃗⃗) + |𝒃
⃗⃗) ∙ (𝒃 ⃗⃗| = 1
Substitute |𝒂 ⃗⃗| = 1:
⃗⃗| = |𝒃
⃗⃗ ∙ ⃗𝒃⃗) + 1 = 1
3 − (2√3)(𝒂
Isolate the dot product:
⃗⃗ ∙ ⃗𝒃⃗) = 3
(2√3)(𝒂
P a g e 80 | 168
Get all the files at: https://bit.ly/azizhandouts
Aziz Manva (azizmanva@gmail.com)
⃗⃗ = √3
⃗⃗ ∙ 𝒃
𝒂
2
⃗⃗| cos 𝜃 = √3
|𝒂
⃗⃗||𝒃
2
√3
cos 𝜃 =
2
√3 𝜋
𝜃 = cos−1 =
2 6
Example 1.256
⃗⃗ and ⃗𝒃⃗ are unit vectors, then find the maximum and minimum value of 𝒂
If 𝒂 ⃗⃗ ∙ ⃗𝒃⃗:
A. Using the angle definition of the dot product
2 2
B. Using the property |𝒂 ⃗⃗ ± ⃗𝒃⃗| = |𝒂 ⃗⃗ ∙ ⃗𝒃⃗ + |𝒃
⃗⃗|2 ± 2𝒂 ⃗⃗|
Part A ⃗⃗ ∙ ⃗𝒃⃗ + 1 ≥ 0
1 ± 2𝒂
𝒂 ⃗⃗ = |𝒂||𝒃| cos 𝜃 = (1)(1)(cos 𝜃) = cos 𝜃
⃗⃗ ∙ 𝒃 ±2𝒂⃗⃗ ∙ ⃗𝒃⃗ ≥ −2
𝑀𝑎𝑥 = 1, 𝑀𝑖𝑛 = −1 Case I:
Part B ⃗⃗ ≥ −2
⃗⃗ ∙ 𝒃
2𝒂
Since the magnitude of a vector is never negative, ⃗⃗ ≥ −1
⃗⃗ ∙ 𝒃
𝒂
we must have:
Case II:
⃗⃗ ± ⃗𝒃⃗| ≥ 0
|𝒂
−2𝒂 ⃗⃗ ∙ ⃗𝒃⃗ ≥ −2
Square both sides:
2
Divide by −2 both sides and flip the sign of the
⃗⃗ ± ⃗𝒃⃗| ≥ 0
|𝒂 inequality
2 2
⃗⃗ ± ⃗𝒃⃗| = |𝒂
Use the property |𝒂 ⃗⃗ ∙ ⃗𝒃⃗ + |𝒃
⃗⃗|2 ± 2𝒂 ⃗⃗| : ⃗⃗ ∙ ⃗𝒃⃗ ≤ 1
𝒂
2 We get the same answer from Part A and from Part
⃗⃗|2 ± 2𝒂
|𝒂 ⃗⃗ + |𝒃
⃗⃗ ∙ 𝒃 ⃗⃗| ≥ 0
B:
2
Substitute |𝒂 ⃗⃗| = 1:
⃗⃗|𝟐 = |𝒃 ⃗⃗ ∙ ⃗𝒃⃗ ∈ [−1,1]
𝒂
D. Square of a Difference
Example 1.258
Find |𝒙 ̂ , (𝒙
⃗⃗| if for a unit vector 𝒂 ⃗⃗) ∙ (𝒙
⃗⃗ − 𝒂 ⃗⃗ + 𝒂
⃗⃗) = 15. (CBSE 2013)
⃗⃗|2 − |𝒂
|𝒙 ⃗⃗|2 = 15
⃗⃗|2 − 1 = 15
|𝒙
⃗⃗|2 = 16
|𝒙
|𝒙
⃗⃗| = 4
Example 1.259
⃗⃗ and 𝑩
Two vectors 𝑨 ⃗⃗ + 𝑩
⃗⃗⃗ are such that |𝑨 ⃗⃗ − 𝑩
⃗⃗⃗| = |𝑨 ⃗⃗⃗|. The angle between the two vectors will be (BITSAT 2011)
P a g e 81 | 168
Get all the files at: https://bit.ly/azizhandouts
Aziz Manva (azizmanva@gmail.com)
2 2
⃗⃗ ∙ ⃗𝒃⃗ + | ⃗𝒃⃗| = | 𝒂
⃗⃗|2 + 2𝒂
|𝒂 ⃗⃗|2 − 2𝒂 ⃗⃗ ∙ ⃗𝒃⃗ + | ⃗𝒃⃗|
⃗⃗ ∙ 𝒃
2𝒂 ⃗⃗ = − 2𝒂 ⃗⃗
⃗⃗ ∙ 𝒃
4𝒂 ⃗⃗ = 0
⃗⃗ ∙ 𝒃
⃗⃗ = 0
⃗⃗ ∙ 𝒃
𝒂
⃗⃗ ⊥ 𝒃
𝒂 ⃗⃗
𝐴𝑛𝑔𝑙𝑒 𝑏𝑒𝑡𝑤𝑒𝑒𝑛 𝑡𝑤𝑜 𝑣𝑒𝑐𝑡𝑜𝑟𝑠 = 90°
Example 1.260
2 2
Let 𝒂 ⃗⃗ and 𝒄
⃗⃗, 𝒃 ⃗⃗ be three unit vectors such that |𝒂 ⃗⃗| + |𝒂
⃗⃗ − 𝒃 ⃗⃗|2 = 8. Then |𝒂
⃗⃗ − 𝒄 ⃗⃗| + |𝒂
⃗⃗ + 2𝒃 ⃗⃗|2 is equal to
⃗⃗ + 2𝒄
(JEE Main, 2020, 2 Sep, Shift-I)
2
⃗⃗| + |𝒂
⃗⃗ − 𝒃
Expand |𝒂 ⃗⃗|2 = 8 to get:
⃗⃗ − 𝒄
2
⃗⃗|2 − 2𝒂
|𝒂 ⃗⃗ + |𝒃
⃗⃗ ∙ 𝒃 ⃗⃗| + |𝒂
⃗⃗|2 − 2𝒂 ⃗⃗|2 = 8
⃗⃗ + |𝒄
⃗⃗ ∙ 𝒄
2
Substitute |𝒂 ⃗⃗| = |𝒄
⃗⃗|2 = |𝒃 ⃗⃗|2 = 1:
−2𝒂 ⃗⃗ − 2𝒂
⃗⃗ ∙ 𝒃 ⃗⃗ ∙ 𝒄
⃗⃗ = 4
𝒂 ⃗⃗ + 𝒂
⃗⃗ ∙ 𝒃 ⃗⃗ ∙ 𝒄
⃗⃗ = −2
2
⃗⃗| + |𝒂
⃗⃗ + 2𝒃
Expand |𝒂 ⃗⃗|2 to get:
⃗⃗ + 2𝒄
2
|𝒂 ⃗⃗ ∙ ⃗𝒃⃗ + 4|𝒃
⃗⃗|2 + 4𝒂 ⃗⃗| + |𝒂
⃗⃗|2 + 4𝒂
⃗⃗ ∙ 𝒄 ⃗⃗|2
⃗⃗ + 4|𝒄
2
Substitute |𝒂 ⃗⃗| = |𝒄
⃗⃗|2 = |𝒃 ⃗⃗|2 = 1:
⃗⃗ ∙ ⃗𝒃⃗ + 4𝒂
10 + 4(𝒂 ⃗⃗ ∙ 𝒄
⃗⃗)
Substitute 𝒂 ⃗⃗ + 𝒂
⃗⃗ ∙ 𝒃 ⃗⃗ ∙ 𝒄
⃗⃗ = −2:
10 + 4(−2) = 10 − 8 = 2
Applying the property (𝑎 + 𝑏 + 𝑐)2 = 𝑎2 + 𝑏 2 + 𝑐 2 + 2(𝑎𝑏 + 𝑏𝑐 + 𝑐𝑎) to the LHS of the given property
immediately gives us the RHS.
Example 1.262
If ⃗𝒂⃗, ⃗𝒃⃗, ⃗𝒄⃗ are three unit vectors such that ⃗𝒂⃗ + ⃗𝒃⃗ + ⃗𝒄⃗ = ⃗𝟎⃗, where ⃗𝟎⃗ is the null vector, then ⃗𝒂⃗ ∙ ⃗𝒃⃗ + ⃗𝒃⃗ ∙ ⃗𝒄⃗ + ⃗𝒄⃗ ∙ ⃗𝒂⃗ is
(BITSAT 2013)
⃗⃗ + ⃗𝒃⃗ + 𝒄
𝒂 ⃗⃗ = ⃗𝟎⃗
Square both sides:
2
|𝒂 ⃗⃗| + |𝒄
⃗⃗|2 + |𝒃 ⃗⃗ ∙ ⃗𝒃⃗ + ⃗𝒃⃗ ∙ 𝒄
⃗⃗|2 + 2(𝒂 ⃗⃗ + 𝒄
⃗⃗ ∙ 𝒂
⃗⃗) = 0
P a g e 82 | 168
Get all the files at: https://bit.ly/azizhandouts
Aziz Manva (azizmanva@gmail.com)
2
Substitute |𝒂 ⃗⃗| = |𝒄
⃗⃗|2 = |𝒃 ⃗⃗|2 = 1:
1 + 1 + 1 + 2(𝒂 ⃗⃗ + 𝒃
⃗⃗ ∙ 𝒃 ⃗⃗ ∙ 𝒄
⃗⃗ + 𝒄
⃗⃗ ∙ 𝒂
⃗⃗) = 0
3
⃗⃗ ∙ ⃗𝒃⃗ + ⃗𝒃⃗ ∙ 𝒄
𝒂 ⃗⃗ + 𝒄
⃗⃗ ∙ 𝒂
⃗⃗ = −
2
Example 1.263
2 2
⃗⃗, ⃗𝒃⃗, and 𝒄
If 𝒂 ⃗⃗ − ⃗𝒃⃗| + |𝒃
⃗⃗ are unit vectors, then |𝒂 ⃗⃗ − 𝒄
⃗⃗| + |𝒄 ⃗⃗|2 does not exceed (JEE Advanced, 2001-
⃗⃗ − 𝒂
Screening)
⃗⃗ + ⃗𝒃⃗ + 𝒄
To find the maximum value of the above, square both sides of |𝒂 ⃗⃗| ≥ 0:
2
|𝒂 ⃗⃗| + |𝒄
⃗⃗|2 + |𝒃 ⃗⃗ ∙ ⃗𝒃⃗ + ⃗𝒃⃗ ∙ 𝒄
⃗⃗|2 + 2(𝒂 ⃗⃗ + 𝒄
⃗⃗ ∙ 𝒂
⃗⃗) ≥ 0
2
Substitute |𝒂 ⃗⃗| = |𝒄
⃗⃗|2 = |𝒃 ⃗⃗|2 = 1:
3 + 2(𝒂 ⃗⃗ + 𝒃
⃗⃗ ∙ 𝒃 ⃗⃗ ∙ 𝒄
⃗⃗ + 𝒄
⃗⃗ ∙ 𝒂
⃗⃗) ≥ 0
Subtract three from both sides:
⃗⃗ ∙ ⃗𝒃⃗ + ⃗𝒃⃗ ∙ 𝒄
2(𝒂 ⃗⃗ + 𝒄 ⃗⃗ ∙ 𝒂
⃗⃗) ≥ −3
Multiply by −1 both sides and flip the inequality:
−2(𝒂 ⃗⃗ ∙ ⃗𝒃⃗ + ⃗𝒃⃗ ∙ 𝒄
⃗⃗ + 𝒄 ⃗⃗ ∙ 𝒂
⃗⃗) ≤ 3
Add six to both sides:
6
⏟− 2(𝒂 ⃗⃗ ∙ 𝒃⃗⃗ + 𝒃⃗⃗ ∙ 𝒄
⃗⃗ + 𝒄 ⃗⃗ ∙ 𝒂
⃗⃗) ≤ 9
𝑬𝒙𝒑𝒓𝒆𝒔𝒔𝒊𝒐𝒏 𝑰
Since Expression I was what we wanted to find the maximum value of, its maximum value is:
9
P a g e 83 | 168
Get all the files at: https://bit.ly/azizhandouts
Aziz Manva (azizmanva@gmail.com)
1.266: Component
⃗⃗ = (𝑥2 , 𝑦2 , 𝑧2 ), the dot product of the vectors is given by:
⃗⃗ = (𝑥1 , 𝑦1 , 𝑧1 ), 𝒃
Given two vectors 𝒂
𝑥2
⃗⃗ 𝑦
⃗⃗ ∙ 𝒃 = (𝑥1 , 𝑦1 , 𝑧1 ) ( 2 ) = 𝑥1 𝑥2 + 𝑦1 𝑦2 + 𝑧1 𝑧2
𝒂
𝑧2
Two-dimensional version
⃗⃗
⃗⃗ ∙ 𝒃
𝒂
Write each vector in component form:
(𝑥1 𝒊̂ + 𝑦1 𝒋̂) ∙ (𝑥2 𝒊̂ + 𝑦2 𝒋̂)
Use the distributive property:
𝑥1 𝑥2 𝒊̂ ∙ 𝒊̂ + 𝑦1 𝑦2 𝒋̂ ∙ 𝒋̂ + 𝑥1 𝑦2 𝒊̂ ∙ 𝒋̂ + 𝑦1 𝑥2 𝒋̂ ∙ 𝒊̂
Substitute 𝑖̂ ∙ 𝑖̂ = 𝑗̂ ∙ 𝑗̂ = 1 and 𝑖̂ ∙ 𝑗̂ = 𝑗̂ ∙ 𝑖̂ = 0
𝑥1 𝑥2 (1) + 𝑦1 𝑦2 (1) + 𝑥1 𝑦2 (0) + 𝑦1 𝑥2 (0)
Simplify:
𝑥1 𝑥2 + 𝑦1 𝑦2
Three-dimensional version
⃗⃗
⃗⃗ ∙ 𝒃
𝒂
Write each vector in component form:
̂) ∙ (𝑥2 𝒊̂ + 𝑦2 𝒋̂ + +𝑧2 𝒌
(𝑥1 𝒊̂ + 𝑦1 𝒋̂ + 𝑧1 𝒌 ̂)
̂∙𝒌
Use the distributive property to get nine terms. Note that we substitute 𝒊̂ ∙ 𝒊̂ = 𝒋̂ ∙ 𝒋̂ = 𝒌 ̂ = 1 and every other
dot product is zero, we are left with
𝑥1 𝑥2 (1) + 𝑦1 𝑦2 (1) + 𝑧1 𝑧2 (1)
Simplify:
𝑥1 𝑥2 + 𝑦1 𝑦2 + 𝑧1 𝑧2
Example 1.267
1
⃗⃗ given that 𝒂
⃗⃗ and 𝒃
A. Find the dot product of 𝒂 ⃗⃗ = (−3, , −4)
⃗⃗ = (2,5, −2), 𝒃
2
1 5
⃗⃗ ∙ ⃗𝒃⃗ = (2)(−3) + (5) ( ) + (−2)(−4) = −6 + + 8 = 4.5
𝒂
2 2
Example 1.268
The scalar product of the vector 𝒂 ̂ with a unit vector along the sum of the vectors 𝒃 = 2𝒊̂ + 4𝒋̂ − 5𝒌
⃗⃗ = 𝒊̂ + 𝒋̂ + 𝒌 ̂
̂ is equal to 1. Find the value of 𝜆 and hence find the unit vector along 𝒃 + 𝒄. (CBSE
and 𝒄 = 𝜆𝒊̂ + 2𝒋̂ + 3𝒌
2008,2009,2014,2019)
⃗⃗ + 𝒄
Unit vector in the direction of 𝒃 ⃗⃗ is
⃗⃗ + 𝒄
𝒃 ⃗⃗ (2 + 𝜆, 6, −2) (2 + 𝜆, 6, −2)
⃗⃗ =
𝒗 = =
⃗⃗ + 𝒄
|𝒃 ⃗⃗| √(2 + 𝜆)2 + 62 + (−2)2 √(2 + 𝜆)2 + 40
⃗⃗ ∙ 𝒗
As per the given condition, 𝒂 ⃗⃗ = 1, which means:
(2 + 𝜆, 6, −2)
(1,1,1) ∙ =1
√(2 + 𝜆)2 + 40
(1,1,1) ∙ (2 + 𝜆, 6, −2) = √(2 + 𝜆)2 + 40
(1)(2 + 𝜆) + (1)(6) + (1)(−2) = √(2 + 𝜆)2 + 40
P a g e 84 | 168
Get all the files at: https://bit.ly/azizhandouts
Aziz Manva (azizmanva@gmail.com)
(6 + 𝜆) = √(2 + 𝜆)2 + 40
Square both sides:
(6 + 𝜆)2 = (2 + 𝜆)2 + 40
36 + 12𝜆 + 𝜆2 = 4 + 4𝜆 + 𝜆2 + 40
8𝜆 = 8
𝜆=1
Example 1.269
Using the component form of the dot product, prove the
A. The commutative property
B. The distributive property
C. That the dot product of a vector with itself is the square of its magnitude
Part A
⃗⃗ = 𝒃
⃗⃗ ∙ 𝒃
𝒂 ⃗⃗ ∙ 𝒂
⃗⃗
⃗⃗ = 𝑥1 𝑥2 + 𝑦1 𝑦2 + 𝑧1 𝑧2 = 𝑥2 𝑥1 + 𝑦2 𝑦1 + 𝑧2 𝑧1 = 𝒃
⃗⃗ ∙ 𝒃
𝐿𝐻𝑆 = 𝒂 ⃗⃗ ∙ 𝒂
⃗⃗ = 𝑅𝐻𝑆
Part B
⃗⃗ = (𝑥, 𝑦, 𝑧), ⃗𝒃⃗ = (𝑥1 , 𝑦1 , 𝑧1 ), 𝒄
Let 𝒂 ⃗⃗ = (𝑥2 , 𝑦2 , 𝑧2 ). Then:
⃗⃗ + 𝒄
⃗⃗ ∙ (𝒃
𝒂 ⃗⃗) = (𝑥, 𝑦, 𝑧) ∙ (𝑥1 + 𝑥2 , 𝑦1 + 𝑦2 , 𝑧1 + 𝑧2 )
Expand
= 𝑥𝑥1 + 𝑥𝑥2 + 𝑦𝑦1 + 𝑦𝑦2 + 𝑧𝑧1 + 𝑧𝑧2
Rearrange:
=⏟𝑥𝑥1 + 𝑦𝑦1 + 𝑧𝑧1 + ⏟ 𝑥𝑥2 + 𝑦𝑦2 + 𝑧𝑧2
⃗⃗
⃗⃗∙𝒃
𝒂 ⃗⃗∙𝒄
𝒂 ⃗⃗
=𝒂 ⃗⃗ + 𝒂
⃗⃗ ∙ 𝒃 ⃗⃗ ∙ 𝒄
⃗⃗
Part C
⃗⃗ = (𝑥, 𝑦, 𝑧). Then:
Let 𝒂
⃗⃗ = (𝑥, 𝑦, 𝑧) ∙ (𝑥, 𝑦, 𝑧) = 𝑥 2 + 𝑦 2 + 𝑧 2 = |𝒂
⃗⃗ ∙ 𝒂
𝐿𝐻𝑆 = 𝒂 ⃗⃗|2
⃗⃗ ∙ 𝒙
𝒙 ⃗⃗|2
⃗⃗ = |𝒙
Example 1.271
⃗⃗ = 2𝒊̂ + 𝜆𝒋 + 𝒌 and 𝒃 = 𝒊 − 2𝒋 + 2𝒌 are perpendicular to each
A. Write the value of 𝜆, so that the vectors 𝒂
other. (CBSE 2008,2013)
B. For what value of 𝜆 are the vectors 𝒊 + 2𝜆𝒋 + 𝒌 and 2𝒊 + 𝒋 − 3𝒌 perpendicular. (CBSE 2011)
C. Find the value of 𝜆, if the vectors 2𝒊 + 𝜆𝒋 + 3𝒌 and 3𝒊 + 2𝒋 − 4𝒌 are perpendicular to each other. (CBSE
2010)
Part A 4 = 2𝜆
(2, 𝜆, 1) ∙ (1, −2,2) = 0 𝜆=2
2 − 2𝜆 + 2 = 0 Part B
P a g e 85 | 168
Get all the files at: https://bit.ly/azizhandouts
Aziz Manva (azizmanva@gmail.com)
⃗⃗ ∙ ⃗𝒃⃗ = |𝒂
Explain why the formula 𝒂 ⃗⃗| cos 𝜃 is not useful in solving the above questions.
⃗⃗||𝒃
Example 1.272
̂, ⃗𝒃⃗ = 2𝒊̂ + 4𝒋̂ + 𝒌
If the vectors ⃗𝒂⃗ = 𝒊̂ − 𝒋̂ + 2𝒌 ̂, and ⃗𝒄⃗ = 𝜆𝒊̂ + 𝒋̂ + 𝜇𝒌
̂ are mutually orthogonal, then (𝜆, 𝜇) is equal
to: (JEE Main 2010)
2𝜆 − 2 + 4𝜇 = 0
2𝜆 + 4 + 𝜇 = 0
(𝜆, 𝜇) = (−3,2)
Example 1.274
̂ and 𝒃
⃗⃗ = 𝒊̂ + 2𝒋̂ + 3𝒌
If 𝜃 be the angle between vectors 𝒂 ⃗⃗ = 3𝒊 + 2𝒋 + 𝒌, then 𝑐𝑜𝑠 𝜃 = (BITSAT 2007)
⃗⃗ ∙ ⃗𝒃⃗
𝒂 1×1+2×2+3×1 3 + 4 + 3 10 5
cos 𝜃 = = = = =
|𝒂
⃗⃗||𝒃⃗⃗| √1 + 2 + 3 √3 + 2 + 1
2 2 2 2 2 2 √14√14 14 7
Example 1.275
̂ and 𝑩
⃗⃗ = 𝒊̂ + 𝒋̂ − 2𝒌
Find the angle between the vectors 𝑨 ̂. (BITSAT 2007)
⃗⃗⃗ = −𝒊̂ + 2𝒋̂ − 𝒌
⃗⃗ ∙ ⃗𝒃⃗
𝒂 −1 + 2 + 2 3 1
𝜃 = cos −1 ( ) = cos −1 ( ) = cos−1 ( ) = cos −1 ( ) = 60°
|𝒂
⃗⃗||𝒃⃗⃗| 2 2 2 2 2
√1 + 1 + 2 √1 + 2 + 1 2 6 2
Example 1.276
P a g e 86 | 168
Get all the files at: https://bit.ly/azizhandouts
Aziz Manva (azizmanva@gmail.com)
The angle between any two diagonals of a cube can be written in the form tan−1 𝑥. Find the value of 𝑥. (BITSAT
2014, Adapted)
Note: There are two kinds of diagonals of a cube: space diagonals and face diagonals. From the options (which
are removed here), we know that the diagonals referred to are space diagonals, since the angle between two
face diagonals is a right angle.
We have cos 𝜃, and we need to find tan 𝜃. Draw a reference triangle with
𝑎𝑑𝑗 = 1, ℎ𝑦𝑝 = 3 and find 𝑜𝑝𝑝 = 2√2.
Example 1.277
̂, 2𝒊̂ + 5𝒋̂, 3𝒊̂ + 2𝒋̂ − 3𝒌
If 𝒊̂ + 𝒋̂ + 𝒌 ̂ and 𝒊̂ − 6𝒋̂ − 𝒌
̂ respectively are the position vectors of point A, B, C and D, then
find:
A. the angle between the straight lines AB and CD.
B. whether 𝑨𝑩 ⃗⃗⃗⃗⃗⃗⃗ and 𝑪𝑫
⃗⃗⃗⃗⃗⃗⃗ are collinear or not. (CBSE 2019)
Part A
If the angle between the lines is 𝜃, then:
⃗⃗⃗⃗⃗⃗⃗ ∙ 𝑪𝑫
𝑨𝑩 ⃗⃗⃗⃗⃗⃗⃗
cos 𝜃 =
⃗⃗⃗⃗⃗⃗⃗||𝑪𝑫
|𝑨𝑩 ⃗⃗⃗⃗⃗⃗⃗|
Substitute using the definition:
(2 − 1,5 − 1,0 − 1) ∙ (1 − 3, −6 − 2, −1 + 3) (1,4, −1) ∙ (−2, −8,2) −2 − 32 − 2 36
= = = =− = −1
√12 + 42 + (−1)2 √22 + (−8)2 + 22 √18 √72 (3√2)(6√2) 36
Since
cos 𝜃 = −1 ⇒ 𝜃 = cos−1(−1) = 𝜋
P a g e 87 | 168
Get all the files at: https://bit.ly/azizhandouts
Aziz Manva (azizmanva@gmail.com)
Part B
Since the angle between the lines is 𝜋, they are in
opposite directions.
They are collinear.
Example 1.278
17: A hall has a square floor of dimension 10𝑚 × 10𝑚 (see the figure) and
1
vertical walls. If the ∠𝐺𝑃𝐻 between the diagonals 𝐴𝐺 and 𝐵𝐻 is cos−1 5, then
the height of the hall (in m) is: (JEE Main 2021, 26 Aug, Shift-II)
Challenge 1.280
2𝜋 4𝜋 𝜋
Let 𝑎, 𝑏, 𝑐 ∈ ℝ such that 𝑎2 + 𝑏 2 + 𝑐 2 = 1. If 𝑎 cos 𝜃 = 𝑏 cos (𝜃 + 3 ) = 𝑐 cos (𝜃 + 3 ), where 𝜃 = 9 , then the
angle between the vectors 𝑎𝒊̂ + 𝑏𝒋̂ + 𝑐𝒌 ̂, and 𝑏𝒊̂ + 𝑐𝒋̂ + 𝑎𝒌
̂ is: (JEE Main 2020, 3 Sep, Shift-II)
Substitute values from 𝑆𝑦𝑠𝑡𝑒𝑚 𝐼 in the angle between the vectors to get:
P a g e 88 | 168
Get all the files at: https://bit.ly/azizhandouts
Aziz Manva (azizmanva@gmail.com)
𝑘 𝑘 𝑘 𝑘 𝑘 𝑘
𝑎𝑏 + 𝑏𝑐 + 𝑐𝑎 = ( )( )+( )( )+( )( )
cos 𝜃 cos (𝜃 + 2𝜋) 2𝜋 4𝜋 4𝜋 cos 𝜃
cos (𝜃 + 3 ) cos (𝜃 + 3 ) cos (𝜃 + 3 )
3
𝜋 1
Substitute cos ( ) = , cos(𝜃 + 𝜋) = − cos 𝜃
3 2
1
= 2(− cos 𝜃) ( ) + cos 𝜃 = − cos 𝜃 + cos 𝜃 = 0
2
C. Projection
⃗⃗ ∙ ⃗𝒃⃗ |𝒂
𝒂 ⃗⃗| cos 𝜃
⃗⃗||𝒃
𝐿𝐻𝑆 = = = |𝒂
⃗⃗| cos 𝜃 = 𝑅𝐻𝑆
⃗⃗|
|𝒃 ⃗⃗|
|𝒃
Example 1.282
If |𝒂 ⃗⃗| = 3, and 𝒂
⃗⃗| = 2, |𝒃 ⃗⃗ = 3, find the projection of |𝒃
⃗⃗ ∙ 𝒃 ⃗⃗| on |𝒂
⃗⃗|. (CBSE 2010)
Note: By projection here is meant the length of the projection, not the projection vector.
⃗𝒂⃗ ∙ ⃗𝒃⃗ 3
=
|𝒂
⃗⃗| 2
P a g e 89 | 168
Get all the files at: https://bit.ly/azizhandouts
Aziz Manva (azizmanva@gmail.com)
Example 1.283
̂ and ⃗𝒃⃗ = 2𝒊̂ + 6𝒋̂ + 3𝒌
⃗⃗ = 7𝒊̂ + 𝒋̂ − 4𝒌
A. If 𝒂 ̂, then find the projection of 𝒂 ⃗⃗ on ⃗𝒃⃗. (CBSE 2013,2015)
B. Write the projection of the vector 𝒂 ̂ on the vector ⃗𝒃⃗ = 𝒊̂ + 2𝒋̂ + 2𝒌
⃗⃗ = 2𝒊̂ − 𝒋̂ + 𝒌 ̂. (CBSE 2014)
C. Write the projection of (𝒃 ⃗⃗ + 𝒄
⃗⃗) on 𝒂
⃗⃗, where 𝒂 ̂, ⃗𝒃⃗ = 𝒊̂ + 2𝒋̂ − 2𝒌
⃗⃗ = 2𝒊̂ − 2𝒋̂ + 𝒌 ̂ and 𝒄⃗⃗ = 2𝒊̂ − 𝒋̂ + 4𝒌 (CBSE
2013)
Part A
⃗⃗ ∙ ⃗𝒃⃗ (7,1, −4) ∙ (2,6,3) 14 + 6 − 12 8
𝒂
= = =
⃗⃗|
|𝒃 √22 + 62 + 32 √49 7
Part B
⃗⃗ ∙ ⃗𝒃⃗ (2, −1,1) ∙ (1,2,2) 2 − 2 + 2 2
𝒂
= = =
⃗⃗|
|𝒃 √12 + 22 + 22 √9 3
Part C
⃗⃗ + 𝒄
(𝒃 ⃗⃗ (3,1,2) ∙ (2, −2,1) 6 − 2 + 2 6
⃗⃗) ∙ 𝒂
= = = =2
|𝒂
⃗⃗| √22 + (−2)2 + 12 √9 3
Example 1.284
̂
⃗⃗ = 𝑎𝒊̂ + 𝑏𝒋̂ + 𝑐𝒌
𝒗
⃗⃗ along
Find the projection of 𝒗
A. 𝒊̂
B. 𝒋̂
C. 𝒌 ̂
Part A
⃗⃗ ∙ 𝒊̂ (𝑎, 𝑏, 𝑐) ∙ (1,0,0) 𝑎
𝒗
= = =𝑎
|𝒊̂| 1 1
Part B
⃗⃗ ∙ 𝒋̂ (𝑎, 𝑏, 𝑐) ∙ (0,1,0) 𝑏
𝒗
= = =𝑏
|𝒋̂| 1 1
Part C
̂ (𝑎, 𝑏, 𝑐) ∙ (0,0,1) 𝑐
⃗⃗ ∙ 𝒌
𝒗
= = =𝑐
̂|
|𝒌 1 1
Part A
⃗𝒂⃗ ∙ ⃗𝒃⃗ (𝜆, 1,4) ∙ (2,6,3) 2𝜆 + 6 + 12 2𝜆 + 18
= = = = ⏟4
⃗⃗|
|𝒃 √22 + 62 + 32 √49 7 𝐺𝑖𝑣𝑒𝑛
2𝜆 + 18 = 28
2𝜆 = 10
𝜆=5
P a g e 90 | 168
Get all the files at: https://bit.ly/azizhandouts
Aziz Manva (azizmanva@gmail.com)
Part B
⃗⃗ ∙ ⃗𝒃⃗ (𝜆, 1,4) ∙ (2, 𝜆, 3) 2𝜆 + 𝜆 + 12
𝒂 3𝜆 + 12
= = = = ⏟4
⃗⃗|
|𝒃 2
√2 + 𝜆 + 32 2 2
√𝜆 + 13 √𝜆2 + 13 𝐺𝑖𝑣𝑒𝑛
3𝜆 + 12 = 4√𝜆2 + 13
9𝜆 + 72𝜆 + 144 = 16(𝜆2 + 13)
2
7𝜆2 − 72𝜆 + 64 = 0
Example 1.286
̂ on the sum of the two vectors 2𝒊̂ + 4𝒋̂ − 5𝒌
18: If the projection of the vector 𝒊̂ + 2𝒋̂ + 𝒌 ̂ and −𝜆𝒊̂ + 2𝒋̂ + 3𝒌
̂ is 1,
then 𝜆 is equal to: (JEE Main 2021, 25 July, Shift-II)
⃗⃗ is
⃗⃗ on 𝒃
The projection of 𝒂
⃗⃗
⃗⃗ ∙ 𝒃
𝒂
=1
⃗⃗|
|𝒃
⃗⃗ = (1,2,1) and ⃗𝒃⃗ = (2,4, −5) + (−𝜆, 2,3) = (2 − 𝜆, 6, −2) in the above:
Substitute 𝒂
(1,2,1) ∙ (2 − 𝜆, 6, −2)
=1
|(2 − 𝜆, 6, −2)|
Expand:
144 − 24𝜆 + 𝜆2 = 4 − 4𝜆 + 𝜆2 + 36 + 4
D. Coplanar Vectors
Example 1.287
⃗⃗ be a vector in the plane containing vectors 𝒂
24: Let 𝒙 ̂ and 𝒃
⃗⃗ = 2𝒊̂ − 𝒋̂ + 𝒌 ⃗⃗ = 𝒊̂ + 2𝒋̂ − 𝒌
̂. If the vector 𝒙
⃗⃗ is
17√6
̂ ) and its projection on 𝒂
perpendicular to (3𝒊̂ + 2𝒋̂ − 𝒌 ⃗⃗ is ⃗⃗|2 is equal to: (JEE Main 2021,
, then the value of |𝒙
2
17 March, Shift-II)
Let
P a g e 91 | 168
Get all the files at: https://bit.ly/azizhandouts
Aziz Manva (azizmanva@gmail.com)
⃗⃗ = λ𝒂
𝒙 ⃗⃗ = λ(2, −1,1) + 𝜇(1,2, −1) = (2𝜆 + 𝜇, −𝜆 + 2𝜇, 𝜆 − 𝜇)
⃗⃗ + 𝜇𝒃
̂)
⃗⃗ is perpendicular to (3𝒊̂ + 2𝒋̂ − 𝒌
Since the vector 𝒙
̂) = 0
⃗⃗ ∙ (3𝒊̂ + 2𝒋̂ − 𝒌
𝒙
⃗⃗ = (2𝜆 + 𝜇, −𝜆 + 2𝜇, 𝜆 − 𝜇)
Substitute 𝒙
̂) = 0
(2𝜆 + 𝜇, −𝜆 + 2𝜇, 𝜆 − 𝜇) ∙ (3𝒊̂ + 2𝒋̂ − 𝒌
17√6
⃗⃗ on 𝒂
Projection of 𝒙 ⃗⃗ is :
2
⃗⃗ ∙ 𝒂
𝒙 ⃗⃗ 17√6
=
|𝒂⃗⃗| 2
⃗⃗ = (2𝜆 + 𝜇, −𝜆 + 2𝜇, 𝜆 − 𝜇), 𝒂
Substitute 𝒙 ⃗⃗ = 2𝒊̂ − 𝒋̂ + 𝒌̂
(2𝜆 + 𝜇, −𝜆 + 2𝜇, 𝜆 − 𝜇) ∙ (2, −1,1) 17√6
=
√22 + (−1)2 + 12 2
⃗⃗:
Finally, calculate the square of the magnitude of 𝒙
2
⃗⃗| = √132 + (−14)2 + 112 = 486
|𝒙
E. Geometry
Example 1.288
Show that the line drawn from the center of a circle, bisecting the chord, is perpendicular to the chord.
P a g e 92 | 168
Get all the files at: https://bit.ly/azizhandouts
Aziz Manva (azizmanva@gmail.com)
Example 1.289
A. Write the number of vectors of unit length perpendicular to both the
̂, and ⃗𝒃⃗ = 𝒋̂ + 𝒌
⃗⃗ = 2𝒊̂ + 𝒋̂ + 2𝒌
vectors 𝒂 ̂. (CBSE 2016)
B. When will two vectors not have exactly two vectors perpendicular to
them?
Part A
Two vectors
Part B
When these two vectors are parallel.
Example 1.290
⃗⃗ + ⃗𝒃⃗ and 𝒂
Find the unit vector perpendicular to both of the vectors 𝒂 ⃗⃗ − ⃗𝒃⃗ where 𝒂 ̂ and ⃗𝒃⃗ = 𝒊̂ + 2𝒋̂ +
⃗⃗ = 𝒊̂ + 𝒋̂ + 𝒌
̂. (CBSE 2014)
3𝒌
Let the unit vector that we wish to find be We need a third equation. Since we wish to find a
̂
𝒓̂ = 𝑥𝒊̂ + 𝑦𝒋̂ + 𝑧𝒌 unit vector:
Since 𝒓̂ ⊥ (𝒂 ⃗⃗), we must have
⃗⃗ + 𝒃 𝒓̂ = 1
|𝑥𝒊̂ + 𝑦𝒋̂ + 𝑧𝒌̂| = 1
⃗⃗ + ⃗𝒃⃗) = 𝟎
𝒓̂ ∙ (𝒂
(𝑥𝒊̂ + 𝑦𝒋̂ + 𝑧𝒌̂) ∙ (𝟐𝒊̂ + 3𝒋̂ + 4𝒌
̂) = 𝟎 √𝑥 2 + 𝑦 2 + 𝑧 2 = 1
Expand using the dot product: ⏟2 + 𝑦 2 + 𝑧 2 = 1
𝑥
𝑬𝒒𝒖𝒂𝒕𝒊𝒐𝒏 𝑰𝑽
2𝑥 + 3𝑦 + 4𝑧 = 0
⏟
𝑬𝒒𝒖𝒂𝒕𝒊𝒐𝒏 𝑰
Substitute the values from Equation II and Equation
III into Equation IV:
Similarly, 𝒓̂ ⊥ (𝒂⃗⃗ − ⃗𝒃⃗), and hence we must have:
𝑧 2 + (−2𝑧)2 + 𝑧 2 = 1
𝒓̂ ∙ (𝒂
⃗⃗ − 𝒃 ⃗⃗) = 𝟎
6𝑧 2 = 1
(𝑥𝒊̂ + 𝑦𝒋̂ + 𝑧𝒌 ̂ ) ∙ (−𝒋̂ − 𝟐𝒌
̂) = 0 1
𝑥=𝑧=±
Expand using the dot product: √6
−𝑦 − 2𝑧 = 0 2
𝑦 = −2𝑧 𝑦 = −2𝑧 = ±
⏟ √6
𝑬𝒒𝒖𝒂𝒕𝒊𝒐𝒏 𝑰𝑰 The vectors that we want are:
Substitute the value of 𝑦 from Equation II into 1 2 1
𝒊̂ − 𝒋̂ + ̂
𝒌
Equation I:
√6 √6 √6
2𝑥 + 3(−2𝑧) + 4𝑧 = 0 1 2 1
2𝑥 − 2𝑧 = 0 − 𝒊̂ + 𝒋̂ − ̂
𝒌
√6 √6 √6
𝑥=𝑧
⏟
𝑬𝒒𝒖𝒂𝒕𝒊𝒐𝒏 𝑰𝑰𝑰
P a g e 93 | 168
Get all the files at: https://bit.ly/azizhandouts
Aziz Manva (azizmanva@gmail.com)
HW 1
Example 1.291
31: A vector 𝒂 ⃗⃗ = 𝛼𝒊̂ + 2𝒋̂ + 𝛽𝒌 ̂ (𝛼, 𝛽 ∈ ℝ) lies in the plane of the vectors 𝒃
⃗⃗ = 𝒊̂ + 𝒋̂ and 𝒄 ̂ . If 𝒂
⃗⃗ = 𝒊̂ − 𝒋̂ + 4𝒌 ⃗⃗
bisects the angle between 𝒃 ⃗⃗ and 𝒄⃗⃗, then:
A. 𝑎. 𝒊̂ + 3 = 0
B. 𝑎. 𝑘 + 2 = 0
C. 𝑎. 𝒊̂ + 1 = 0
D. 𝑎. 𝑘 + 4 = 0 (JEE Main 2020, 7 Jan, Shift-I)
B. Projections/Dot Product
Example 1.292
37: Let 𝐴𝐵𝐶𝐷 be a parallelogram such that 𝑨𝑩⃗⃗⃗⃗⃗⃗⃗ = 𝒒
⃗⃗, ⃗⃗⃗⃗⃗⃗⃗
𝑨𝑫 = 𝒑 ⃗⃗ and ∠𝐵𝐴𝐷 be an acute angle. If 𝒓
⃗⃗ is the vector that
coincides with the altitude directed from the vertex 𝐵 to the side AD, then 𝒓 ⃗⃗ is given by:
3(𝑝∙𝑞)
A. 𝑟 = 3𝑞 𝑝∙𝑝
𝑝
B. r=-q+(p\bulletq)/(p\bulletp) p
C. r=q-(p\bulletq)/(p\bulletp) p
D. r=-3q+3(p\bulletq)/(p\bulletp) p
C. Square of a Difference
Example 1.294
⃗⃗ and ⃗𝒃⃗ are unit vectors, then the greatest value of √3|𝒂
29: If 𝒂 ⃗⃗ + ⃗𝒃⃗| + |𝒂
⃗⃗ − ⃗𝒃⃗| is: (JEE Main 2020, 6 Sep, Shift-I)
Example 1.295
28: Let the vectors 𝒂 ⃗⃗, 𝒄
⃗⃗, 𝒃 ⃗⃗ be such that |𝒂 ⃗⃗| = 4, and |𝒄
⃗⃗| = 2, |𝒃 ⃗⃗| = 4. If the projection of 𝒃 ⃗⃗ on 𝒂
⃗⃗ is equal to the
projection of ⃗𝒄⃗ on ⃗𝒂⃗ and ⃗𝒃⃗ is perpendicular to ⃗𝒄⃗, then the value of |𝒂 ⃗⃗ + ⃗𝒃⃗ − ⃗𝒄⃗| is: (JEE Main 2020, 5 Sep, Shift-II)
Example 1.296
41: Let 𝒖 ⃗⃗, 𝒗
⃗⃗, 𝒘
⃗⃗⃗⃗ be such that |𝒖
⃗⃗| = 1, |𝒗
⃗⃗| = 2, |𝒘
⃗⃗⃗⃗| = 3. If the projection 𝒗 ⃗⃗ along 𝒖
⃗⃗ is equal to that of 𝒘
⃗⃗⃗⃗ along 𝒖
⃗⃗ and
⃗⃗, 𝒘
𝒗 ⃗⃗⃗⃗ are perpendicular to each other, then |𝒖 ⃗⃗ − 𝒗 ⃗⃗ + 𝒘
⃗⃗⃗⃗| is equal to: (JEE Main 2004)
P a g e 94 | 168
Get all the files at: https://bit.ly/azizhandouts
Aziz Manva (azizmanva@gmail.com)
Example 1.297
⃗⃗, ⃗𝒃⃗, 𝒄
42: 𝒂 ⃗⃗ + ⃗𝒃⃗ + 𝒄
⃗⃗ are three vectors such that 𝒂 ⃗⃗ = 0, |𝒂 ⃗⃗| = 2, |𝒄
⃗⃗| = 1, |𝒃 ⃗⃗ ∙ ⃗𝒃⃗ + ⃗𝒃⃗ ∙ 𝒄
⃗⃗| = 3, then 𝒂 ⃗⃗ + 𝒄
⃗⃗ ∙ 𝒂
⃗⃗ is equal to:
(JEE Main 2003)
E. Perpendicularity
HW
Example 1.298
̂, 𝒊̂ − 3𝒋̂ − 5𝒌
⃗⃗, for which the points 𝐴, 𝐵, 𝐶 with position vectors 2𝒊̂ − 𝒋̂ + 𝒌
The value(s) of 𝒂 ̂ and 𝑎𝒊̂ − 3𝒋̂ + 𝒌
̂
𝜋
respectively are the vertices of a right-angled triangle with 𝐶 = 2 are: (JEE Main 2006)
Simplify:
(𝑎 − 2, −4,0) ∙ (𝑎 − 1,0, −6) = 0
Use the definition of the dot product:
(𝑎 − 2)(𝑎 − 1) = 0 ⇒ 𝑎 ∈ {1,2}
Example 1.300
Given two vectors are 𝒊̂ − 𝒋̂ and 𝒊̂ + 2𝒋̂, the unit vector coplanar with the two vectors and perpendicular to the
first is: (JEE Main 2002)
⃗⃗ is given by:
Hence, the unit vector in the direction of 𝒂
⃗⃗
𝒂 (𝑥, 𝑥) 1 1
= =( , )
|𝒂
⃗⃗| √2𝑥 √2 √2
P a g e 95 | 168
Get all the files at: https://bit.ly/azizhandouts
Aziz Manva (azizmanva@gmail.com)
HW 2
Example 1.301
35: Let 𝒖⃗⃗ be a vector coplanar with the vectors 𝒂 ̂ and ⃗𝒃⃗ = 𝒋̂ + 𝒌
⃗⃗ = 2𝒊̂ + 3𝒋̂ − 𝒌 ̂. If 𝒖
⃗⃗⃗ is perpendicular to 𝒂
⃗⃗ and 𝒖
⃗⃗ ∙
⃗𝒃⃗ = 24, then |𝒖 2
⃗⃗| is equal to: (JEE Main 2018)
Note that
⃗⃗|2 = 22 + 32 + (−1)2 = 4 + 9 + 1 = 14
|𝒂
⃗𝒃⃗ ∙ 𝒂
⃗⃗ = (0,1,1) ∙ (2,3, −1) = 0 + 3 − 1 = 2
Use the
⃗⃗) ∙ ⃗𝒃⃗ = 24
⃗⃗ + 𝜇𝒃
(𝜆𝒂
2
⃗⃗ ∙ ⃗𝒃⃗ + 𝜇|𝒃
𝜆𝒂 ⃗⃗| = 24
HW 3
Example 1.302
⃗⃗ = 𝒊̂ + 𝒋̂ + √2𝒌
33: Let 𝒂 ̂ ,𝒃
⃗⃗ = 𝑏1 𝒊̂ + 𝑏2 𝒋̂ + √2𝒌
̂ and 𝒄⃗⃗ = 5𝒊̂ + 𝒋̂ + √2𝒌̂ be three vectors such that the projection
vector of 𝒃⃗⃗ on 𝒂
⃗⃗ is 𝒂
⃗⃗. If 𝒂
⃗⃗ + 𝒃⃗⃗ is perpendicular to 𝒄⃗⃗, then |𝒄⃗⃗| is equal to: (JEE Main 2019, 9 Jan, Shift-II)
Example 1.303
̂, 𝒃
⃗⃗ = 2𝒊̂ + 𝜆1 𝒋̂ + 3𝒌
34: Let 𝒂 ⃗⃗ = 4𝒊̂ + (3 − 𝜆2 )𝒋̂ + 6𝒌
̂ and 𝒄 ̂ be three vectors such that
⃗⃗ = 3𝒊̂ + 6𝒋̂ + (𝜆3 − 1)𝒌
⃗⃗ = 2𝒂
𝒃 ⃗⃗ and 𝒂
⃗⃗ is perpendicular to 𝒄 ⃗⃗. Then a possible value of 𝜆1 , 𝜆2 , 𝜆_3 is:
A. (1,3,1)
B. (1,5,1)
C. (-1/2, 4,0)
D. (1/2,4,-2) (JEE Main 2019, 10 Jan, Shift-I)
F. Applications
Example 1.304
⃗⃗, ⃗𝒃⃗, 𝒄
20: Let 𝒂 ⃗⃗ be three mutually perpendicular vectors of the same magnitude and equally inclined at an angle 𝜃
P a g e 96 | 168
Get all the files at: https://bit.ly/azizhandouts
Aziz Manva (azizmanva@gmail.com)
⃗⃗ + 𝒄
⃗⃗ + 𝒃
with the vector 𝒂 ⃗⃗. Then, 36 cos2 𝜃 is equal to: (JEE Main 2021, 20 July, Shift-II)
Example 1.305
21: For 𝑝 > 0, a vector 𝑉2 = 2𝒊̂ + (𝑝 + 1)𝒋̂ is obtained by rotating the vector 𝑣1 = 𝑟𝑡(3𝑝)𝒊̂ + 𝒋̂ by an angle 𝜃
𝛼√3−2
about the origin in counter clockwise direction. If tan 𝜃 = 4√3+3
, then the value of 𝛼 is equal to: (JEE Main 2021,
20 July, Shift-II)
G. Section Formula
Example 1.306
32: Let 𝐴(3,0, −1), 𝐵(2,10,6) and 𝐶(1,2,1) be the vertices of a triangle and 𝑀 be the mid-point of 𝐴𝐶. If 𝐺
divides 𝐵𝑀 in the ratio 2: 1, then cos ∠𝐺𝑂𝐴 (𝑂 being the origin) is equal to: (JEE Main 2019, 10 April, Shift-I)
Example 1.308
2 𝜋
⃗⃗ × ⃗𝒃⃗| if |𝒂
A. (𝐷𝑒𝑓𝑖𝑛𝑖𝑡𝑖𝑜𝑛) Find |𝒂 ⃗⃗| = 3, and (𝒂
⃗⃗| = 2, |𝒃 ⃗⃗, ⃗𝒃⃗) = . (AP EAPCET, 17 Sep 2020, Shift-I)
6
⃗⃗ and ⃗𝒃⃗ are two vectors such that |𝒂
B. (𝐵𝑎𝑐𝑘𝑐𝑎𝑙𝑐𝑢𝑙𝑎𝑡𝑖𝑜𝑛) If 𝒂 ⃗⃗ ∙ ⃗𝒃⃗| = |𝒂
⃗⃗ × ⃗𝒃⃗|, then find the angle between 𝒂
⃗⃗
⃗⃗
and 𝒃. (CBSE 2010)
Part A
2 2 𝜋 2 1 2
⃗⃗
⃗⃗ × 𝒃| = (|𝒂
|𝒂 ⃗⃗
⃗⃗||𝒃| sin 𝜃) = (2 ∙ 3 ∙ sin ) = (2 ∙ 3 ∙ ) = 32 = 9
6 2
Part B
𝜋
|𝒂 ⃗⃗| cos 𝜃 = |𝒂
⃗⃗||𝒃 ⃗⃗| sin 𝜃 ⇒ cos 𝜃 = sin 𝜃 ⇒ 𝜃 =
⃗⃗||𝒃
4
P a g e 97 | 168
Get all the files at: https://bit.ly/azizhandouts
Aziz Manva (azizmanva@gmail.com)
Identify the first step, in the following “proof” where a mistake is made, or incorrect logic is used, explain the
issue, and fix it.
⃗⃗ × ⃗𝒃⃗| is |𝒂
Step I: The magnitude of the cross product |𝒂 ⃗⃗| sin 𝜃. Since both |𝒂
⃗⃗||𝒃 ⃗⃗| are nonnegative, the
⃗⃗| and |𝒃
⃗⃗| is controlled by sin 𝜃.
⃗⃗ × 𝒃
sign of |𝒂
Step II: It is a standard property that −1 ≤ sin 𝑥 ≤ 1
Step III: Multiply −1 ≤ sin 𝜃 ≤ 1 throughout by |𝒂 ⃗⃗| to get −|𝒂
⃗⃗||𝒃 ⃗⃗| ≤ |𝒂
⃗⃗||𝒃 ⃗⃗| sin 𝜃 ≤ |𝒂
⃗⃗||𝒃 ⃗⃗|
⃗⃗||𝒃
Step IV: Substitute |𝒂 ⃗⃗| sin 𝜃 = |𝒂
⃗⃗||𝒃 ⃗⃗| to get −|𝒂
⃗⃗ × 𝒃 ⃗⃗| ≤ |𝒂
⃗⃗||𝒃 ⃗⃗| ≤ |𝒂
⃗⃗ × 𝒃 ⃗⃗|
⃗⃗||𝒃
And multiplying the inequality above with |𝒂 ⃗⃗| sin 𝜃 gives us the correct version
⃗⃗||𝒃
0 ≤ |𝒂 ⃗⃗| sin 𝜃 ≤ |𝒂
⃗⃗||𝒃 ⃗⃗|
⃗⃗||𝒃
⃗⃗ × ⃗𝒃⃗| = |𝒂
Where we can substitute |𝒂 ⃗⃗| sin 𝜃 to get
⃗⃗||𝒃
⃗⃗ × ⃗𝒃⃗| ≤ |𝒂
0 ≤ |𝒂
⏟ ⃗⃗|
⃗⃗||𝒃
𝑪𝒐𝒓𝒓𝒆𝒄𝒕 𝑽𝒆𝒓𝒔𝒊𝒐𝒏
Example 1.311
1 3
A. If |𝒂 ⃗⃗| = , 𝑥 = |𝒂
⃗⃗| = , |𝒃 ⃗⃗|, 𝑀 = 𝑀𝑎𝑥(𝑥), 𝑚 = 𝑀𝑖𝑛(𝑥), then find 𝑀 − 𝛼𝑚, where 𝛼 ∈ ℝ.
⃗⃗ × 𝒃
2 5
1 3
𝑀 − 2𝑚 = ( ) ( ) − 𝛼(0) = 0.3
2 5
MCQ 1.312
𝑀𝑎𝑟𝑘 𝑡ℎ𝑒 𝐶𝑜𝑟𝑟𝑒𝑐𝑡 𝑂𝑝𝑡𝑖𝑜𝑛
Let 𝒖 ⃗⃗ be two non-zero vectors. Then the magnitude of the cross product |𝒖
⃗⃗ and 𝒗 ⃗⃗ × 𝒗
⃗⃗|is always: (AP EAPCET, 18
Sep 2020, Shift-I)
A. ≤ |𝒖 ⃗⃗||𝒗
⃗⃗|
B. = |𝒖 ⃗⃗||𝒗
⃗⃗|
C. ≥ |𝒖 ⃗⃗||𝒗
⃗⃗|
D. = 0
⃗⃗ × ⃗𝒃⃗| ≤ |𝒂
Using the property 𝟎 ≤ |𝒂 ⃗⃗|
⃗⃗||𝒃
𝑂𝑝𝑡𝑖𝑜𝑛 𝐴 𝑖𝑠 𝑐𝑜𝑟𝑟𝑒𝑐𝑡
P a g e 98 | 168
Get all the files at: https://bit.ly/azizhandouts
Aziz Manva (azizmanva@gmail.com)
A. Justify Step I.
B. Justify Step II. Explain why we do not need to reverse the sign of the inequality.
C. Justify Step III. Explain why 𝜃 cannot be in Quadrant III.
Step I: Substitute the definition of the magnitude of the cross product: |𝒂 ⃗⃗| = |𝒂
⃗⃗ × 𝒃 ⃗⃗| sin 𝜃
⃗⃗||𝒃
Step II: Divide both sides |𝒂 ⃗⃗|, and note that |𝒂
⃗⃗||𝒃 ⃗⃗| > 0 since they are both magnitudes and greater than zero.
⃗⃗||𝒃
Step III: sin 𝜃 is positive in the first and second quadrants.
𝑂𝑝𝑡𝑖𝑜𝑛 𝐴
Option B is not correct since:
𝜃 = {0, 𝜋} ⇒ sin 𝜃 = 0 ⇒ |𝒂 ⃗⃗| sin 𝜃 = 0 ⇒ 𝐶𝑟𝑜𝑠𝑠 𝑝𝑟𝑜𝑑𝑢𝑐𝑡 = 0 ≠ 𝑝𝑜𝑠𝑖𝑡𝑖𝑣𝑒
⃗⃗||𝒃
𝑂𝑝𝑡𝑖𝑜𝑛 𝐶
Example 1.316
What can you conclude about the angle 𝜃 between two non-zero vectors if their cross product is zero?
P a g e 99 | 168
Get all the files at: https://bit.ly/azizhandouts
Aziz Manva (azizmanva@gmail.com)
Part A
If the two vectors are in the same direction, then the angle between them is zero.
If the two vectors are in opposite directions, then the angle between them is 𝜋.
⃗⃗ × ⃗𝒃⃗ = |𝒂
𝜃 ∈ {0, 𝜋} ⇒ sin 𝜃 = 0 ⇒ 𝒂 ⃗⃗| sin 𝜃 𝒏
⃗⃗||𝒃 ̂=𝟎
Part B
A vector is parallel to itself. And the angle between two parallel vectors is zero. Hence, the cross product is zero.
Example 1.318
If the cross product of two vectors is zero, what can you conclude about the vectors.
⃗⃗ and ⃗𝒃⃗.
Let the vectors be 𝒂
⃗⃗ × ⃗𝒃⃗ = 0
𝒂
Take the magnitude on both sides:
⃗⃗| = 0
⃗⃗ × 𝒃
|𝒂
Use the definition of magnitude:
|𝒂 ⃗⃗| sin 𝜃 = 0
⃗⃗||𝒃
Use the zero-product property:
|𝒂 ⃗⃗| = 0 𝑂𝑅 sin 𝜃 = 0 ⇒ 𝜃 ∈ {0, 𝜋}
⃗⃗| = 0 𝑂𝑅 |𝒃
If the cross product of two vectors is zero, then one of the following holds:
➢ one of the vectors is zero
➢ both of the vectors are zero
➢ the vectors are collinear.
Note: The collinearity condition is equivalent to saying that one vector is a scalar multiple of the other.
1.320: Properties
Show that:
A. The magnitude of the cross product of two perpendicular vectors is the product of their magnitudes.
B. The magnitude of the cross product of two vectors is maximum when the angle between them is a right
angle. (Assume that their lengths are given, and cannot be changed).
Part A
⃗⃗ × 𝒃
|𝒂 ⃗⃗| = |𝒂⃗⃗||𝒃⃗⃗| sin 𝜃
If the two vectors are perpendicular, then the angle between them is a right angle:
𝜋
𝜃 = ⇒ sin 𝜃 = 1
2
⃗⃗ × ⃗𝒃⃗| = |𝒂
|𝒂 ⃗⃗||𝒃⃗⃗|
Part B
⃗⃗ × ⃗𝒃⃗| = |𝒂
|𝒂 ⃗⃗||𝒃⃗⃗| sin 𝜃
P a g e 100 | 168
Get all the files at: https://bit.ly/azizhandouts
Aziz Manva (azizmanva@gmail.com)
Example 1.323
C. Cross Product
➢ When taking the angle between vectors, we take the smaller angle when they are arranged tip to tip, or
tail to tail. That is 0 ≤ 𝜃 ≤ 𝜋.
➢ 𝒏 ⃗⃗ and ⃗𝒃⃗, such that 𝒂
̂ is a unit vector perpendicular to both 𝒂 ⃗⃗, ⃗𝒃⃗ and 𝒏
̂ form a
right-handed coordinate system.
Example 1.326
̂ × 𝒋̂) ∙ 𝒊̂ + 𝒋̂ ∙ 𝒌
Write the value of (𝒌 ̂ (CBSE 2012)
(−𝒊̂) ∙ 𝒊̂ + 0 = −1 + 0 = −1
P a g e 101 | 168
Get all the files at: https://bit.ly/azizhandouts
Aziz Manva (azizmanva@gmail.com)
D. Distributive Property
Example 1.328
A. Write the value of the following 𝒊̂ × (𝒋̂ + 𝒌 ̂) + 𝒋̂ × (𝒌
̂ + 𝒊̂) + 𝒌
̂ × (𝒊̂ + 𝒋̂) (CBSE 2014)
̂ find (𝒓
⃗⃗ = 𝑥𝒊̂ + 𝑦𝒋̂ + 𝑧𝒌
B. If 𝒓 ⃗⃗ × 𝒊̂) ∙ (𝑟⃗ × 𝒋̂) + 𝑥𝑦
Part A
Use the distributive property:
̂) + (𝒋̂ × 𝒌
(𝒊̂ × 𝒋̂ + 𝒊̂ × 𝒌 ̂ + 𝒋̂ × 𝒊̂) + (𝒌
̂ × 𝒊̂ + 𝒌
̂ × 𝒋̂)
Use the cross products of the unit vectors:
= (𝒌̂−𝒌 ̂) + (𝒊̂ − 𝒌̂ ) + (𝒋̂ − 𝒊̂) = 0
Part B
Find the value of each term:
⃗⃗ × 𝒊̂ = (𝑥𝒊̂ + 𝑦𝒋̂ + 𝑧𝒌
𝒓 ̂) × 𝒊̂ = 𝑥(𝒊̂ × 𝒊̂) + 𝑦(𝒋̂ × 𝒊̂) + 𝑧(𝒌̂ × 𝒊̂) = −𝑦𝒌̂ + 𝑧𝒋̂
⃗⃗ × 𝒋̂ = (𝑥𝒊̂ + 𝑦𝒋̂ + 𝑧𝒌
𝒓 ̂) × 𝒋̂ = 𝑥(𝒊̂ × 𝒋̂) + 𝑦(𝒋̂ × 𝒋̂) + 𝑧(𝒌
̂ × 𝒋̂) = 𝑥𝒌
̂ − 𝑧𝒊̂
And finally:
(𝒓
⃗⃗ × 𝒊̂) ∙ (𝑟⃗ × 𝒋̂) + 𝑥𝑦 = −𝑦𝑥 + 𝑥𝑦 = 0
MCMC 1.329
𝑀𝑎𝑟𝑘 𝑎𝑙𝑙 𝑐𝑜𝑟𝑟𝑒𝑐𝑡 𝑜𝑝𝑡𝑖𝑜𝑛𝑠
⃗⃗ × 𝒒
Which condition ensures that 𝒑 ⃗⃗ = 𝒑
⃗⃗ × 𝒓
⃗⃗ and 𝒑
⃗⃗ ∙ 𝒒
⃗⃗ = 𝒑
⃗⃗ ∙ 𝒓
⃗⃗:
A. 𝒑⃗⃗ = 𝒓
⃗⃗
B. 𝒒⃗⃗ = 𝒓
⃗⃗
C. 𝒑⃗⃗ = 𝒒
⃗⃗
D. 𝒑⃗⃗ + 𝒒
⃗⃗ = 0
E. 𝒑⃗⃗ = 0 (AP EAPCET, 22 Sep 2020, Shift-II, Adapted)
The vectors cannot be collinear and perpendicular simultaneously. Hence, one of them must be zero. 3
⃗⃗ = 0 ⇒ 𝑂𝑝𝑡𝑖𝑜𝑛 𝐸
𝒑
⃗⃗ − 𝒓
𝒒 ⃗⃗ = 𝟎 ⇒ 𝒒⃗⃗ = 𝒓
⃗⃗ ⇒ 𝑂𝑝𝑡𝑖𝑜𝑛 𝐵
Hence, the final answer is:
P a g e 102 | 168
Get all the files at: https://bit.ly/azizhandouts
Aziz Manva (azizmanva@gmail.com)
𝑂𝑝𝑡𝑖𝑜𝑛𝑠 𝐵, 𝐸
E. Anti-Commutative Property
Example 1.331
A. Recall that the Cartesian product of two sets 𝐴 and 𝐵, is the set of all ordered pairs (𝑎, 𝑏) where 𝑎 is in 𝐴
and 𝑏 is in 𝐵. Find the sum of the cross products of each element of the Cartesian product of the set
⃗⃗, ⃗𝒃⃗, 𝒄
{𝒂 ⃗⃗} with itself.
B. Is the above result applicable to the set of 𝑛 vectors {𝒗
⃗⃗⃗⃗⃗,
𝟏 ⃗⃗⃗⃗⃗,
𝒗𝟐 … , ⃗⃗⃗⃗⃗}.
𝒗𝒏 Explain why or why not.
Part A
(𝒂
⃗⃗ × 𝒂 ⃗⃗ × ⃗𝒃⃗) + (𝒄
⃗⃗) + (𝒃 ⃗⃗ × 𝒄⃗⃗) + (𝒂⃗⃗ × ⃗𝒃⃗) + (𝒂⃗⃗ × 𝒄⃗⃗) + (𝒃 ⃗⃗ × 𝒄⃗⃗) + (𝒃 ⃗⃗ × 𝒂⃗⃗) + (𝒄
⃗⃗ × 𝒂 ⃗⃗ × ⃗𝒃⃗)
⃗⃗) + (𝒄
Note that the first three terms all have the cross product of a vector with itself, and hence the output is zero:
0 + 0 + 0 + (𝒂 ⃗⃗ × 𝒃 ⃗⃗) + (𝒂⃗⃗ × 𝒄 ⃗⃗ × 𝒄
⃗⃗) + (𝒃 ⃗⃗) + (𝒃 ⃗⃗ × 𝒂⃗⃗) + (𝒄 ⃗⃗ × 𝒂
⃗⃗) + (𝒄⃗⃗ × 𝒃⃗⃗)
Use the anti-commutative property in the last three terms:
= (𝒂⃗⃗ × 𝒃⃗⃗) + (𝒂⃗⃗ × 𝒄 ⃗⃗ × 𝒄
⃗⃗) + (𝒃 ⃗⃗) + [−(𝒂 ⃗⃗ × 𝒃 ⃗⃗)] + [−(𝒂 ⃗⃗ × 𝒄⃗⃗)] + [−(𝒃 ⃗⃗ × 𝒄⃗⃗)]
And these all add up to zero:
=0
Part B
Since we are taking the Cartesian product of the set with itself, there are two cases:
Case I: You take the cross product of a vector with itself. This is zero.
⃗⃗),
⃗⃗, 𝒃
Case II: You take the cross product of a vector with a vector that is not itself. In such a case, for every pair (𝒂
⃗⃗, 𝒂
there will be a pair (𝒃 ⃗⃗) whose cross product will be the negative of the first pair.
Hence, the total must equal zero.
F. Scalar Multiplication
By the properties of scalar multiplication, the second and third terms are equal to the first.
Example 1.333
Find the value of 𝜆:
P a g e 103 | 168
Get all the files at: https://bit.ly/azizhandouts
Aziz Manva (azizmanva@gmail.com)
G. Triangles
Example 1.334
If 𝒂 ⃗⃗ and 𝒄
⃗⃗, 𝒃 ⃗⃗ are position vectors of the vertices of Δ𝐴𝐵𝐶, then, in terms of the angles of the triangle (AP EAPCET,
22 Sep 2020, Shift-II, Adapted)
⃗⃗ − 𝒄
|(𝒂 ⃗⃗) × (𝒃 ⃗⃗ − 𝒂
⃗⃗)|
=
⃗⃗ − 𝒂
(𝒃 ⃗⃗) ∙ (𝒄
⃗⃗ − 𝒂
⃗⃗)
⃗⃗ and ⃗𝒃⃗ for its sides. From trigonometry, we know that the area of the triangle is:
Consider a triangle with 𝒂
1 1
|𝒂 ⃗⃗| sin 𝜃 = |𝒂
⃗⃗||𝒃 ⃗⃗ × ⃗𝒃⃗|
2 2
Example 1.336
⃗⃗, 𝒚
Vectors 𝒙 ⃗⃗, and 𝒛
⃗⃗ are the sides of a triangle.
⃗⃗ and 𝒚
A. Determine the magnitude of the cross product of 𝒙 ⃗⃗ if the cross product of 𝒚⃗⃗ and 𝒛
⃗⃗ is 7.
⃗⃗ and 𝒚
B. Determine the magnitude of the cross product of 𝒙 ⃗⃗ if the area of the triangle formed by the
vectors is 7.
P a g e 104 | 168
Get all the files at: https://bit.ly/azizhandouts
Aziz Manva (azizmanva@gmail.com)
Part A
2 × 𝐴𝑟𝑒𝑎 𝑜𝑓 𝑇𝑟𝑖𝑎𝑛𝑔𝑙𝑒 = |𝒙 ⃗⃗| = |𝒚
⃗⃗ × 𝒚 ⃗⃗ × 𝒛
⃗⃗| = 7
Part B
|𝒙
⃗⃗ × 𝒚
⃗⃗| = 2 × 𝐴𝑟𝑒𝑎 𝑜𝑓 𝑇𝑟𝑖𝑎𝑛𝑔𝑙𝑒 = 2 × 7 = 14
𝑂𝑝𝑡𝑖𝑜𝑛 𝐴 𝑖𝑠 𝑐𝑜𝑟𝑟𝑒𝑐𝑡
Example 1.338
⃗⃗, ⃗𝒃⃗, and 𝒄
Given three vectors 𝒂 ⃗⃗ + ⃗𝒃⃗ + 𝒄
⃗⃗, such that 𝒂 ⃗⃗ = 0, show that:
A. 𝒂 ⃗⃗ ⃗⃗
⃗⃗ × 𝒃 = 𝒃 × 𝒄
⃗⃗ = 𝒄 ⃗⃗ × 𝒂
⃗⃗
B. the magnitude of the cross product of any two of them is the same.
Part A
Setup:
The possible pairs are:
(𝒂 ⃗⃗), (𝒂
⃗⃗ × 𝒃 ⃗⃗ × 𝒄 ⃗⃗ × 𝒄
⃗⃗), (𝒃 ⃗⃗ × 𝒂
⃗⃗), (𝒃 ⃗⃗), (𝒄
⃗⃗ × 𝒂
⃗⃗), (𝒄 ⃗⃗)
⃗⃗ × 𝒃
⃗⃗ × 𝒄
Prove (𝒃 ⃗⃗) = 𝒂 ⃗⃗ × ⃗𝒃⃗
⃗⃗ + ⃗𝒃⃗ + 𝒄
Substitute 𝒂 ⃗⃗ = 0 ⇒ 𝒄 ⃗⃗ + ⃗𝒃⃗):
⃗⃗ = −(𝒂
⃗⃗ × 𝒄
𝒃 ⃗⃗ = 𝒃⃗⃗ × [−(𝒂 ⃗⃗)]
⃗⃗ + 𝒃
Move the minus sign outside (scalar multiple property), and distribute the cross product over the addition:
= −𝒃⃗⃗ × [(𝒂
⃗⃗ + ⃗𝒃⃗)] = −(𝒃 ⃗⃗ × 𝒂
⃗⃗ + ⃗𝒃⃗ × ⃗𝒃⃗)
Use the anti-commutative property (first term), and note that the product of a vector with itself is zero (for the
second term):
= −(−𝒂 ⃗⃗ × 𝒃⃗⃗ + 𝟎
⃗⃗) = 𝒂 ⃗⃗
⃗⃗ × 𝒃
Prove (𝒄 ⃗⃗ × 𝒂 ⃗⃗ × ⃗𝒃⃗
⃗⃗) = 𝒂
We can prove this using similar steps as above:
⃗⃗ × 𝒂
𝒄 ⏟ ⃗⃗ + ⃗𝒃⃗)] × 𝒂
⃗⃗ = [−(𝒂 ⏟ ⃗⃗ + ⃗𝒃⃗)] × 𝒂
⃗⃗ = −[(𝒂 ⃗⃗ = −(𝒂 ⃗⃗ + ⃗𝒃⃗ × 𝒂
⏟ ⃗⃗ × 𝒂 ⃗⃗) = −(−𝒂
⏟ ⃗⃗ × 𝒂 ⃗⃗ × ⃗𝒃⃗) = ⏟
⃗⃗ − 𝒂 ⃗⃗ × ⃗𝒃⃗ = 𝒂
0+𝒂 ⃗⃗ × ⃗𝒃⃗
𝑆𝑢𝑏𝑠𝑡𝑖𝑡𝑢𝑡𝑒 𝑆𝑐𝑎𝑙𝑎𝑟 𝑀𝑢𝑙𝑡𝑖𝑝𝑙𝑒 𝐷𝑖𝑠𝑡𝑟𝑖𝑏𝑢𝑡𝑖𝑣𝑒 𝐴𝑛𝑡𝑖 𝐶𝑜𝑚𝑚𝑢𝑡𝑎𝑡𝑖𝑣𝑒 𝐶𝑟𝑜𝑠𝑠 𝑃𝑟𝑜𝑑𝑢𝑐𝑡
𝑃𝑟𝑜𝑝𝑒𝑟𝑡𝑦 𝑃𝑟𝑜𝑝𝑒𝑟𝑡𝑦 𝑃𝑟𝑜𝑝𝑒𝑟𝑡𝑦 𝑜𝑓∥𝑣𝑒𝑐𝑡𝑜𝑟𝑠
Part B
By the anti-commutative property:
⃗𝒃⃗ × 𝒂 ⃗⃗ × ⃗𝒃⃗)
⃗⃗ = −(𝒂
⃗⃗ × 𝒄
𝒂 ⃗⃗ = −(𝒄
⃗⃗ × 𝒂 ⃗⃗ × ⃗𝒃⃗)
⃗⃗) = −(𝒂
⃗⃗ = −(𝒃
⃗⃗ × 𝒃
𝒄 ⃗⃗ × 𝒄
⃗⃗) = −(𝒂
⃗⃗ × 𝒃⃗⃗)
P a g e 105 | 168
Get all the files at: https://bit.ly/azizhandouts
Aziz Manva (azizmanva@gmail.com)
MCMC 1.339:
𝑀𝑎𝑟𝑘 𝑎𝑙𝑙 𝐶𝑜𝑟𝑟𝑒𝑐𝑡 𝑂𝑝𝑡𝑖𝑜𝑛𝑠
If the vectors 𝒂 ⃗⃗
⃗⃗, 𝒃, and 𝒄 ⃗⃗ form the sides 𝐵𝐶, 𝐶𝐴 and 𝐴𝐵 respectively of a Δ𝐴𝐵𝐶, then:
A. 𝒂 ⃗⃗ ⃗
⃗⃗ ∙ 𝒃 = 𝒃 ∙ 𝒄⃗ ⃗⃗ = 𝒄 ⃗⃗ ∙ ⃗𝒃⃗ = 0
B. 𝒂⃗⃗ × ⃗𝒃⃗ = ⃗𝒃⃗ × 𝒄 ⃗⃗ = 𝒄 ⃗⃗ × 𝒂⃗⃗
C. 𝒂 ⃗⃗ ⃗
⃗⃗ ∙ 𝒃 = 𝒃 ∙ 𝒄⃗ ⃗⃗ = 𝒄 ⃗⃗ ∙ 𝒂⃗⃗ = 0
D. 𝒂⃗⃗ × 𝒂 ⃗ ⃗
⃗⃗ + 𝒃 × 𝒃 + 𝒄⃗ ⃗ ⃗⃗ × 𝒄⃗⃗ = 0
E. 𝒂⃗⃗ × 𝒂 ⃗⃗ + 𝒂 ⃗⃗ × 𝒄 ⃗⃗ + 𝒂 ⃗⃗ × 𝒂⃗⃗ = 0 (JEE Main 2002, Adapted)4
Since the vectors form the sides of the triangle, the sum of the vectors is zero.
⃗⃗ + ⃗𝒃⃗ + 𝒄
𝒂 ⃗⃗ × ⃗𝒃⃗ = ⃗𝒃⃗ × 𝒄
⃗⃗ = 0 ⇒ 𝒂 ⃗⃗ = 𝒄
⃗⃗ × 𝒂
⃗⃗ ⇒ 𝑂𝑝𝑡𝑖𝑜𝑛 𝐴
Since a vector is parallel to itself, the cross product of a vector with itself is zero:
⃗⃗ × 𝒂
𝒂 ⃗⃗ + 𝒂
⃗⃗ × 𝒄
⃗⃗ + 𝒂
⃗⃗ × 𝒂
⃗⃗ = 0 + 0 + 0 = 𝟎 ⇒ 𝑂𝑝𝑡𝑖𝑜𝑛 𝐷
Is there any flaw in the proof? Is so, correct and complete it.
4The original question was easier. It was a single correct question (Option D was missing). This is a question directly based
on your knowledge of properties.
P a g e 106 | 168
Get all the files at: https://bit.ly/azizhandouts
Aziz Manva (azizmanva@gmail.com)
1 1 1
𝐴𝑟𝑒𝑎 𝑜𝑓 Δ𝐴 = |𝒂 ⃗⃗| sin(180 − 𝜃 ⃗⃗ ) = |𝒃
⃗⃗||𝒃 ⃗⃗||𝒄
⃗
⃗| sin(180 − 𝜃 ⃗⃗ ) = |𝒄 ⃗⃗| sin(180 − 𝜃𝒄⃗⃗𝒂⃗⃗ )
⃗⃗||𝒂
2 ⃗
𝒂⃗𝒃 2 𝒃𝒄⃗⃗ 2
The proof is correct when the vectors form the sides of a triangle. We need to consider the cases when the
vectors do not form a triangle.
P a g e 107 | 168
Get all the files at: https://bit.ly/azizhandouts
Aziz Manva (azizmanva@gmail.com)
Example 1.343
⃗⃗ × ⃗𝒃⃗ if 𝒂
A. Find 𝒂 ̂ and ⃗𝒃⃗ = 3𝒊̂ + 5𝒋̂ − 2𝒌
⃗⃗ = 2𝒊̂ + 𝒋̂ + 3𝒌 ̂ (CBSE 2019)
B. Find 𝜆 and 𝜇 if (𝒊̂ + 3𝒋̂ + 9𝒌 ̂) × (3𝒊̂ − 𝜆𝒋̂ + 𝜇𝒌̂) = ⃗𝟎⃗. (CBSE 2016)
C. In Part B, you would have found three equations in two variables. You can find the solution to both the
variables from two equations. Is the step where we check in the third equation useful?
Part A
𝒊̂ 𝒋̂ 𝒌̂
Using the definition, we get 𝒂⃗⃗ × ⃗𝒃⃗ = |2 1 3|
3 5 −2
1 3 2 3 2 1 ̂ ̂ = −17𝒊̂ + 13𝒋̂ + 7𝒌
̂
=| | 𝒊̂ − | | 𝒋̂ + | | 𝒌 = (−2 − 15)𝒊̂ − (−4 − 9)𝒋̂ + (10 − 3)𝒌
5 −2 3 −2 3 5
Part B
𝒊̂ 𝒋̂ ̂
𝒌
Expanding the determinant definition of the cross product |1 3 9 | = 𝟎 ⃗⃗:
3 −𝜆 𝜇
̂ = 𝟎𝒊̂ + 0𝒋̂ + 0𝒌
(3𝜇 + 9𝜆)𝒊̂ − (𝜇 − 27)𝒋̂ + (−𝜆 − 9)𝒌 ̂
Comparing coefficients on both sides:
𝜇 27
𝜇 − 27 = 0 ⇒ 𝜇 = 27 ,
⏟ 3𝜇 + 9𝜆 = 0 ⇒ 𝜆 = − = − = −9
⏟ 3 3
𝑪𝒐𝒎𝒑𝒂𝒓𝒆 𝒋̂
𝑪𝒐𝒎𝒑𝒂𝒓𝒆 𝒊̂
Check in the third equation:
̂: −𝜆 − 9 = 0 ⇒ 𝜆 = −9
𝐶𝑜𝑚𝑝𝑎𝑟𝑒 𝒌
Part C
We have two variables and three equations, which means the system is overdetermined. If the values of 𝜇 and 𝜆
that we find from the first two equations do not satisfy the third equation, then
➢ The system of equations is inconsistent
➢ Hence, it has no solutions
➢ Hence, there would be no values of 𝜇 and 𝜆 to be found.
Therefore, it is useful/important to check the third equation.
P a g e 108 | 168
Get all the files at: https://bit.ly/azizhandouts
Aziz Manva (azizmanva@gmail.com)
𝒊̂ 𝒋̂ ̂
𝒌 𝒊̂ 𝒋̂ ̂
𝒌
⃗⃗ = |𝑎1
⃗⃗ × 𝒃
𝒂 𝑎2 𝑎3 | = |𝑎1 𝑎2 0|
𝑏1 𝑏2 𝑏3 𝑏1 𝑏2 0
The determinant is:
̂
= (𝑎2 𝑏3 − 𝑎3 𝑏2 )𝒊̂ − (𝑎1 𝑏3 − 𝑎3 𝑏1 )𝒋̂ + (𝑎1 𝑏2 − 𝑎2 𝑏1 )𝒌
➢ Rearranging the definition of the cross product gives us the property above.
Example 1.347
A. If |𝒂 ⃗⃗| = 7 and 𝒂
⃗⃗| = 2, |𝒃 ⃗⃗ × ⃗𝒃⃗ = 3𝒊̂ + 2𝒋̂ + 6𝒌
̂, find the angle between 𝒂 ⃗⃗ and ⃗𝒃⃗. (CBSE 2019)
B. If 𝜃 is the angle between two vectors 𝒊̂ − 2𝒋̂ + 3𝒌 ̂ and 3𝒊̂ − 2𝒋̂ + 𝒌
̂, find sin 𝜃. (CBSE 2018)
Part A
⃗⃗ × ⃗𝒃⃗| √32 + 22 + 62
|𝒂 √49 7 1 1 𝜋
sin 𝜃 = = = = = ⇒ 𝜃 = sin−1 =
|𝒂 ⃗⃗|
⃗⃗||𝒃 2×7 2×7 2×7 2 2 6
Part B
The cross product of the two vectors is:
𝒊̂ 𝒋̂ 𝒌 ̂
|1 −2 3| = (−2 + 6)𝒊̂ − (1 − 9)𝒋̂ + (−2 + 6)𝒌 ̂ = 4𝒊̂ + 8𝒋̂ + 4𝒌
̂ = 𝟒(𝒊̂ + 2𝒋̂ + 1𝒌
̂)
3 −2 1
Rearranging the definition of the cross product to solve for sin 𝜃:
⃗⃗ × ⃗𝒃⃗|
|𝒂 4√12 + 22 + 12 4√6 4√6 2√6
sin 𝜃 = = = = =
|𝒂 ⃗⃗|
⃗⃗||𝒃 √12 + (−2)2 + 32 √32 + (−2)2 + 12 √14√14 14 7
P a g e 109 | 168
Get all the files at: https://bit.ly/azizhandouts
Aziz Manva (azizmanva@gmail.com)
Example 1.349
⃗⃗ = 𝒊̂ + 2𝒋̂ + 𝒌
If 𝒂 ̂, 𝒃 = 2𝒊̂ + 𝒋̂ and 𝒄 = 3𝒊̂ − 4𝒋̂ − 5𝒌
̂, then find a unit vector perpendicular to both of the vectors
⃗⃗ − ⃗𝒃⃗) and (𝒄
(𝒂 ⃗⃗ − ⃗𝒃⃗) (CBSE 2015)
⃗⃗ − ⃗𝒃⃗ = 𝒊̂ + 2𝒋̂ + 𝒌
𝒂 ̂ − (2𝒊̂ + 𝒋̂) = −𝒊̂ + 𝒋̂ + 𝒌̂
⃗⃗ − ⃗𝒃⃗ = 3𝒊̂ − 4𝒋̂ − 5𝒌
𝒄 ̂ − (2𝒊̂ + 𝒋̂) = 𝒊̂ − 5𝒋̂ − 5𝒌̂
A vector perpendicular to the above two vectors is given by their cross product:
𝒊̂ 𝒋̂ ̂
𝒌
|−1 1 ̂ = −4𝒋̂ + 4𝒌
1 | = (−5 + 5)𝒊̂ − (5 − 1)𝒋̂ + (5 − 1)𝒌 ̂ = 4(−𝒋̂ + 𝒌
̂)
1 −5 −5
Convert the above vector into a unit vector by dividing it by its magnitude:
̂)
4(−𝒋̂ + 𝒌 ̂
−𝒋̂ + 𝒌 √2 √2 ̂
= =− 𝒋̂ + 𝒌
2
4√(−1) + 1 2 √𝟐 2 2
The question asks for only one vector, but we can find one more vector by taking the negative:
√2 √2 ̂
𝒋̂ − 𝒌
2 2
D. Collinearity
Example 1.350
̂ and 𝒃
⃗⃗ = 𝛼𝒊̂ + 3𝒋̂ − 6𝒌
If 𝒂 ⃗⃗ = 2𝒊̂ − 𝒋̂ + 𝛽𝒌
̂, then the values of 𝛼, 𝛽 so that 𝒂 ⃗⃗ may be collinear are: (AP
⃗⃗ and 𝒃
EAPCET, 17 Sep 2020, Shift-I)
P a g e 110 | 168
Get all the files at: https://bit.ly/azizhandouts
Aziz Manva (azizmanva@gmail.com)
E. Dot Product
Example 1.352
A. Let 𝒂⃗⃗ = 4𝒊̂ + 5𝒋̂ − 𝒌 ̂, 𝒃
⃗⃗ = 𝒊̂ − 4𝒋̂ + 5𝒌
̂ and 𝒄 ̂. Find a vector 𝒅
⃗⃗ = 3𝒊̂ + 𝒋̂ − 𝒌 ⃗⃗ which is perpendicular to both
⃗⃗ and 𝒃
𝒄 ⃗⃗ and 𝒅⃗⃗ ∙ 𝒂
⃗⃗ = 21. (CBSE 2019)
B. If 𝒂 ̂, 𝒃
⃗⃗ = 𝒊̂ + 𝒋̂ + 𝒌 ⃗⃗ = 𝒋̂ − 𝒌
̂ then, find a vector 𝒄⃗⃗ such that 𝒂⃗⃗ × 𝒄 ⃗⃗ and 𝒂
⃗⃗ = 𝒃 ⃗⃗ ∙ 𝒄
⃗⃗ = 3. (CBSE 2008, 2013)
Part A
⃗⃗ is perpendicular to both 𝒄
⃗⃗ × 𝒃
𝒄 ⃗⃗. Hence, 𝒅
⃗⃗ and 𝒃 ⃗⃗ must be a scalar multiple of the cross product of 𝒄⃗⃗ and 𝒃⃗⃗.
𝒊̂ 𝒋̂ ̂
𝒌
⃗𝒅⃗ = 𝜆(𝒄
⃗⃗ × ⃗𝒃⃗) = 𝜆 |3 1 −1| = 𝜆[(5 − 4)𝒊̂ − (15 + 1)𝒋̂ + (−12 − 1)𝒌 ̂] = 𝜆(𝒊̂ − 16𝒋̂ − 13𝒌 ̂)
1 −4 5
5 2 2
̂ = 𝒊̂ + 𝒋̂ + 𝒌
⃗⃗ = 𝑥𝒊̂ + 𝑦𝒋̂ + 𝑧𝒌
𝒄 ̂
3 3 3
P a g e 111 | 168
Get all the files at: https://bit.ly/azizhandouts
Aziz Manva (azizmanva@gmail.com)
F. Area of Parallelogram
Example 1.354
̂ and 4𝒋̂ + 2𝒌
Find the area of a parallelogram whose adjacent sides are represented by the vectors 2𝒊̂ − 3𝒌 ̂.
(CBSE 2019)
Example 1.356
̂ and 2𝒊̂ + 2𝒋̂ + 3𝒌
57: The two adjacent sides of a parallelogram are 2𝒊̂ − 4𝒋̂ − 5𝒌 ̂. Find the area of the
parallelogram using its diagonal vectors. (CBSE 2016)
⃗⃗⃗⃗⃗⃗
𝑨𝑪 = 𝑨𝑩 ⃗⃗⃗⃗⃗⃗⃗ + ⃗⃗⃗⃗⃗⃗⃗
𝑨𝑫 = 4𝒊̂ − 2𝒋̂ − 2𝒌 ̂
⃗⃗⃗⃗⃗⃗⃗ = ⃗⃗⃗⃗⃗⃗⃗
𝑩𝑫 𝑨𝑫 − 𝑨𝑩 ̂
⃗⃗⃗⃗⃗⃗⃗ = 6𝒋̂ + 8𝒌
The cross product of the two vectors above is:
𝒊̂ 𝒋̂ ̂
𝒌
⃗⃗⃗⃗⃗⃗ ⃗⃗⃗⃗⃗⃗⃗
𝑨𝑪 × 𝑩𝑫 = |4 −2 −2| = (−16 + 12)𝒊̂ − (32 + 0)𝒋̂ + (24 − 0)𝒌 ̂ = 4(−𝒊̂ − 8𝒋̂ + 6𝒌
̂)
0 6 8
P a g e 112 | 168
Get all the files at: https://bit.ly/azizhandouts
Aziz Manva (azizmanva@gmail.com)
And the area of the parallelogram is half the magnitude of the cross product of the vectors:
1 1
⃗⃗⃗⃗⃗⃗ × 𝑩𝑫
|𝑨𝑪 ⃗⃗⃗⃗⃗⃗⃗| = × 4√(−1)2 + (−8)2 + 62 = 2√101 𝑢𝑛𝑖𝑡𝑠 2
2 2
G. Area of Triangle
Example 1.358
1
Two lines with slopes 2 and 2 intersect at (2,2). What is the area of the triangle enclosed by these two lines and
the line 𝑥 + 𝑦 = 10? (AMC 10A 2019/7)
H. Torque
1.359: Torque
P a g e 113 | 168
Get all the files at: https://bit.ly/azizhandouts
Aziz Manva (azizmanva@gmail.com)
Example 1.360
Example 1.361
44: Let 𝒂 = 𝒊̂ + 5𝒋̂ + 𝛼𝒌̂, 𝒃 = 𝒊̂ + 3𝒋̂ + 𝛽𝒌
̂ and 𝒄 = −𝒊̂ + 2𝒋̂ − 3𝒌
̂ be three vectors such that |𝒃
⃗⃗ × 𝒄
⃗⃗| = 5√3 and 𝒂
⃗⃗
⃗⃗
is perpendicular to 𝒃. Then, the greatest among the values of |𝒂 2
⃗⃗| is (JEE 2021, 27 Aug, Shift-I)
𝒊̂ 𝒋̂ ̂
𝒌
⃗𝒃⃗ × 𝒄
⃗⃗ = | 1 3 ̂ (2 + 3) = 𝒊̂(−1)(9 + 2𝛽) + 𝒋̂(3 − 𝛽) + 5𝒌
𝛽 | = 𝒊̂(−9 − 2𝛽) − 𝒋̂(−3 + 𝛽) + 𝒌 ̂
−1 2 −3
⃗⃗ × 𝒄
|𝒃 ⃗⃗| = 5√3
√(9 + 2𝛽) + (3 − 𝛽)2 + 52 = 5√3
2
Example 1.362
⃗⃗ = 2𝒊̂ + 3𝒋̂ + 𝒌
46: Let 𝒑 ̂ and 𝒒
⃗⃗ = 𝒊̂ + 2𝒋̂ + 𝒌̂ be two vectors. If a vector 𝒓 ̂ is perpendicular to each
⃗⃗ = 𝛼𝒊̂ + 𝛽𝒋̂ + 𝛾𝒌
of the vectors (𝒑⃗⃗ + 𝒒
⃗⃗) and (𝒑
⃗⃗ − 𝒒
⃗⃗) and |𝒓⃗⃗| = √3, then |𝛼| + |𝛽| + |𝛾| is equal to: (JEE Main 2021, 25 July, Shift-I)
⃗⃗ ∙ (𝒑
𝒓 ⃗⃗ + 𝒒
⃗⃗) = 0
(𝛼, 𝛽, 𝛾) ∙ (2 + 1,3 + 2,1 + 1) = 0
(𝛼, 𝛽, 𝛾) ∙ (3,5,2) = 0
3𝛼 + 5𝛽 + 2𝛾 = 0
⏟
𝑬𝒒𝒖𝒂𝒕𝒊𝒐𝒏 𝑰
⃗⃗ ∙ (𝒑
𝒓 ⃗⃗ − 𝒒
⃗⃗) = 0
(𝛼, 𝛽, 𝛾) ∙ (2 − 1,3 − 2,1 − 1) = 0
(𝛼, 𝛽, 𝛾) ∙ (1,1,0) = 0
𝛼+𝛽 =0
P a g e 114 | 168
Get all the files at: https://bit.ly/azizhandouts
Aziz Manva (azizmanva@gmail.com)
𝛼 = −𝛽
⏟
𝑬𝒒𝒖𝒂𝒕𝒊𝒐𝒏 𝑰𝑰
3(−𝛽) + 5𝛽 + 2𝛾 = 0
2𝛽 + 2𝛾 = 0
𝛾 = −𝛽
|𝒓
⃗⃗| = √3
√𝛼 2 + 𝛽 2 + 𝛾 2 = √3
𝛼 2 + 𝛽2 + 𝛾 2 = 3
(−𝛽)2 + 𝛽 2 + (−𝛽)2 = 3
3𝛽 2 = 3
𝛽2 = 1
𝛼 2 = 𝛽2 = 𝛾 2 = 1
|𝛼| + |𝛽| + |𝛾| = 1 + 1 + 1 = 3
Example 1.363
⃗⃗| = 5 and |𝒂
⃗⃗| = 2,|𝒃
47: If |𝒂 ⃗⃗| = 8, then |𝒂
⃗⃗ × 𝒃 ⃗⃗| is equal to: (JEE Main 2021, 25 July, Shift-II)
⃗⃗ ∙ 𝒃
⃗⃗ × ⃗𝒃⃗| = 8
|𝒂
|𝒂 ⃗⃗| sin 𝜃 = ±8
⃗⃗||𝒃
(2)(5) sin 𝜃 = ±8
8 4
sin 𝜃 = ± =±
10 5
3
cos 𝜃 = ±
5
3
⃗⃗ ∙ ⃗𝒃⃗ = |𝒂
𝒂 ⃗⃗| cos 𝜃 = (2)(5) (± ) = ±6
⃗⃗||𝒃
5
⃗⃗ ∙ ⃗𝒃⃗| = 6
|𝒂
Example 1.364
⃗⃗ and ⃗𝒃⃗ be two non-zero vectors perpendicular to each other, and |𝒂
48: Let 𝒂 ⃗⃗|. If |𝒂
⃗⃗| = |𝒃 ⃗⃗ × ⃗𝒃⃗| = |𝒂
⃗⃗|, then the
angle between (𝒂 ⃗⃗
⃗⃗ + 𝒃 + 𝒂 ⃗⃗
⃗⃗ × 𝒃) and 𝒂
⃗⃗ is equal to: (JEE Main 2021, 18 March, Shift-II)
|𝒂⃗⃗ × ⃗𝒃⃗| = |𝒂
⃗⃗|
|𝒂 ⃗⃗
⃗⃗||𝒃| sin 𝜃 = |𝒂 ⃗⃗|
⃗⃗
|𝒃| sin 90° = 1
⃗⃗| = 1
|𝒃
|𝒂
⃗⃗| = |𝒃⃗⃗| = 1
P a g e 115 | 168
Get all the files at: https://bit.ly/azizhandouts
Aziz Manva (azizmanva@gmail.com)
⃗⃗ = 𝒂
𝒙 ⃗⃗ + 𝒂
⃗⃗ + 𝒃 ⃗⃗ = 𝒊̂ + 𝒋̂ + 𝒊̂ × 𝒋̂ = 𝒊̂ + 𝒋̂ + 𝒌
⃗⃗ × 𝒃 ̂
⃗⃗| = √12 + 12 + 12 = √3
|𝒙
𝒙 ⃗⃗ = |𝒙
⃗⃗ ∙ 𝒂 ⃗⃗||𝒂
⃗⃗| cos 𝜃 = √3 ∙ 1 ∙ cos 𝜃 = √3 cos 𝜃
̂) ∙ (𝒊̂) = √3 cos 𝜃
(𝒊̂ + 𝒋̂ + 𝒌
1 = √3 cos 𝜃
1
cos 𝜃 =
√3
1
𝜃 = cos −1 ( )
√3
Example 1.365
Let 𝒂 ̂ and ⃗𝒃⃗ = 7𝒊̂ + 𝒋̂ − 6𝒌
⃗⃗ = 2𝒊̂ − 3𝒋̂ + 4𝒌 ̂. If 𝒓
⃗⃗ × 𝒂 ⃗⃗ × ⃗𝒃⃗, 𝒓 ∙ (𝒊̂ + 2𝒋̂ + 𝒌
⃗⃗ = 𝒓 ̂) = −3, then 𝒓 ̂ ) is equal
⃗⃗ ∙ (2𝒊̂ − 3𝒋̂ + 𝒌
to: (JEE Main 2021, 17 March, Shift-I)
⃗⃗ × 𝒂
𝒓 ⃗⃗ × ⃗𝒃⃗
⃗⃗ = 𝒓
Collate all terms on the LHS:
⃗⃗ × 𝒂
𝒓 ⃗⃗ × ⃗𝒃⃗ = 0
⃗⃗ − 𝒓
Use the distributive property
⃗⃗ − ⃗𝒃⃗) = 0
⃗⃗ × (𝒂
𝒓
The cross product of two vectors is zero when they are parallel. Hence:
⃗⃗ − ⃗𝒃⃗) = 𝜆(−5, −4,10)
⃗⃗ = 𝜆(𝒂
𝒓
𝒓 ∙ (1,2,1) = −3
𝜆(−5, −4,10) ∙ (1,2,1) = −3
𝜆(−5 − 8 + 10) = −3
𝜆(−3) = −3
𝜆=1
⃗⃗ = (−5, −4,10)
𝒓
Example 1.366
̂ and ⃗𝒃⃗ = 2𝒊̂ − 3𝒋̂ + 5𝒌
⃗⃗ = 𝒊̂ + 2𝒋̂ − 3𝒌
Let 𝒂 ̂. If 𝒓 ⃗⃗ = ⃗𝒃⃗ × 𝒓
⃗⃗ × 𝒂 ̂) = 3 and 𝒓
⃗⃗, 𝒓 ∙ (𝛼𝒊̂ + 2𝒋̂ + 𝒌 ̂) = −1,
⃗⃗ ∙ (2𝒊̂ + 5𝒋̂ − 𝛼𝒌
2
𝛼 ∈ ℝ, then the value of 𝛼 + |𝒓 ⃗⃗| is equal to: (JEE Main 2021, 16 March, Shift-II)
𝒓 ⃗⃗ = ⃗𝒃⃗ × 𝒓
⃗⃗ × 𝒂 ⃗⃗
Use the anti-commutative property
⃗⃗ × 𝒂
𝒓 ⃗⃗ = −(𝒓 ⃗⃗ × 𝒃⃗⃗)
⃗⃗ × 𝒂
𝒓 ⃗⃗ × ⃗𝒃⃗ = 0
⃗⃗ + 𝒓
Use the distributive property:
⃗⃗ × (𝒂
𝒓 ⃗⃗) = 0
⃗⃗ + 𝒃
P a g e 116 | 168
Get all the files at: https://bit.ly/azizhandouts
Aziz Manva (azizmanva@gmail.com)
̂) = 3
𝒓 ∙ (𝛼𝒊̂ + 2𝒋̂ + 𝒌
𝜆(3, −1,2) ∙ (𝛼, 2,1) = 3
𝜆(3𝛼 − 2 + 2) = 3
𝛼𝜆 = 1
⃗⃗|𝟐 = 32 + 12 + 22 = 9 + 1 + 4 = 14
|𝒓
⃗⃗|2 = 1 + 14 = 15
𝛼 + |𝒓
Example 1.367
List the eight possible triple products
Explain which ones which are defined and which ones are not
𝑎 𝑎3 𝑎1 𝑎3 𝑎1 𝑎2
̂ ) ∙ (| 2
(𝑐1 𝒊̂ + 𝑐2 𝒋̂ + 𝑐3 𝒌 ̂
𝑏2 𝑏3 | 𝒊̂ − |𝑏1 𝑏3 | 𝒋̂ + |𝑏1 𝑏2 | 𝒌)
𝑎2 𝑎3 𝑎1 𝑎3 𝑎1 𝑎2
= |𝑏 𝑏 | 𝑐1 − |𝑏 𝑏 | 𝑐2 + |𝑏 𝑏 | 𝑐3
2 3 1 3 1 2
P a g e 117 | 168
Get all the files at: https://bit.ly/azizhandouts
Aziz Manva (azizmanva@gmail.com)
𝑐1 𝑐2 𝑐3
𝑎
=| 1 𝑎2 𝑎3 |
𝑏1 𝑏2 𝑏3
C. Vector Triple Product
P a g e 118 | 168
Get all the files at: https://bit.ly/azizhandouts
Aziz Manva (azizmanva@gmail.com)
⃗⃗.
Consider an arbitrary point 𝑄 on the line with position vector 𝒓
⃗⃗
Since 𝒃 is parallel to the line:
𝑷𝑸 ∥ ⃗𝒃⃗
⃗⃗⃗⃗⃗⃗⃗
The two vectors above have the same direction. Hence, one must be a
scalar multiple of the other:
⃗⃗⃗⃗⃗⃗⃗
𝑷𝑸 = 𝜆𝒃 ⃗⃗
Write ⃗⃗⃗⃗⃗⃗⃗
𝑷𝑸 as a displacement vector:
⃗⃗ − 𝒂
𝒓 ⃗⃗ = 𝜆𝒃 ⃗⃗
Solve for 𝒓 ⃗⃗:
⃗⃗ = 𝒂
𝒓 ⃗⃗ + 𝜆𝒃⃗⃗
➢ As we will see later, on the applications of the vector equation is to take time as a parameter. For
example, in moving objects (or Kinematics).
Example 2.3
A. Find the vector equation of a line which passes through the point (3,4,5) and is parallel to the vector
̂. (CBSE 2019)
2𝒊̂ + 2𝒋̂ − 3𝒌
B. Write the equation of the straight line through the point (𝛼, 𝛽, 𝛾) and parallel to the 𝑧 −axis. (CBSE
2014)
Part A
Substitute 𝒂 ⃗⃗ = (2,2, −3) in 𝒓
⃗⃗ = (3,4,5), 𝒃 ⃗⃗ = 𝒂
⃗⃗ + 𝜆𝒃 ⃗⃗:
⃗⃗ = (3,4,5) + 𝜆(2,2, −3)
𝒓
Part B
Substitute 𝒂 ⃗⃗ = (0,0,1) in 𝒓
⃗⃗ = (𝛼, 𝛽, 𝛾), 𝒃 ⃗⃗ = 𝒂⃗⃗ + 𝜆𝒃⃗⃗:
⃗⃗ = (𝛼, 𝛽, 𝛾) + 𝜆(0,0,1)
𝒓
P a g e 119 | 168
Get all the files at: https://bit.ly/azizhandouts
Aziz Manva (azizmanva@gmail.com)
Example 2.5
The points 𝐴(4,5,10), 𝐵(2,3,4) and 𝐶(1,2, −1) are three vertices of parallelogram 𝐴𝐵𝐶𝐷. Find the vector
equations of sides 𝐴𝐵 and 𝐵𝐶 and find coordinates of point 𝐷. (CBSE 2010)
Example 2.7
Show from the previous example that
⃗⃗⃗⃗⃗⃗⃗ = (4,5,10) + 𝜆1 (1,1,3)
𝑨𝑩
⃗⃗⃗⃗⃗⃗⃗ = (2,3,4) + 𝜆1 (1,1,5)
𝑩𝑪
Example 2.8
Find the equation of:
A. the 𝑧 axis.
B. the 𝑦 axis.
C. the 𝑥 axis.
D. a line parallel to the z axis, and passing through the point (1,2,3).
Part A
The 𝑧 axis includes the origin. Hence, (0,0,0) is a point on the line.
The unit vector 𝒌̂ is parallel to the z axis.
Hence, the equation of the 𝑧 axis is:
⃗⃗ = (0,0,0) + 𝜆(0,0,1) = 𝜆𝒌
𝒓 ̂
Parts B and C
The equations of the other axes are:
⃗⃗ = (0,0,0) + 𝜆(0,1,0) = 𝜆𝒋̂
𝒓
⃗⃗ = (0,0,0) + 𝜆(1,0,0) = 𝜆𝒊̂
𝒓
Part B
(1,2,0) is a point on the line. The unit vector 𝒌 ̂ is parallel to the z axis. The equation of the line is:
⃗⃗ = (1,2,0) + 𝜆(0,0,1) = 𝜆𝒌
𝒓 ̂
In the above example, the equations that have been found are:
⃗⃗ = 𝜆𝒌
𝒓 ̂, ⃗⃗ = 𝜆𝒋̂,
𝒓 ⃗⃗ = 𝜆𝒊̂
𝒓
Is it a good idea to use the same variable (𝒓⃗⃗) for all three equations, or should we have written something like
⃗⃗ = 𝜆𝒌
𝒂 ̂, ⃗𝒃⃗ = 𝜆𝒋̂, ⃗⃗ = 𝜆𝒊̂
𝒄
When we write equations in 2D in Cartesian form, we do not use different variables. For example, two lines
P a g e 120 | 168
Get all the files at: https://bit.ly/azizhandouts
Aziz Manva (azizmanva@gmail.com)
could be:
𝑦 = 2𝑥 + 3
1
𝑦 = 𝑥+4
2
And both of the equations use the variables 𝑥 and 𝑦. If we wanted to find the intersection of these lines, we
would note that the LHS of each equation is the same, and hence the RHS must also be the same.
1
2𝑥 + 3 = 𝑥 + 4
2
Find the point of intersection of the axes by solving the system of equations:
𝒓 ̂,
⃗⃗ = 𝜆𝒌 ⃗⃗ = 𝜆𝒋̂,
𝒓 ⃗⃗ = 𝜆𝒊̂
𝒓
We know that the point of intersection is the origin (0,0,0). We prove it by solving the system of equations
given.
Note that the LHS of each equation is the same, and hence the RHS must also be the same.
̂ = 𝜆𝒋̂ = 𝜆𝒊̂
𝜆𝒌
Hence, the only possible value of 𝜆 is a multiple that has no direction, which means:
𝜆 = 0 ⇒ 𝑃𝑜𝑖𝑛𝑡 𝑜𝑓 𝐼𝑛𝑡𝑒𝑟𝑠𝑒𝑐𝑡𝑖𝑜𝑛 = (0,0,0)
Carry out the scalar multiplication and add the vectors on the RHS of the given equation:
⃗⃗ = (2 + 𝜆, 3 + 𝜆, 4 + 5𝜆)
𝒓
The point that we want has coordinates (7, 𝑦, 𝑧). Equate the x coordinates:
2+𝜆 =7⇒𝜆 =5
Substitute the value of the parameter 𝜆 = 5 to find the value of the y and the z coordinates:
𝑦 =3+𝜆 =3+5=8
𝑧 = 4 + 5𝜆 = 4 + 5(5) = 29
Part A
Carry out the scalar multiplication and add the vectors on the RHS of the given equation:
⃗⃗ = (2𝜆, 2 + 2𝜆, 3 − 5𝜆)
𝒓
The point that we are investigating is (1,2,3). Equate the 𝑥, 𝑦 and 𝑧 coordinates:
1
2𝜆 = 1 ⇒ 𝜆 =
2
2 + 2𝜆 = 2 ⇒ 𝜆 = 0
3 − 5𝜆 = 3 ⇒ 𝜆 = 0
P a g e 121 | 168
Get all the files at: https://bit.ly/azizhandouts
Aziz Manva (azizmanva@gmail.com)
Since the value of the parameter is not the same from all three equations, the point does not lie on the line.
Part B
From the y and z coordinates, we get 𝜆 = 0. Find the value of the x coordinate, when 𝜆 = 0:
𝑥 = 2𝜆 = 2(0) = 0
The point that we want is:
(0,2,3)
B. Direction Ratios
The direction ratios, as the name says, indicate the ratio of the change in different coordinates:
𝑎: 𝑏: 𝑐 = Δ𝑥: Δ𝑦: Δz = (𝑥2 − 𝑥1 ): (𝑦2 − 𝑦1 ): (𝑧2 − 𝑧1 )
Example 2.12
Compare direction ratios with slope for a 2D line.
Δ𝑦
𝑆𝑙𝑜𝑝𝑒 = = Δ𝑦: Δ𝑥
Δ𝑥
Slope tells you the rate of change in the 𝑦 coordinate with respect to rate of change
of the 𝑥 coordinate.
Direction ratios are like slope, except that they compare 𝑥, 𝑦 and 𝑧 coordinates:
Δ𝑥: Δ𝑦: Δz
Example 2.13
Find the direction ratios of the line passing through the points (1,2,3) and
(−3, −2, −1).
Example 2.14
⃗⃗ = (4,5,10) + 𝜆(1,2,5)
𝒓
A. What are the direction ratios of the line above.
B. Interpret these direction ratios in terms of change in 𝑥 coordinate, change in 𝑦 coordinate, and change
in 𝑧 coordinate.
C. If the 𝑥 coordinate increases by 10, then find the change in the y and the z coordinates.
D. Suppose the point 𝑃 = (𝑥, 𝑦, 𝑧) lies on the line above. Find the point with 𝑥 coordinate 2 units greater
than 𝑃 that lies on the line.
Part A
The direction ratios are:
1: 2: 5
Part B
P a g e 122 | 168
Get all the files at: https://bit.ly/azizhandouts
Aziz Manva (azizmanva@gmail.com)
Δ𝑥: Δ𝑦: Δz = 1: 2: 5
Part C
Δ𝑥: Δ𝑦: Δz = 1: 2: 5 = 10: 20: 50
Δ𝑥 = 𝐶ℎ𝑎𝑛𝑔𝑒 𝑖𝑛 𝑥 𝑐𝑜𝑜𝑟𝑑𝑖𝑛𝑎𝑡𝑒 = 10
Δy = 𝐶ℎ𝑎𝑛𝑔𝑒 𝑖𝑛 𝑦 𝑐𝑜𝑜𝑟𝑑𝑖𝑛𝑎𝑡𝑒 = 20
Δz = 𝐶ℎ𝑎𝑛𝑔𝑒 𝑖𝑛 𝑧 𝑐𝑜𝑜𝑟𝑑𝑖𝑛𝑎𝑡𝑒 = 50
Part D
Δ𝑥: Δ𝑦: Δz = 1: 2: 5 = 2: 4: 10
The point that we require is:
𝑄 = (𝑥 + 2, 𝑦 + 4, 𝑧 + 10)
C. Direction Cosines
Angle with positive
direction of
2.15: Direction Angles
𝑥 − 𝑎𝑥𝑖𝑠 𝛼
If 𝑂 is the origin, then the angles made by ⃗⃗⃗⃗⃗⃗⃗
𝑶𝑷 with the positive 𝑦 − 𝑎𝑥𝑖𝑠 𝛽
direction of 𝑥-axis, 𝑦-axis and 𝑧-axis are the direction angles. 𝑧 − 𝑎𝑥𝑖𝑠 𝛾
Example 2.17
A. If a line makes angles 90°,135°, 45° with the 𝑥, 𝑦 and 𝑧 axes respectively, find its direction cosines (CBSE
2019)
B. Write the direction cosines as a triplet.
Part A
𝑙 = cos 90° = 0
1
𝑚 = cos 135° = −
√2
1
𝑛 = cos 45° =
√2
Part B
The direction cosines for the line are:
1 1
(𝑙, 𝑚, 𝑛) = (0, − , )
√2 √2
Example 2.18
P a g e 123 | 168
Get all the files at: https://bit.ly/azizhandouts
Aziz Manva (azizmanva@gmail.com)
Part A
𝑥 𝑦 𝑧
𝑙: 𝑚: 𝑛 = cos 𝛼 : cos 𝛽 : cos 𝛾 = : : = 𝑥: 𝑦: 𝑧
|𝒓
⃗⃗| |𝒓
⃗⃗| |𝒓
⃗⃗|
Part B
This ratio gives the change in 𝑥, 𝑦 and 𝑧 for change in the magnitude of the associated vector. In other words,
this is a direction ratio.
Example 2.19
Find direction cosines for the axes.
𝑥 2 𝑦 2 𝑧 2 𝑥2 + 𝑦2 + 𝑧2 𝑥2 + 𝑦2 + 𝑧2
𝑙 2 + 𝑚2 + 𝑛2 = ( ) +( ) +( ) = = 2 =1
|𝒓
⃗⃗| |𝒓
⃗⃗| |𝒓
⃗⃗| ⃗⃗|2
|𝒓 𝑥 + 𝑦2 + 𝑧2
Example 2.21
A. What are the direction cosines of a line that makes equal angles with the coordinate axes? (CBSE
2008,2009,2011,2019)
B. If a line makes angles 90° and 60° respectively with the positive directions of X and 𝑌 −axes, find the
angle which it makes with the positive direction of 𝑍-axis. (CBSE 2017)
C. If a line makes angles 90°, 60° and 𝜃 with 𝑋, 𝑌 and 𝑍 axes respectively, where 𝜃 is an acute angle, then
find 𝜃. (CBSE 2015)
D. If a line makes angles 𝛼, 𝛽 and 𝛾 with the positive direction of coordinate axes, then write the value of
sin2 𝛼 + sin2 𝛽 + sin2 𝛾. (CBSE 2015C)
Part A 3
2 2 2 cos2 𝛾 =
𝑙 +𝑚 +𝑛 =1 4
But the angles are equal, which means their cosines √3
cos 𝛾 = ±
are equal: 2
𝑙=𝑚=𝑛 𝛾 = {30°, 150°}
Hence: Part C
1 Same as Part B, above except that we reject the
3𝑙 2 = 1 ⇒ 𝑙 = ± angle in the second quadrant.
√3
The direction cosines are: 𝜃 = 30°
1 1 1 1 1 1 Part D
{ , , } 𝑂𝑅 {− ,− ,− } sin2 𝛼 + sin2 𝛽 + sin2 𝛾
√3 √3 √3 √3 √3 √3
= (1 − cos 2 𝛼) + (1 − cos 2 𝛽) + (1 − cos2 𝛾)
Part B = 3 − cos 2 𝛼 + cos 2 𝛽 + cos2 𝛾
cos 2 90° + cos2 60° + cos2 𝛾 = 1 =3−1
1 2 =2
02 + ( ) + cos 2 𝛾 = 1
2
P a g e 124 | 168
Example 2.22
𝜋 𝜋
̂, then the possible values of 𝜃 are: (JEE
̂ makes angles with 𝒊̂, with 𝒋̂ and 𝜃 ∈ {0, 𝜋} with 𝒌
If a unit vector 𝒂 3 4
Main 2019, 9 April, Shift-II)
𝑙 2 + 𝑚2 + 𝑛2 = 1
𝜋 𝜋
cos 2 ( ) + cos2 ( ) + cos 2(𝜃) = 1
3 4
1 1
+ + cos 2 (𝜃) = 1
4 2
3 1
cos2 (𝜃) = 1 − =
4 4
1
cos 𝜃 = ±
2
𝜋 2𝜋
𝜃={ , }
3 3
Δ𝑥 2 Δ𝑦 2 Δz 2
( ) + ( ) + ( ) = 1, 𝑋 = √(Δ𝑥)2 + (Δ𝑦)2 + (Δz)2
𝑋 𝑋 𝑋
Δ𝑥 Δ𝑦 Δ𝑧
, , , 𝑋 = √(Δ𝑥)2 + (Δ𝑦)2 + (Δz)2
𝑋 𝑋 𝑋
Example 2.24
Determine the direction cosines of the line
⃗⃗ = (−2,3,7) + 𝜆(1,2,5)
A. 𝒓
⃗⃗ = (−2,3,7) + 𝜆(3,4,5)
B. 𝒓
Part A
𝐷𝑖𝑟𝑒𝑐𝑡𝑖𝑜𝑛 𝑅𝑎𝑡𝑖𝑜𝑠 = Δ𝑥: Δ𝑦: Δz = 1: 2: 5
12 + 22 + 52 = 1 + 4 + 25 = 30
Divide both sides by 30:
12 + 22 + 52 30
= =1
30 30
12 22 52
+ + =1
30 30 30
1 2 2 2 5 2
( ) +( ) +( ) =1
√30 √30 √30
Part B
𝑋 = √32 + 42 + 52 = √50
3 4 5
𝐷𝑖𝑟𝑒𝑐𝑡𝑖𝑜𝑛 𝐶𝑜𝑠𝑖𝑛𝑒𝑠 = ( , , )
√50 √50 √50
Get all the files at: https://bit.ly/azizhandouts
Aziz Manva (azizmanva@gmail.com)
MCQ 2.25
𝑀𝑎𝑟𝑘 𝑡ℎ𝑒 𝑐𝑜𝑟𝑟𝑒𝑐𝑡 𝑜𝑝𝑡𝑖𝑜𝑛
Consider the vector equation of a line. Changing the point 𝒂 ⃗⃗ to another point on the same line will change the
direction cosines.
⃗⃗ = 𝒂
A. If the vector equation is in point vector form 𝒓 ⃗⃗ + 𝜆𝒃 ⃗⃗
B. If the vector equation is in two point vector form 𝒓⃗⃗ = 𝒂 ⃗⃗ − 𝒂
⃗⃗ + 𝜆(𝒃 ⃗⃗)
C. Both of the above
D. None of the above
Changing to another point on the same line keeps the parallel vector the same, and hence the direction ratios
(and directions cosines) also the same.
𝑂𝑝𝑡𝑖𝑜𝑛 𝐷
Example 2.26
𝑀𝑎𝑟𝑘 𝑎𝑙𝑙 𝑐𝑜𝑟𝑟𝑒𝑐𝑡 𝑜𝑝𝑡𝑖𝑜𝑛𝑠
Two lines are parallel or the same line, if:
A. Their direction ratios are the same.
B. Their direction cosines are the same.
C. The direction ratios of one line are 𝑥: 𝑦: 𝑧 and the direction ratios of the second line are 𝑎𝑥: 𝑎𝑦: 𝑎𝑧.
D. The direction cosines of one line are a multiple of the direction cosines of the other line.
If two lines have the same direction cosines, they are going in the same direction, and hence they are parallel.
𝑂𝑝𝑡𝑖𝑜𝑛 𝐵
If two lines have equal direction ratios, then they point in the same direction. Again, the lines are parallel.
𝑂𝑝𝑡𝑖𝑜𝑛 𝐴
Simplify the direction ratios of both lines, and then the direction ratios will be equal. Again, they point in the
same direction, and they are parallel.
𝑂𝑝𝑡𝑖𝑜𝑛 𝐶
Example 2.29
A. (𝐶𝑎𝑟𝑡𝑒𝑠𝑖𝑎𝑛 𝑓𝑜𝑟𝑚) A line passes through the point with position vector 2𝒊̂ − 𝒋̂ + 4𝒌 ̂ and is in the
̂. Find the equation of the line in Cartesian form. (CBSE 2019)
direction of the vector 𝒊̂ + 𝒋̂ − 2𝒌
𝑥−5 𝑦+4 𝑧−6
B. (𝐶𝑎𝑟𝑡𝑒𝑠𝑖𝑎𝑛 𝑡𝑜 𝑉𝑒𝑐𝑡𝑜𝑟 𝑓𝑜𝑟𝑚) 17: Write the vector equation for the line = = . (CBSE 2011)
3 7 2
C. (𝐶𝑜𝑛𝑣𝑒𝑟𝑡 𝑓𝑟𝑜𝑚 𝑉𝑒𝑐𝑡𝑜𝑟 𝑡𝑜 𝐶𝑎𝑟𝑡𝑒𝑠𝑖𝑎𝑛 𝑓𝑜𝑟𝑚)
Part A
Substitute (𝑥1 , 𝑦1 , 𝑧1 ) = (2, −1,4), (𝑎, 𝑏, 𝑐) = (1,1 − 2):
𝑥−2 𝑦+1 𝑧−4
= =
1 1 −2
Part B
Substitute (𝑥1 , 𝑦1 , 𝑧1 ) = (5, −4,6), (𝑎, 𝑏, 𝑐) = (3,7,2):
𝑥 5 3
⃗⃗ = 𝒂
𝒓 ⃗⃗
⃗⃗ + 𝜆𝒃 ⇒ (𝑦) = (−4) + 𝜆 (7)
𝑧 6 2
Example 2.31
A student attempted the question “Write the equation 3𝑥 = 4𝑦 in such a way that the coefficient of both 𝑥 and 𝑦
is 1”,:
3𝑥 4𝑦
= ⇒𝑥=𝑦
3 4
Is this correct? If it is not correct, explain why and write the corrected version.
Corrected Version
Divide both sides by 𝐿𝐶𝑀(3,4) = 12
3𝑥 4𝑦 𝑥 𝑦
= ⇒ =
12 12 4 3
Example 2.32
P a g e 127 | 168
Get all the files at: https://bit.ly/azizhandouts
Aziz Manva (azizmanva@gmail.com)
A student attempted the question “Write the equation 3𝑥 = 4𝑦 = 5𝑧 in such a way that the coefficient of 𝑥, 𝑦
and 𝑧 is 1”:
3𝑥 4𝑦 5𝑧
= = ⇒𝑥=𝑦=𝑧
3 4 5
Is this correct? If it is not correct, explain why and write the corrected version.
Example 2.33
A student attempted the question “Write the equation 3𝑥 = 4𝑦 in such a way that the coefficient of both 𝑥 and 𝑦
is 1”,:
3𝑥 4𝑦
= ⇒𝑥=𝑦
3 4
Is this correct? If it is not correct, explain why and write the corrected version.
Example 2.34
Identify the direction ratios and a point on each line below:
𝑥−3 𝑦+5 2𝑧−1
A. 2
= −4 = 6
3𝑥−𝑥1 𝑦 −2𝑦 𝑧 −𝑧
B. = 1 = 1
𝑎 𝑏 𝑐
Part A
1 1
𝑥 − 3 𝑦 + 5 2 (2𝑧 − 1) 𝑥 − 3 𝑦 + 5 𝑧 − 2 1
= = ⇒ = = ⇒ (2,
⏟ −4,3) , (3, −5, )
2 −4 1 2 −4 3 ⏟ 2
2×6 Direction Ratios
Point on the Line
Part B
1 1 𝑥 𝑦
(3𝑥 − 𝑥1 ) (− ) (𝑦1 − 2𝑦) −(𝑧1 − 𝑧) 𝑥 − 1 𝑦 − 21 𝑧 − 𝑧1
3 = 2 = = 3 = =
1 1 −𝑐 1 1 −𝑐
×𝑎 − ×𝑏 ×𝑎 − ×𝑏
3 2 3 2
𝑎 𝑏 𝑥1 𝑦1
( , − , −𝑐) , ( , , 𝑧1 )
⏟3 2 ⏟3 2
𝐷𝑖𝑟𝑒𝑐𝑡𝑖𝑜𝑛 𝑅𝑎𝑡𝑖𝑜𝑠 𝑃𝑜𝑖𝑛𝑡 𝑜𝑛 𝑡ℎ𝑒 𝐿𝑖𝑛𝑒
Example 2.35
4−𝑥 𝑦 1−𝑧
= =
2 6 3
A. Identify the direction ratios in the above line.
B. Use the direction ratios to find a vector parallel to the line.
C. Use the direction ratios to find all vectors parallel to the line.
P a g e 128 | 168
Get all the files at: https://bit.ly/azizhandouts
Aziz Manva (azizmanva@gmail.com)
Part A 10 5 10 5 15
(3, , ) ⇒ 𝑦1 + 𝑧1 = + = =5
3 3 3 3 3
𝑥−2 𝑦−3 𝑧−1
Let = = = 𝜆, and solve for 𝑥, 𝑦 and 𝑧:
3 1 2
Part B
𝑥−2
= 𝜆 ⇒ 𝑥 = 3𝜆 + 2, From the given equation, a point on the line is:
3
𝑦−3 (𝑥2 , 𝑦2 , 𝑧2 ) = (2,3,1)
=𝜆 ⇒𝑦 =𝜆+3 We wish to find the coordinates of another point on
1
𝑧−1 the line:
= 𝜆 ⇒ 𝑧 = 2𝜆 + 1 (𝑥1 = 3, 𝑦1 , 𝑧1 )
2
Substitute 𝑥 = 3, and solve for 𝑥: Δ𝑥 = 𝑥1 − 𝑥2 = 3 − 2 = 1
1
𝑥 = 3𝜆 + 2 ⇒ 3 = 3𝜆 + 2 ⇒ 𝜆 =
3 We can find the point on the line without finding
1
Solve for 𝑦 and substitute 𝜆 = 3: the vector form or the parameter 𝜆.
1 10 1 2
𝑦 =𝜆+3= +3= 𝐷𝑅 = Δ𝑥: Δ𝑦: Δ𝑧 = 3: 1: 2 = 1: :
3 3 3 3
1 2 5
𝑧 = 2𝜆 + 1 = 2 ( ) + 1 = + 1 = 1 10
3 3 3 𝑦1 = 𝑦2 + Δ𝑦 = 3 + =
The point is: 3 3
2 5
𝑧1 = 𝑧2 + Δ𝑧 = 1 + =
3 3
Part B
5 1
𝑥− 𝑧−
2 = 𝑦 − 12 = 2 ⇒ (𝑥 , 𝑦 , 𝑧 ) = (5 , 12, 1)
2 2 2
1 −12 1 2 2
Δ𝑦 = 𝑦2 − 𝑦1 = −4 − 12 = −16
P a g e 129 | 168
Get all the files at: https://bit.ly/azizhandouts
Aziz Manva (azizmanva@gmail.com)
4 4
𝐷𝑅 = Δ𝑥: Δ𝑦: Δ𝑧 = 1: −12: 1 = : −16:
3 3
4 5 4 15 8 23
𝑥1 = 𝑥2 + = + = + =
3 2 3 6 6 6
4 1 4 3 8 11
𝑧1 = 𝑧2 + = + = + =
3 2 3 6 6 6
23 11 34
𝑥1 + 𝑧1 = + =
6 6 6
Example 2.39
The equation of a line can be written, in point-slope form as 𝑦 − 𝑦1 = 𝑚(𝑥 − 𝑥1 ). Show that it is equivalent to:
𝑦 − 𝑦1 𝑥 − 𝑥1
=
Δ𝑦 Δ𝑥
𝑦 −𝑦
Substitute 𝑆𝑙𝑜𝑝𝑒 = 𝑚 = 𝑥2 −𝑥1 :
2 1
𝑦2 − 𝑦1
𝑦 − 𝑦1 = ( ) (𝑥 − 𝑥1 )
𝑥2 − 𝑥1
Rearrange:
𝑦 − 𝑦1 𝑥 − 𝑥1
=
𝑦2 − 𝑦1 𝑥2 − 𝑥1
Substitute Δ𝑦 = 𝑦2 − 𝑦1 , Δ𝑥 = 𝑥2 − 𝑥1 :
𝑦 − 𝑦1 𝑥 − 𝑥1
=
Δ𝑦 Δ𝑥
𝑥 − 𝑥1 𝑦 − 𝑦1 𝑧 − 𝑧1
𝜆= = =
Δ𝑥 Δ𝑦 Δ𝑧
Example 2.41
Find the equation of the line passing through points 𝐴(0,6, −9) and 𝐵(−3, −6,3) in Cartesian Form. (CBSE
2010C, Adapted)
Substitute the given data into the two-point form of the equation of a line:
𝑥−0 𝑦−6 𝑧 − (−9)
𝜆1 = = =
−3 − 0 −6 − 6 3 − (−9)
Simplify:
𝑥 𝑦−6 𝑧+9
𝜆1 = = =
−3 −12 12
Divide throughout by 3, and substitute 𝜆1 = 𝜆:
𝑥−0 𝑦−6 𝑧+9
𝜆= = =
−1 −4 4
Example 2.42
P a g e 130 | 168
Get all the files at: https://bit.ly/azizhandouts
Aziz Manva (azizmanva@gmail.com)
Equation of 𝒛 axis
𝑥 𝑦 𝑧
= =
0 0 1
This does not mean that we are dividing by zero. This is simply a way of communicating that:
Δ𝑥 = 𝑥2 − 𝑥1 = 0
Δ𝑦 = 𝑦2 − 𝑦1 = 0
Equation of 𝒚 axis
𝑥 𝑦 𝑧
= =
0 1 0
Equation of 𝒙 axis
𝑥 𝑦 𝑧
= =
1 0 0
Example 2.43
The vector (2,6,3) is parallel to a line.
A. If there is a change of 2 units in the 𝑥 direction, identify the change in the 𝑦 and 𝑧 directions,
respectively.
Example 2.44
𝑥 𝑦 𝑧
The equation of the z axis is given by 0 = 0 = 1. Interpret the direction ratios in this equation.
B. Direction Cosines
Direction ratios and direction cosines are both vectors in the same direction.
However, directions cosines have magnitude 1, and directions ratios can have any magnitude.
Hence, dividing the direction ratios by the magnitude scales the direction ratio vector to make it a unit vector.
Example 2.46
The equation of a line is 5𝑥 − 3 = 15𝑦 + 7 = 3 − 10𝑧. Write the direction cosines of the line. (CBSE 2015)
Convert the given equation into standard form by making the coefficient of each variable 1. For this, we divide
throughout by 𝐿𝐶𝑀(5,15,3) = 30:
P a g e 131 | 168
Get all the files at: https://bit.ly/azizhandouts
Aziz Manva (azizmanva@gmail.com)
3 7 3
5 (𝑥 − ) 15 (𝑦 − (− 15)) −10 (𝑧 − 10)
5 = =
30 30 30
Simplify:
3 7 3
𝑥− 𝑦 − (− ) 𝑧 −
5= 15 = 10
6 2 −3
Identify the direction ratios:
𝐷𝑖𝑟𝑒𝑐𝑡𝑖𝑜𝑛 𝑅𝑎𝑡𝑖𝑜𝑠 = (Δ𝑥, Δ𝑦, Δ𝑧) = (6,2, −3)
We want a vector in the same direction, but with magnitude 1, and hence, we divide the vector by the
magnitude:
(6,2, −3) 1 6 2 3
= (6,2, −3) = ( , , − )
7 7 7 7 7
Example 2.47
4−𝑥 𝑦 1−𝑧
Find the direction cosines of the line = = . (CBSE 2013)
2 6 3
We want a vector in the same direction, but with magnitude 1, and hence, we divide the vector by the
magnitude:
(−2,6, −3) 1 2 6 3
= (−2,6, −3) = (− , , − )
7 7 7 7 7
Example 2.48
Write the direction cosines of the lines joining the points (1,0,0) and (0,1,1). (CBSE 2011)
C. Collinearity
P a g e 132 | 168
Get all the files at: https://bit.ly/azizhandouts
Aziz Manva (azizmanva@gmail.com)
2.49: Collinearity
If 𝑛 points lie on the same line, then they are collinear.
Example 2.50
What is the number of distinct lines that pass through:
A. A point
B. Two points
C. Three Points
𝐴 𝑝𝑜𝑖𝑛𝑡: 𝐼𝑛𝑓𝑖𝑛𝑖𝑡𝑒
𝑇𝑤𝑜 𝑝𝑜𝑖𝑛𝑡𝑠: 𝐸𝑥𝑎𝑐𝑡𝑙𝑦 𝑂𝑛𝑒
𝑇ℎ𝑟𝑒𝑒 𝑝𝑜𝑖𝑛𝑡𝑠: 𝑂𝑛𝑒 𝑜𝑟 𝑍𝑒𝑟𝑜
Example 2.51
A. Are two points always collinear?
B. Are three points always collinear?
We can check the distance between two points (𝑥1 , 𝑦1 , 𝑧1 ) and (𝑥2 , 𝑦2 , 𝑧2 ) using the distance formula:
√(𝑥2 − 𝑥1 )2 + (𝑦2 − 𝑦1 )2 + (𝑧2 − 𝑧1 )2
P a g e 133 | 168
Get all the files at: https://bit.ly/azizhandouts
Aziz Manva (azizmanva@gmail.com)
cos 𝜃 = ±1
⃗⃗⃗⃗⃗⃗⃗
|𝑨𝑩||𝑩𝑪 ⃗⃗⃗⃗⃗⃗⃗| cos 𝜃
= ±1
⃗⃗⃗⃗⃗⃗⃗||𝑩𝑪
|𝑨𝑩 ⃗⃗⃗⃗⃗⃗⃗|
⃗⃗⃗⃗⃗⃗⃗| ∙ |𝑩𝑪
|𝑨𝑩 ⃗⃗⃗⃗⃗⃗⃗|
= ±1
⃗⃗⃗⃗⃗⃗⃗||𝑩𝑪
|𝑨𝑩 ⃗⃗⃗⃗⃗⃗⃗|
Take the absolute value on both sides:
⃗⃗⃗⃗⃗⃗⃗| ∙ |𝑩𝑪
|𝑨𝑩 ⃗⃗⃗⃗⃗⃗⃗|
| |=1
⃗⃗⃗⃗⃗⃗⃗||𝑩𝑪
|𝑨𝑩 ⃗⃗⃗⃗⃗⃗⃗|
If the area of the triangle formed by the points (𝑥1 , 𝑦1 , 𝑧1 ), (𝑥2 , 𝑦2 , 𝑧2 ) and (𝑥3 , 𝑦3 , 𝑧3 ) is zero. That is, if:
𝑥1 𝑦1 𝑧1
𝐴𝑟𝑒𝑎 𝑜𝑓 𝑇𝑟𝑖𝑎𝑛𝑔𝑙𝑒 = |𝑥2 𝑦2 𝑧3 | = 0
𝑥3 𝑦3 𝑧3
𝒊̂ 𝒋̂ ̂
𝒌
𝒂 ⃗⃗
⃗⃗ × 𝒃 = |𝑎1 𝑎2 𝑎3 |
𝑏1 𝑏2 𝑏3
Also, note that ∠𝐴𝑂𝐶 and ∠𝐴𝑂𝐵 are supplementary. If you know one,
you know the other.
⃗⃗⃗⃗⃗⃗
𝒃𝟏 is a vector parallel to the first line. ⃗⃗⃗⃗⃗⃗
𝒃𝟐 is a vector parallel to the second line. The dot product of the vectors
P a g e 134 | 168
Get all the files at: https://bit.ly/azizhandouts
Aziz Manva (azizmanva@gmail.com)
⃗⃗⃗⃗⃗
𝒃𝟏 and ⃗⃗⃗⃗⃗
𝒃𝟐 is given by:
⃗⃗⃗⃗⃗
𝒃𝟏 ∙ ⃗⃗⃗⃗⃗ ⃗⃗⃗⃗⃗𝟏 ||𝒃
𝒃𝟐 = |𝒃 ⃗⃗⃗⃗⃗𝟐 | cos 𝜃
Rearranging gives us:
⃗⃗⃗⃗⃗⃗
𝒃𝟏 ∙ ⃗⃗⃗⃗⃗⃗
𝒃𝟐
cos 𝜃 =
|⃗⃗⃗⃗⃗⃗
𝒃𝟏 ||⃗⃗⃗⃗⃗⃗
𝒃𝟐 |
However, note that 𝜃 can be acute or obtuse, and if it is obtuse, we wish to find the supplement. In that case, we
want:
⃗⃗⃗⃗⃗⃗
𝒃𝟏 ∙ ⃗⃗⃗⃗⃗⃗
𝒃𝟐
cos(180 − 𝜃) = −
|⃗⃗⃗⃗⃗⃗
𝒃𝟏 ||⃗⃗⃗⃗⃗⃗
𝒃𝟐 |
We can combine both the formulas into a single formula by taking the absolute value of the dot product:
⃗⃗⃗⃗⃗⃗
𝒃𝟏 ∙ ⃗⃗⃗⃗⃗⃗
𝒃𝟐
cos 𝜃 = | |
|⃗⃗⃗⃗⃗⃗
𝒃𝟏 ||⃗⃗⃗⃗⃗⃗
𝒃𝟐 |
Example 2.60
̂ + 𝜆(3𝒊̂ + 2𝒋̂ + 6𝒌
⃗⃗ = 2𝒊̂ − 5𝒋̂ + 𝒌
Find the angle between the lines 𝒓 ̂) and 𝒓 ̂ + 𝜇(𝒊̂ + 2𝒋̂ + 2𝒌
⃗⃗ = 7𝒊̂ − 6𝒋̂ − 6𝒌 ̂)
(CBSE 2014, 2008)
Example 2.61
𝑥 − 𝑥1 𝑦 − 𝑦1 𝑧 − 𝑧1 𝑥 − 𝑥2 𝑦 − 𝑦2 𝑧 − 𝑧2 𝑎1 𝑎2 + 𝑏1 𝑏2 + 𝑐1 𝑐2
= = , = = ⇒ cos 𝜃 = | |
𝑎1 𝑏1 𝑐1 𝑎2 𝑏2 𝑐2 √𝑎12 + 𝑏12 + 𝑐12 √𝑎22 + 𝑏22 + 𝑐22
Show that given two lines, the angle 𝜃 between the two lines above is given by the above formula. Do you need
to memorize it?
𝑥 − 𝑥1 𝑦 − 𝑦1 𝑧 − 𝑧1
= = = 𝜆1
𝑎1 𝑏1 𝑐1
𝑥 = 𝑥1 + 𝜆1 𝑎1 , 𝑦 = 𝑦1 + 𝜆1 𝑏1 , 𝑧 = 𝑧1 + 𝜆1 𝑐1
⃗⃗ = (𝑥1 , 𝑦1 , 𝑧1 ) + 𝜆1 (𝑎1 , 𝑏1 , 𝑐1 )
𝒓
𝑥 = 𝑥2 + 𝜆2 𝑎2 , 𝑦 = 𝑦2 + 𝜆2 𝑏2 , 𝑧 = 𝑧2 + 𝜆2
⃗⃗ = (𝑥2 , 𝑦2 , 𝑧2 ) + 𝜆2 (𝑎2 , 𝑏2 , 𝑐2 )
𝒓
Substitute ⃗⃗⃗⃗⃗⃗
𝒃𝟏 = (𝑎1 , 𝑏1 , 𝑐1 ), ⃗⃗⃗⃗⃗⃗
𝒃𝟐 = (𝑎2 , 𝑏2 , 𝑐2 ) in the formula for the angle between two lines:
⃗⃗⃗⃗⃗⃗
𝒃𝟏 ∙ ⃗⃗⃗⃗⃗⃗
𝒃𝟐 𝑎1 𝑎2 + 𝑏1 𝑏2 + 𝑐1 𝑐2
cos 𝜃 = | |=| |
|⃗⃗⃗⃗⃗⃗
𝒃𝟏 ||⃗⃗⃗⃗⃗⃗
𝒃𝟐 | √𝑎12 + 𝑏12 + 𝑐12 √𝑎22 + 𝑏22 + 𝑐22
Example 2.62
4−𝑥 𝑦 1−𝑧
Find the angle between the lines 5𝑥 − 3 = 15𝑦 + 7 = 3 − 10𝑧 and = = .
2 6 3
P a g e 135 | 168
Get all the files at: https://bit.ly/azizhandouts
Aziz Manva (azizmanva@gmail.com)
If two lines are parallel, then then have the same direction ratios.
In other words, a vector parallel to the first line is a scalar multiple of a vector parallel to the other line.
Example 2.64
5
⃗⃗ = (3, −4, − ) + 𝜆1 (2, −1,3) parallel to the line 𝒓
A. Is the line 𝒓 ⃗⃗ = (2, −3, −9) + 𝜆2 (4, −2,6).
3
5 231 521
B. If the line ⃗𝒓⃗ = (3, −4, − ) + 𝜆1 (𝑎, 𝑏, ) is parallel to the line ⃗𝒓⃗ = (2, −3, −9) + 𝜆2 (4, −2, ), then
3 439 1001
find 𝑎: 𝑏.
C. Write the vector equation of a line passing through point (1, −1,2) and parallel to the line whose
𝑥+3 4−𝑦 𝑧+8
equation is 3 = 5 = 6 . (CBSE 2013)
Part A
1
(2, −1,3) = (4, −2,6) ⇒ 𝐷𝑖𝑟𝑒𝑐𝑡𝑖𝑜𝑛 𝑉𝑒𝑐𝑡𝑜𝑟𝑠 𝑎𝑟𝑒 ∥⇒ 𝐿𝑖𝑛𝑒𝑠 𝑎𝑟𝑒 𝑝𝑎𝑟𝑎𝑙𝑙𝑒𝑙.
2
Part B
231 521
(Δ𝑥, Δ𝑦, Δ𝑧) = (𝑎, 𝑏, ) = (4, −2, )
439 1001
𝑎: 𝑏 = 4: −2 = 2: −1
Part C
(1, −1,2) + 𝜆1 (3,5,6)
⃗⃗ = ⏟
𝒓 ⏟ = (1, −1,2) + 𝜆(3,5,6)
⃗⃗⃗⃗⃗
𝒂𝟏 ⃗⃗⃗⃗⃗⃗⃗
𝒃𝟏
P a g e 136 | 168
Get all the files at: https://bit.ly/azizhandouts
Aziz Manva (azizmanva@gmail.com)
𝑥−𝑥1 𝑦−𝑦1 𝑧−𝑧1 𝑥−𝑥2 𝑦−𝑦2 𝑧−𝑧2
Two lines 𝑎1
= 𝑏1
= 𝑐1
and 𝑎2
= 𝑏2
= 𝑐3
are parallel if their direction ratios are equal. That is if:
𝑎1 𝑏1 𝑐1 𝑙1 𝑚1 𝑛1
= = 𝑂𝑅 = =
𝑎2 𝑏2 𝑐2
⏟ 𝑙⏟2 𝑚2 𝑛2
𝑫𝒊𝒓𝒆𝒄𝒕𝒊𝒐𝒏 𝑹𝒂𝒕𝒊𝒐𝒔 𝑫𝒊𝒓𝒆𝒄𝒕𝒊𝒐𝒏 𝑪𝒐𝒔𝒊𝒏𝒆𝒔
Example 2.66
𝑀𝑎𝑟𝑘 𝑡ℎ𝑒 𝑐𝑜𝑟𝑟𝑒𝑐𝑡 𝑂𝑝𝑡𝑖𝑜𝑛
Line 𝑙1 has direction ratios (1,1,1) and line 𝑙2 has direction ratios (−1, −1, −1).
A. The lines are parallel since the direction ratios are .
B. The lines are not parallel since the direction ratios are different.
C. The lines are parallel since the direction cosines are the same.
D. The lines are not parallel since the direction cosines are not the same.
Example 2.67
𝑥+5 2𝑦−2 𝑧−4 −10−𝑥 𝑦−1 𝑧−4
A. Are the lines −3
= 4
= −6
and −3 = −2
= 6
parallel?
B. Find the cartesian equation of the line which passes through the point (−2,4, −5) and is parallel to the
𝑥+3 4−𝑦 𝑧+8
line 3 = 5 = 6 . (CBSE 2013)
Part A
Write the first line in standard form:
𝑥+5 𝑦−1 𝑧−4
= = ⇒ 𝑃𝑎𝑟𝑎𝑙𝑙𝑒𝑙 𝑉𝑒𝑐𝑡𝑜𝑟 = (−3,2, −6)
−3 2 −6
Write the second line in standard form:
𝑥 + 10 𝑦 − 1 𝑧 − 4
= = ⇒ 𝑃𝑎𝑟𝑎𝑙𝑙𝑒𝑙 𝑉𝑒𝑐𝑡𝑜𝑟 = (3, −2,6) = −(−3,2, −6)
3 −2 6
And since the direction ratios are equivalent, the lines are parallel.
Part B
𝑥+3 𝑦−4 𝑧+8
= = ⇒∥ 𝑣𝑒𝑐𝑡𝑜𝑟 = (3, −5,6)
3 −5 6
P a g e 137 | 168
Get all the files at: https://bit.ly/azizhandouts
Aziz Manva (azizmanva@gmail.com)
⃗⃗⃗⃗⃗⃗
𝒃𝟏 ∙ ⃗⃗⃗⃗⃗
𝒃𝟐 = 0
Example 2.70
1−𝑥 7𝑦−14 𝑧−3 7−7𝑥 𝑦−5 6−𝑧
Find the value of 𝜆 so that the lines 3
= 𝜆
= 2
and 3𝜆 = 1
= 5
are at right angles. (CBSE 2019)
➢ We cannot solve for 𝑎, 𝑏 and 𝑐 since there are two equations and three variables.
➢ But we can find the relation between the variables.
P a g e 138 | 168
Get all the files at: https://bit.ly/azizhandouts
Aziz Manva (azizmanva@gmail.com)
A line perpendicular to two lines is parallel to the cross product of the two lines.
Example 2.74
Find the vector and cartesian equations of the line through the point (1,2, −4) and perpendicular to the two
⃗⃗ = (8, −19,10) + 𝜆(3, −16,7) and 𝒓
lines 𝒓 ⃗⃗ = (15,29,5) + 𝜆(3,8, −5)
The line that we want will be parallel to the cross product of the direction ratios of the two given lines.
Hence, we want to find:
𝒊̂ 𝒋̂ ̂
𝒌
⃗⃗⃗⃗⃗
𝒃𝟏 × ⃗⃗⃗⃗⃗
𝒃𝟐 = (3, −16,7) × (3,8, −5) = |3 −16 7 |
3 8 −5
−16 7 3 7 3 −16 ̂
=| | 𝒊̂ − | | 𝒋̂ + | |𝒌
8 −5 3 −5 3 8
= [(−16)(−5) − (8)(7)]𝒊̂ − [(3)(−5) − (3)7]𝒋̂ + [(3)(8) − (−16)(3)]𝒌 ̂
= (80 − 56)𝒊̂ − (15 + 21)𝒋̂ + (24 + 48)𝒌 ̂
= 24𝒊̂ + 36𝒋̂ + 72𝒌 ̂
= 12(2𝒊̂ + 3𝒋̂ + 6𝒌 ̂)
= 12(2,3,6)
The line passing through (1,2, −4) and with direction vector 12(2,3,6) will have equation:
⃗⃗ = (1,2, −4) + 𝛾(2,3,6)
𝒓
D. Skew and Intersecting Lines
The minimum number of dimensions needed for skew lines is 3, since Lines on a 2D coordinate plane are either
parallel or intersecting. There is no third possibility.
Example 2.76
⃗⃗ = (1,1, −1) + 𝜆(3, −1,0) and 𝒓
Show that the lines 𝒓 ⃗⃗ = (4,0, −1) + 𝜇(2,0,3) intersect. Also, find their point of
intersection. (CBSE 2014)
⃗⃗ = (1 + 3𝜆, 1 − 𝜆, −1)
𝒓
⃗⃗ = (4 + 2𝜇, 0, −1 + 3𝜇)
𝒓
P a g e 139 | 168
Get all the files at: https://bit.ly/azizhandouts
Aziz Manva (azizmanva@gmail.com)
A. Using the value of 𝜆 that you found check if the lines are intersecting or not. (CBSE 2019)
B. Are these lines parallel? What kind of lines are these?
Part A
Substitute the value of 𝜆 = 7 that we found earlier:
𝑥−1 𝑦−2 𝑧−3 𝑥−1 𝑦−5 𝑧−6
= = = 𝑡, = = =𝑇
−3 1 2 −3 1 −5
1 + 3𝑡 = 1 + 3𝑇 ⇒ 𝑡 = 𝑇
Substituting 𝑡 = 𝑇
2 + 𝑡 = 5 + 𝑡 ⇒ 2 = 5 ⇒ 𝑁𝑜 𝑆𝑜𝑙𝑢𝑡𝑖𝑜𝑛𝑠.
Example 2.79
⃗⃗ = (1,1,0) + 𝜆(1, −3,2) and 𝒓
37: Find the shortest distance between the lines whose vector equations are 𝒓 ⃗⃗ =
(4,5,6) + 𝜇(2,3,1). (CBSE 2008, 2014)
If two lines are parallel, then they have the same direction ratios.
Example 2.81
⃗⃗ = (1,2, −4) + 𝜆(2,3,6) and 𝒓
40: Find the distance between the lines 𝒓 ⃗⃗ = (3,3, −5) + 𝜇(4,6,12). (CBSE 2014)
F. Distance Formula
P a g e 140 | 168
Get all the files at: https://bit.ly/azizhandouts
Aziz Manva (azizmanva@gmail.com)
Example 2.83
𝑥+2 𝑦+1 𝑧−3
53: Find the points on the line = = at a distance of 5 units from the point 𝑃(1,3,3). (CBSE 2010)
3 2 2
Example 2.86
51
𝑥 𝑦−2 𝑧−3
Consider the point 𝑃(3, −1,11) and the line 2 = 3 = 4 . Find the:
A. coordinates of foot of perpendicular from P to the line.
B. equation of the perpendicular from P to the line.
C. length of the perpendicular from P to the line.
D. Image of P in the line. (CBSE 2011, Adapted)
2.87: Plane
A two-dimensional surface that extends to infinity in all directions is a plane.
2.88: Normal
In maths, normal is used to indicate perpendicularity.
The normal vector is perpendicular to any vector that lies on the plane.
The dot product of two perpendicular vectors is zero.
Since we already have one point on the plane, the locus of the plane is the set of all points 𝑃(𝑥, 𝑦, 𝑧) that satisfy
the equation:
⃗⃗ ∙ ⃗⃗⃗⃗⃗⃗⃗⃗⃗
𝒏 𝑷𝟎 𝑷 = 0
P a g e 141 | 168
Get all the files at: https://bit.ly/azizhandouts
Aziz Manva (azizmanva@gmail.com)
But the direction ratios for the two are not the same.
Example 2.91
The vector equation of a plane requires a point on the plane, and a vector perpendicular to it. For a given plane,
is this vector unique?
Specifically, the unit vector in the direction of the 𝑧 −axis is perpendicular to the
(𝑥, 𝑦) coordinate plane.
̂ = (0,0,1)
𝒌
̂.
And so is any non-zero scalar multiple of 𝒌
Example 2.92
⃗⃗ is perpendicular to a plane. The point 𝐵 lies on the plane. Determine the equation of the plane.
The vector 𝒂
Let 𝑃(𝑥, 𝑦, 𝑧) be a general point on the plane. Then, the equation of the plane is:
⃗⃗⃗⃗⃗⃗⃗ = 0
⃗⃗ ∙ 𝑩𝑷
𝒂
If we want a vector on the plane, it will have direction ratios which are not unique.
However, a vector perpendicular to the plane has unique direction ratios.
P a g e 142 | 168
Get all the files at: https://bit.ly/azizhandouts
Aziz Manva (azizmanva@gmail.com)
⃗⃗ ∙ ⃗⃗⃗⃗⃗⃗⃗⃗⃗
Expand the equation 𝒏 𝑷𝟎 𝑷 = 0:
(𝐴, 𝐵, 𝐶) ∙ (𝑥 − 𝑥0 , 𝑦 − 𝑦0 , 𝑧 − 𝑧0 ) = 0
Multiply using the dot product. This gives us the Cartesian Equation:
𝐴(𝑥 − 𝑥0 ) + 𝐵(𝑦 − 𝑦0 ) + 𝐶(𝑧 − 𝑧0 ) = 0
𝑂𝑝𝑡𝑖𝑜𝑛 𝐴: 𝐿𝑖𝑛𝑒𝑎𝑟
B. Finding the Equation of a Plane
Example 2.97
Find the equation of a plane passing through the point (1,3,7), and with normal vector (7,3,1).
(7,3,1) ∙ (𝑥 − 1, 𝑦 − 3, 𝑧 − 7) = 0
Example 2.98
𝐴𝑛𝑠𝑤𝑒𝑟 𝑒𝑎𝑐ℎ 𝑝𝑎𝑟𝑡 𝑠𝑒𝑝𝑎𝑟𝑎𝑡𝑒𝑙𝑦
Determine the equation of a plane that meets the conditions below. Is the plane unique.
A. (𝑥, 𝑦) coordinate plane.
B. (𝑥, 𝑧) coordinate plane.
C. (𝑦, 𝑧) coordinate plane.
D. plane parallel to the (𝑥, 𝑦) coordinate plane, and passing through (1,2,3)
E. plane perpendicular to the (𝑥, 𝑦) coordinate plane, and passing through (1,2,3)
Part A
The origin lies on the (𝑥, 𝑦) coordinate plane with coordinates
𝑂(0,0,0)
The direction ratios of the normal vector to the coordinate plane are:
(0,0,1)
P a g e 143 | 168
Get all the files at: https://bit.ly/azizhandouts
Aziz Manva (azizmanva@gmail.com)
𝑧=3
Part E
Two equations that work are:
𝑥=1
𝑦=2
C. Defining a Plane
Example 2.99
What is the number of distinct planes that pass through:
A. A point
B. Two points
C. Three collinear points
D. Three non-collinear points
E. Four points, three of which are collinear
F. Four points, 𝑒𝑥𝑎𝑐𝑡𝑙𝑦 three of which are collinear
G. Four points, no three of which are collinear
H. Four points
I. A line and a point
J. A line and a point not on the line
K. Two coplanar lines
L. Two skew lines
Example 2.100
𝑀𝑎𝑟𝑘 𝑡ℎ𝑒 𝑐𝑜𝑟𝑟𝑒𝑐𝑡 𝑜𝑝𝑡𝑖𝑜𝑛
Three points define a:
A. Circle
B. Triangle
P a g e 144 | 168
Get all the files at: https://bit.ly/azizhandouts
Aziz Manva (azizmanva@gmail.com)
C. Line
D. Plane
E. Either {𝐴, 𝐵, 𝐷} or {𝐶}
Note:
➢ Exactly one triangle has its vertices as three non-collinear points and hence three non collinear points
define a triangle.
➢ Exactly one circle passes through three non-collinear points and hence three non collinear points define
a circle.
In general, three non-collinear points define a plane, a triangle and the circle that passes the vertices of the
triangle.
Example 2.101
What is the number of planes that pass through the points:
A.
Example 2.104
Find a nonzero vector that is perpendicular to both of the vectors (2,3,2) and (4,9,5). (Phillips Exeter, Math-3,
2021/537)
Let the vector we want be (𝑥, 𝑦, 𝑧). Dot product of two perpendicular vectors is zero:
(2,3,2)(𝑥, 𝑦, 𝑧) = 0 ⇒ 2𝑥 + 3𝑦 + 2𝑧 = 0
(4,9,5)(𝑥, 𝑦, 𝑧) = 0 ⇒ 4𝑥 + 9𝑦 + 5𝑧 = 0
Note that we have three variables and two equations. Since, the number of equations is less than the number of
variables, the system of equations is 𝑢𝑛𝑑𝑒𝑟𝑑𝑒𝑡𝑒𝑟𝑚𝑖𝑛𝑒𝑑, and it may have infinite solutions.
Since the system is underdetermined, we substitute 𝑧 = 1, and solve the resulting system of two equations in
two variables:
2𝑥 + 3𝑦 = −2, 4𝑥 + 9𝑦 = −5
Multiply Equation I by 2, and subtract the new equation from Equation II:
1
0 + 3𝑦 = −1 ⇒ 𝑦 = −
3
1
Substitute 𝑦 = − 3:
P a g e 145 | 168
Get all the files at: https://bit.ly/azizhandouts
Aziz Manva (azizmanva@gmail.com)
1 1
4𝑥 + 6 (− ) = −4 ⇒ 4𝑥 − 2 = −4 ⇒ 4𝑥 = −2 ⇒ 𝑥 = −
3 2
Hence, if 𝑧 = 1, we have determined unique values of 𝑥 and 𝑦. Hence, one vector that meets the conditions is:
1 1
(− , − , 1)
2 3
If we wish to eliminate fractions, we can multiply by 6 to get:
1 1
= 6 (− , − , 1) = (−3, −2,6)
2 3
D. Parallel Planes
Two planes are parallel when they are oriented in the same direction. We can check this in a few ways.
Normal vectors of the two planes have the same direction cosines. Direction cosines are precise. Hence, they
must be the 𝑠𝑎𝑚𝑒.
𝑂𝑝𝑡𝑖𝑜𝑛 𝐴 𝑖𝑠 𝑐𝑜𝑟𝑟𝑒𝑐𝑡
Normal vectors of the two planes have equivalent direction ratios. We can check this in a couple of ways:
➢ Convert direction ratios to direction cosines and see if they are equal.
𝑂𝑝𝑡𝑖𝑜𝑛 𝐵 𝑖𝑠 𝑐𝑜𝑟𝑟𝑒𝑐𝑡
➢ See if the normal vector of one plane is a non-zero. scalar multiple of the other
𝑂𝑝𝑡𝑖𝑜𝑛 𝐺 𝑖𝑠 𝑐𝑜𝑟𝑟𝑒𝑐𝑡
➢ If the cross product is zero, then the vectors are parallel.
𝑂𝑝𝑡𝑖𝑜𝑛 𝐸 𝑖𝑠 𝑐𝑜𝑟𝑟𝑒𝑐𝑡
If the dot product is zero, the vectors are perpendicular. If the dot product is 1, the vectors may or not be
parallel.
𝑂𝑝𝑡𝑖𝑜𝑛𝑠 𝐶 𝑎𝑛𝑑 𝐷 𝑎𝑟𝑒 𝑛𝑜𝑡 𝑐𝑜𝑟𝑟𝑒𝑐𝑡
P a g e 146 | 168
Get all the files at: https://bit.ly/azizhandouts
Aziz Manva (azizmanva@gmail.com)
holds:
➢ Normal vectors of the two planes have the same direction cosines.
➢ Normal vectors of the two planes have equivalent direction ratios.
➢ Cross product of the normal vectors is zero.
➢ Normal vector of one plane is a scalar multiple of the other
Example 2.108
Plane 𝑃 is parallel to −4𝑥 + 5𝑦 − 2𝑧 = 37 and passes through (−2,8,5). Write the Vector and Cartesian
equations of 𝑃 in point form.
Vector Form
The Vector equation of a plane in point form is:
⃗⃗ ∙ ⃗⃗⃗⃗⃗⃗⃗⃗⃗
𝒏
⏟ 𝑷𝟎 𝑷 = 0
𝑬𝒒𝒖𝒂𝒕𝒊𝒐𝒏 𝑰
⃗⃗ = (−4,5, −2) and use 𝑃0 (𝑥0 , 𝑦0 , 𝑧0 ) = (−2,8,5) and 𝑃(𝑥, 𝑦, 𝑧) to find:
Note that ⏟
𝒏
𝑬𝒒𝒖𝒂𝒕𝒊𝒐𝒏 𝑰𝑰
⃗⃗⃗⃗⃗⃗⃗⃗⃗
⏟𝟎 𝑷 = (𝑥 + 2, 𝑦 − 8, 𝑧 − 5)
𝑷
𝑬𝒒𝒖𝒂𝒕𝒊𝒐𝒏 𝑰𝑰𝑰
Substitute Equation II and III in Equation I to get the Vector equation in point form:
(−4,5, −2) ∙ (𝑥 + 2, 𝑦 − 8, 𝑧 − 5) = 0
Cartesian Form
To convert to the Cartesian form, expand the LHS using the dot product definition:
−4(𝑥 + 2) + 5(𝑦 − 8) − 2(𝑧 − 5) = 0
Example 2.110
The plane 𝑃 passes through the point (𝑎, 𝑏, 𝑐) and is parallel to the plane 𝒓
⃗⃗ ∙ (1,1,1) = 2. Write the Vector and
Cartesian equations of 𝑃 in point form. (CBSE 2014, Adapted)
⃗⃗ ∙ (1,1,1) = 2 is parallel to plane 𝑃. Since the two planes are parallel, the direction ratios of
We have that plane 𝒓
their normal vectors are equivalent. Hence, the direction ratios of the normal vector to plane 𝑃 are:
⃗⃗ = (1,1,1)
𝒏
Vector Form
The Vector equation of a plane in point form is:
⃗⃗ ∙ ⃗⃗⃗⃗⃗⃗⃗⃗⃗
𝒏
⏟ 𝑷𝟎 𝑷 = 0
𝑬𝒒𝒖𝒂𝒕𝒊𝒐𝒏 𝑰
⃗⃗ = (1,1,1) and use 𝑃0 (𝑥0 , 𝑦0 , 𝑧0 ) = (𝑎, 𝑏, 𝑐) and 𝑃(𝑥, 𝑦, 𝑧) to find:
Note that ⏟
𝒏
𝑬𝒒𝒖𝒂𝒕𝒊𝒐𝒏 𝑰𝑰
P a g e 147 | 168
Get all the files at: https://bit.ly/azizhandouts
Aziz Manva (azizmanva@gmail.com)
⃗⃗⃗⃗⃗⃗⃗⃗⃗
⏟𝟎 𝑷 = (𝑥 − 𝑎, 𝑦 − 𝑏, 𝑧 − 𝑐)
𝑷
𝑬𝒒𝒖𝒂𝒕𝒊𝒐𝒏 𝑰𝑰𝑰
Substitute Equation II and III in Equation I to get the Vector equation in point form:
(1,1,1) ∙ (𝑥 − 𝑎, 𝑦 − 𝑏, 𝑧 − 𝑐) = 0
Cartesian Form
To convert to Cartesian form, expand the LHS using the dot product definition:
(𝑥 − 𝑎) + (𝑦 − 𝑏) + (𝑧 − 𝑐) = 0
Example 2.112
Write a Cartesian equation in general form for the plane that contains the point 𝐴 = (4,5, −3) and that is
perpendicular to the line through 𝐵 = (5, −2, −2) and 𝐶 = (7,1,4). (Phillips Exeter, Math-3, 2021/548)
Example 2.113
𝑥−2 𝑦−1 𝑧+5
Find the value of 𝜆 such that the line 6
= 𝜆
= −4
is perpendicular to the plane 3𝑥 − 𝑦 − 2𝑧 = 7. (CBSE
2010)
P a g e 148 | 168
Get all the files at: https://bit.ly/azizhandouts
Aziz Manva (azizmanva@gmail.com)
Since the line is perpendicular to the plane, its direction ratios are proportional to the normal vector:
2(3, −1, −2) = (6, −2, −4) = (6, 𝜆, −4)
𝜆 = −2
Example 2.115
(1,2, −3) ∙ (𝑥 − 3, 𝑦 − 7, 𝑧 + 4) = 0
In the equation of the plane above, identify:
A. The direction ratios of a vector normal to the plane.
B. A point on the plane
Example 2.116
Write the vector equation of the line passing through (1,2,3) and perpendicular to the plane ⃗𝒓⃗ ∙ (1,2, −5) + 9 =
0. (CBSE 2015)
Example 2.117
Find the equation of the plane passing through the points 𝑃(8,3, −2), 𝑄(−2, −5, −1) and 𝑅(2, −4,3).
P a g e 149 | 168
Get all the files at: https://bit.ly/azizhandouts
Aziz Manva (azizmanva@gmail.com)
There are three equations and two variables. Let 𝑎 = 1 in the above system:
−10 − 8𝑏 + 𝑐 = 0
−6 − 7𝑏 + 5𝑐 = 0
Solving the above system gives us the values 𝐴, 𝐵, 𝐶 and hence the direction ratios of the normal vector:
4 2
(𝐴, 𝐵, 𝐶) = [1, − , − ]
3 3
Find the Constant
4 2
Substitute 𝑃(8,3, −2) and the normal (𝐴, 𝐵, 𝐶) = [1, − , − ] into the equation of a plane to find 𝐷:
3 3
4 2 16
𝐴𝑥 + 𝐵𝑦 + 𝐶𝑧 = 𝐷 ⇒ 1(8) + (− ) (3) − ( ) (−2) = =𝐷
3 3 3
Find the Equation of the Plane
4 2 16
Substitute (𝐴, 𝐵, 𝐶) = [1, − 3 , − 3] and 𝐷 =
into the equation of a plane:
3
4 2 16
𝐴𝑥 + 𝐵𝑦 + 𝐶𝑧 = 𝐷 ⇒ 1𝑥 − 𝑦 − 𝑧 = ⇒ 3𝑥 − 4𝑦 − 2𝑧 = 16
3 3 3
A. In the above solution, after finding the equations −10𝑎 − 8𝑏 + 𝑐 = 0 and −6𝑎 − 7𝑏 + 5𝑐 = 0, we
substituted 𝑎 = 1. Why did we choose 1? Can you substitute any other value?
B. In the system of the equations, if you substitute 𝑎 = 0, what values of 𝑏 and 𝑐 do you get? Are they
correct? How do you interpret them?
Part A
There are two parts to this question. The first is, why is the substitution valid.
From an algebraic point of view:
There are three variables and two equations. Hence, you will not be to arrive at a unique solution. Hence, it
makes sense to substitute one variable.
Part B
Try making the substitution 𝑎 = 0:
−8𝑏 + 𝑐 = 0 ⇒ 𝑐 = 8𝑏
7
−7𝑏 + 5𝑐 = 0 ⇒ 𝑐 = − 𝑏
5
7
8𝑏 = − 𝑏 ⇒ 𝑏 = 0 ⇒ 𝑐 = 0
5
We get
𝑏=𝑐=0
We are looking for the normal vector. The normal vector gives us the direction ratios. If 𝑎 = 0, then the change
in the 𝑥 −direction is zero, and hence (in this case), the change in the other two directions is also zero.
P a g e 150 | 168
Get all the files at: https://bit.ly/azizhandouts
Aziz Manva (azizmanva@gmail.com)
The information that we get is correct, but not useful to us for determining the normal vector.
B. Perpendicular Planes
For two vectors to be perpendicular their normal vectors must be perpendicular to each other. Hence, checking
their dot product is sufficient.
Example 2.119
Consider plane 𝑃. Is a plane perpendicular to 𝑃 and one point on 𝑃 sufficient to define 𝑃?
Suppose P is perpendicular to the (𝑥, 𝑦) coordinate plane, and it passes through (0,0,0).
However, if you rotate you will get an infinite number of planes.
Example 2.120
Write the equation for a plane perpendicular to −4𝑥 + 5𝑦 − 2𝑧 = 37, and passing through (−2,8,5).
No.
Example 2.121
Write the equation for the plane perpendicular to −4𝑥 + 5𝑦 − 2𝑧 = 37, and passing through (−2,8,5) and
(1,1,1).
P a g e 151 | 168
Get all the files at: https://bit.ly/azizhandouts
Aziz Manva (azizmanva@gmail.com)
Yes.
Example 2.123
The intersection of two planes is:
A.
P a g e 152 | 168
Get all the files at: https://bit.ly/azizhandouts
Aziz Manva (azizmanva@gmail.com)
Example 2.124
Find the equation of the line where the plane 2𝑥 − 𝑦 − 4𝑧 = 8 and 4𝑥 + 𝑦 + 2𝑧 = 10 intersect.
1 10
Substitute 𝒂 ⃗⃗ = (− ,
⃗⃗ = (3, −2,0), 𝒃 , −1) in the equation of a line:
3 3
1 10 𝜆 10𝜆
⃗⃗ = 𝒂
𝒓 ⃗⃗ = (3, −2,0) + 𝜆 (− , , −1) = (3 − , −2 +
⃗⃗ + 𝜆𝒃 , −𝜆)
3 3 3 3
Example 2.125
𝑥−2 𝑦+1 𝑧−12
15: Find the distance between the point (−1, −5, −10) and the point of intersection of the line 3
= 4
= 2
and the plane 𝑥 − 𝑦 + 𝑧 = 5. (CBSE 2015)
Example 2.126
Which point on the plane 2𝑥 + 3𝑦 + 6𝑧 = 98 is closest to the origin?
The point on the plane closest to any point lies on the line
perpendicular to that point. The vector of coefficients (2,3,6) is
perpendicular to the plane.
∴ Both the origin, and (2,3,6) lie on the line perpendicular to the plane.
P a g e 153 | 168
Get all the files at: https://bit.ly/azizhandouts
Aziz Manva (azizmanva@gmail.com)
The point on the plane closest to any point lies on the line perpendicular to
that point.
The vector of coefficients (2,5, −1) is perpendicular to the plane.
∴ Both (−1.2, −12.5,6.1), and (2,5,-1) lie on the line perpendicular to the
plane.
Find the intersection of the line and the plane. Substituting the above in the
equation of the plane 2𝑥 + 3𝑦 + 6𝑧 = 98:
2(−1.2 + 2𝑡) + 5(−12.5 + 5𝑡) − 1(6.1 − 𝑡) = −8 ⇒ 𝑡 = 2.1
Hence,
(𝑥, 𝑦, 𝑧) = (−1.2 + 2𝑡, −12.5 + 5𝑡, 6.1 − 𝑡) = (3, −2,4)
Use the distance formula to find the distance between (−1.2, −12.5,6.1) and (3, −2,4)
𝑑 = √(−1.2 − 3)2 + (−12.5 + 2)2 + (6.1 − 4)2 = 11.502
The points (𝑥1 , 𝑦1 , 𝑧1 ) and (𝐴, 𝐵, 𝐶) lie on a line perpendicular to the plane with equation:
(𝑥, 𝑦, 𝑧) = (𝑥1 , 𝑦1 , 𝑧1 ) + 𝜆(𝐴, 𝐵, 𝐶) = (𝑥1 + 𝜆𝐴, 𝑦1 + 𝜆𝐵, 𝑧1 + 𝜆𝐶)
Substitute the above into 𝐴𝑥 + 𝐵𝑦 + 𝐶𝑧 = 𝐷 to determine the point of intersection of the line and the plane:
𝐴(𝑥1 + 𝜆𝐴) + 𝐵(𝑦1 + 𝜆𝐵) + 𝐶(𝑧1 + 𝜆𝐶) = 𝐷
Move the constants to the other side, and factor 𝜆 on the LHS:
𝜆(𝐴2 + 𝐵2 + 𝐶 2 ) = 𝐷 − 𝐴𝑥1 − 𝐵𝑦1 − 𝐶𝑧1
Solve for 𝜆:
𝐷 − 𝐴𝑥1 − 𝐵𝑦1 − 𝐶𝑧1
𝜆=
⏟ 𝐴2 + 𝐵2 + 𝐶 2
𝑬𝒒𝒖𝒂𝒕𝒊𝒐𝒏 𝑰
We now want to find the distance between the points for the value of 𝜆 above.
(𝑥1 , 𝑦1 , 𝑧1 ) 𝑎𝑛𝑑 (𝑥1 + 𝜆𝐴, 𝑦1 + 𝜆𝐵, 𝑧1 + 𝜆𝐶)
By the distance formula, the distance is:
√(𝑥1 − 𝑥1 + 𝜆𝐴)2 + (𝑦1 − 𝑦1 + 𝜆𝐵)2 + (𝑧1 − 𝑧1 + 𝜆𝐶)2
This simplifies to:
√(𝜆𝐴)2 + (𝜆𝐵)2 + (𝜆𝐶)2
Which on expansion gives:
P a g e 154 | 168
Get all the files at: https://bit.ly/azizhandouts
Aziz Manva (azizmanva@gmail.com)
10
Example 2.129
The formula that we found above is for distance of a point from a plane. We had earlier found a formula for the
distance of a plane from the origin. What is the connection between the two formulas. Justify.
Note that
𝐹𝑜𝑟𝑚𝑢𝑙𝑎 1 =
𝐹𝑜𝑟𝑚𝑢𝑙𝑎 2 =
Substitute the coordinates of the origin (0,0,0) in the formula for the distance of a point from a plane:
Example 2.131
Convert the direction ratios (𝐴, 𝐵, 𝐶) to direction cosines.
P a g e 155 | 168
Get all the files at: https://bit.ly/azizhandouts
Aziz Manva (azizmanva@gmail.com)
Since each the coefficients on the LHS are direction cosines, use the conventional notation (𝑙, 𝑚, 𝑛) for them,
𝐷
and let = 𝑝:
√𝐴2 +𝐵2+𝐶 2
𝑙𝑥 + 𝑚𝑦 + 𝑛𝑧 = 𝑝
Example 2.133
Write the plane 2𝑥 − 3𝑦 + 𝑧 = 3 in normal form.
Divide both sides by the magnitude of the direction ratio vector = √22 + (−3)2 + 12 = √14:
2 3 𝑧 3
𝑥− 𝑦+ =
√14 √14 √14 √14
𝑥
𝑙= = cos 𝛼 ⇒ 𝑥 = 𝑙 |𝒓
⃗⃗|
|𝒓
⃗⃗|
𝑦
𝑚= = cos 𝛽 ⇒ 𝑦 = 𝑚|𝒓 ⃗⃗|
|𝒓
⃗⃗|
𝑧
𝑛= = cos 𝛾 ⇒ 𝑧 = 𝑛|𝒓 ⃗⃗|
|𝒓
⃗⃗|
P a g e 156 | 168
Get all the files at: https://bit.ly/azizhandouts
Aziz Manva (azizmanva@gmail.com)
⃗⃗ is the vector perpendicular to the plane with tail at the origin and origin and tip at the plane.
And recall that 𝒓
Hence
⃗⃗ = |𝒓
𝐿𝑒𝑛𝑔𝑡ℎ 𝑜𝑓 𝒓 ⃗⃗| = 𝑝 = 𝐷𝑖𝑠𝑡𝑎𝑛𝑐𝑒 𝑜𝑓 𝑃𝑙𝑎𝑛𝑒 𝑓𝑟𝑜𝑚 𝑜𝑟𝑖𝑔𝑖𝑛
Example 2.135
A. Determine the distance of the plane 𝑥 − 4𝑦 + 2𝑧 = 7 from the origin.
B. 12: Write the distance of the following plane from the origin: 2𝑥 − 𝑦 + 2𝑧 + 1 = 0. (CBSE 2010)
C. Find the length of the perpendicular drawn from the origin to the plane 2𝑥 − 3𝑦 + 6𝑧 + 21 = 0 from the
origin. (CBSE 2011)
Part A
Divide both sides by the magnitude of the direction ratio vector = √12 + (−4)2 + 22 = √21:
𝑥 − 4𝑦 + 2𝑧 7 7
= ⇒ 𝐷𝑖𝑠𝑡𝑎𝑛𝑐𝑒 =
√21 √21 √21
Part B
2𝑥 − 𝑦 + 2𝑧 = −1
Divide both sides by the magnitude of the direction ratio vector = √22 + (−1)2 + 22 = √9 = 3:
2𝑥 − 𝑦 + 2𝑧 1
=−
3 3
Distance is
1 1
|− | =
3 3
Part C
The length of the perpendicular to the plane from the origin is just the distance of the plane from the origin.
−2𝑥 + 3𝑦 − 6𝑧 = 21
Divide both sides by the magnitude of the direction ratio vector = √(−2)2 + 32 + (−6)2 = √49 = 7:
−2𝑥 + 3𝑦 − 6𝑧
=3
7
Distance from the origin
=3
Since each the coefficients on the LHS are direction cosines, use the conventional notation (𝑙, 𝑚, 𝑛) for them, d
𝐷
let = 𝑝:
√𝐴2 +𝐵2 +𝐶 2
P a g e 157 | 168
Get all the files at: https://bit.ly/azizhandouts
Aziz Manva (azizmanva@gmail.com)
𝐷
𝑙𝑥 + 𝑚𝑦 + 𝑛𝑧 =
√𝐴2 + 𝐵2 + 𝐶 2
Since the above is in normal, the distance from the origin is:
𝐷
√𝐴2 + 𝐵2 + 𝐶 2
Part A
➢ The LHS of the normal form has a unit vector multiplying the vector (𝑥, 𝑦, 𝑧), whereas the general form
has some non-zero multiple of it.
➢ The RHS of the normal form indicates the distance from the origin, but this may not be true in the
general form.
Part B
Divide both sides by the magnitude of the direction ratios.
Example 2.139
A. Find the vector equation of a plane which is a distance of 5 units from the origin, and its normal vector
is (2, −3,6). (CBSE 2016)
B. Write the vector equation of a plane which is at a distance of 5√3 from the origin and the normal to
which is equally inclined to the coordinate axes. (CBSE 2016)
Part A
̂∙𝒓
𝒏 ⃗⃗ = 𝑝
⃗⃗
𝒏
⃗⃗ = 𝑝
∙𝒓
|𝒏
⃗⃗|
(2, −3,6)
⃗⃗ = 5
∙𝒓
√22 + (−3)2 + 62
(2, −3,6) ∙ 𝒓
⃗⃗ = 5√49
(2, −3,6) ∙ 𝒓⃗⃗ = 35
Part B
The direction cosines meet the constraint:
𝑙 2 + 𝑚2 + 𝑛2 = 1
P a g e 158 | 168
Get all the files at: https://bit.ly/azizhandouts
Aziz Manva (azizmanva@gmail.com)
Now the distance of 𝑃1 from the origin is |𝑝1 |. And the distance of 𝑃2 from the origin is |𝑝2 |.
Hence, the origin, the closest point of 𝑃1 from the origin, and the closest point of 𝑃2 from the origin lie on a
straight line.
|3 − (−5)| = 8
Example 2.142
Find the distance between the planes given below:
1 2 2
𝑥− 𝑦+ 𝑧=7
5 √5 5
1 2 2
𝑥− 𝑦 + 𝑧 = −12
5 √5 5
Since the sum is 1, the equations are already in normal form, and hence the distance between the planes is:
|𝑝1 − 𝑝2 | = |7 − (−12)| = 19
Example 2.143
𝐴𝑥 + 𝐵𝑦 + 𝐶𝑧 = 𝐷1
P a g e 159 | 168
Get all the files at: https://bit.ly/azizhandouts
Aziz Manva (azizmanva@gmail.com)
𝐴𝑥 + 𝐵𝑦 + 𝐶𝑧 𝐷
=
√𝐴2 + 𝐵2 + 𝐶 2 √𝐴2 + 𝐵2 + 𝐶 2
Hence, if the direction ratios are scaled by a factor of 𝑞, the magnitude of the normal vector is also scaled by 𝑞.
Example 2.144
1 3
The two planes 𝑥 − 4𝑦 + 7𝑧 = 𝑝1 and 𝑥 − 𝑏𝑦 + 𝑐𝑧 = 𝑝2 are parallel. Let the distance between the planes be
3 5
𝐷. Find 𝐷 + 𝑏 + 𝑐.
Instead of dividing both sides by the magnitude, multiply both sides by the reciprocal of the number above:
3 1
( ) ( 𝑥 − 4𝑦 + 7𝑧)
√586 3 =
3
× 𝑝1
√586 √586
3
1 3 9
𝑎= ⇒𝑎=
3 5 5
9 1 3 36 63 3
( , −4,7) = ( , − , ) = ( , 𝑏, 𝑐)
5 3 5 5 5 5
36 63
𝑏=− ,𝑐 =
5 5
3 36 63
𝑥 + 𝑦 + 𝑧 = 𝑝2
5 5 5
√586 9 3√586
Divide both sides by the magnitude of the direction vector 3 × 5 = 5 :
5 3 36 63 5
( ) ( 𝑥 + 𝑦 + 𝑧) = × 𝑝2
3√586 5 5 5 3√586
P a g e 160 | 168
Get all the files at: https://bit.ly/azizhandouts
Aziz Manva (azizmanva@gmail.com)
11
Example 2.145
1: Find the distance between the planes 2𝑥 − 𝑦 + 2𝑧 = 5 and 5𝑥 − 2.5𝑦 + 5𝑧 = 20. (CBSE 2017)
12
Example 2.146
Find the distance between two parallel planes in general form. Assume variables as necessary.
Two planes are parallel when their normal vectors are oriented in the same direction. Hence, we can write the
planes in general form as:
𝐴𝑥 + 𝐵𝑦 + 𝐶𝑧 = 𝐷1
𝐴𝑥 + 𝐵𝑦 + 𝐶𝑧 = 𝐷2
Since the above are now in normal form, the distance between them is:
𝐷1 𝐷2
| − |
2 2
√𝐴 + 𝐵 + 𝐶 2 √𝐴 + 𝐵2 + 𝐶 2
2
Which simplifies to:
𝐷1 − 𝐷2
=| |
√𝐴2 + 𝐵2 + 𝐶 2
No. It is better to convert to normal form, and use the simpler formula.
At the 𝑥 − 𝑎𝑥𝑖𝑠, we must have 𝑦 = 0, 𝑧 = 0, which when we substitute in the above equation:
𝐷
𝐴𝑥 + 0𝑦 + 0𝑧 = 𝐷 ⇒ 𝐴𝑥 = 𝐷 ⇒ 𝑥 =
𝐴
P a g e 161 | 168
Get all the files at: https://bit.ly/azizhandouts
Aziz Manva (azizmanva@gmail.com)
Similarly:
𝐷
𝑦 − 𝑖𝑛𝑡𝑒𝑟𝑐𝑒𝑝𝑡: 0𝑥 + 𝐵𝑦 + 0𝑧 = 𝐷 ⇒ 𝐵𝑦 = 𝐷 ⇒ 𝑦 =
𝐵
𝐷
𝑧 − 𝑖𝑛𝑡𝑒𝑟𝑐𝑒𝑝𝑡: 0𝑥 + 0𝑦 + 𝐶𝑧 = 𝐷 ⇒ 𝐶𝑧 = 𝐷 ⇒ 𝑧 =
𝐶
Example 2.148
3 2 5
What is the sum of the intercepts that the plane 5 𝑥 + 3 𝑦 − 9 𝑧 = 11 makes with the axes?
3 2 5
𝑥+ 𝑦− 𝑧=1
55 33 99
𝑥 𝑦 𝑧
+ 𝑦− 𝑧=1
55 33 99
3 2 5
Sum of the intercepts is:
55 33 99 550 495 594 451
+ − = + − =
3 2 5 30 30 30 30
Example 2.149
Find the sum of the intercepts cut off by the plane 2𝑥 + 𝑦 − 𝑧 = 5 on the coordinate axes. (CBSE 2015, 2011)
P a g e 162 | 168
Get all the files at: https://bit.ly/azizhandouts
Aziz Manva (azizmanva@gmail.com)
𝑥 𝑦 𝑧
+ + =1
𝑎 𝑏 𝑐
Factor:
1 1 1
( , , ) ∙ (𝑥, 𝑦, 𝑧) = 1
𝑎 𝑏 𝑐
1 1 1
⃗⃗ = (𝑥, 𝑦, 𝑧)
⃗⃗ = ( , , ) , 𝒓
Substitute 𝒏 𝑎 𝑏 𝑐
⃗⃗ ∙ 𝒓
𝒏 ⃗⃗ = 1
Example 2.151
A. Find the vector equation of the plane with intercepts 3, −4 and 2 on 𝑋, 𝑌 and 𝑍 axes. (CBSE 2016)
⃗⃗ ∙ (2,1, −1) − 5 = 0 on the three axes. (CBSE 2015)
B. Write the sum of intercepts cut off by the plane 𝒓
2 3 5 1
⃗⃗ ∙ ( , − , ) − = 0 on the three axes.
C. Find the intercepts cut off by the plane 𝒓 3 5 7 3
B. Revision
P a g e 163 | 168
Get all the files at: https://bit.ly/azizhandouts
Aziz Manva (azizmanva@gmail.com)
Part A
Divide both sides of 𝐴𝑥 + 𝐵𝑦 + 𝐶𝑧 = 𝐷 by the magnitude of the normal vector = √𝐴2 + 𝐵2 + 𝐶 2 to get:
𝐴 𝐵 𝐶 𝐷
𝑥+ 𝑦+ 𝑧=
√𝐴2 + 𝐵2 + 𝐶 2 √𝐴2 + 𝐵2 + 𝐶 2 √𝐴2 + 𝐵2 + 𝐶 2 √𝐴2 + 𝐵2 + 𝐶 2
Part B
Divide both sides of 𝐴𝑥 + 𝐵𝑦 + 𝐶𝑧 = 𝐷 by 𝐷 to get:
𝐴 𝐵 𝐶 𝑥 𝑦 𝑧
𝑥+ 𝑦+ 𝑧=1⇒ + + =1
𝐷 𝐷 𝐷 𝐷 𝐷 𝐷
𝐴 𝐵 𝐶
Part C
Expand the LHS, simplify and send the constant to the RHS
Part D
Convert to general form, and then convert from general form to point form.
Part E
Convert to general form, and then convert from general form to point intercept.
Part F
Multiply on both sides by any non-zero quantity
Part G
𝑙 𝑚 𝑛
Divide both sides of 𝑙𝑥 + 𝑚𝑦 + 𝑛𝑧 = 𝑝 by 𝑝 to get 𝑝 𝑥 + 𝑝
𝑦 +𝑝𝑧 = 1
Part H
𝑥 𝑦 𝑧
Multiply sides of 𝑎 + 𝑏 + 𝑐 = 1 by 𝑎𝑏𝑐 to get:
𝑏𝑐𝑥 + 𝑎𝑐𝑦 + 𝑎𝑏𝑧 = 1
𝐴 = 𝑏𝑐, 𝐵 = 𝑎𝑐, 𝐶 = 𝑎𝑏
Example 2.154
Plane 𝑃 is parallel to −4𝑥 + 5𝑦 − 2𝑧 = 37 and passes through (−2,8,5). Write the Vector and Cartesian
equations of 𝑃 in point form, general form, normal form and intercept form.
Point Form
The Vector equation of a plane in point form is:
⃗⃗ ∙ ⃗⃗⃗⃗⃗⃗⃗⃗⃗
𝒏
⏟ 𝑷𝟎 𝑷 = 0
𝑬𝒒𝒖𝒂𝒕𝒊𝒐𝒏 𝑰
⃗ = (−4,5, −2) and use 𝑃0 (𝑥0 , 𝑦0 , 𝑧0 ) = (−2,8,5) and 𝑃(𝑥, 𝑦, 𝑧) to find:
⃗⏟
Note that 𝒏
𝑬𝒒𝒖𝒂𝒕𝒊𝒐𝒏 𝑰𝑰
P a g e 164 | 168
Get all the files at: https://bit.ly/azizhandouts
Aziz Manva (azizmanva@gmail.com)
⃗⃗⃗⃗⃗⃗⃗⃗⃗
⏟𝟎 𝑷 = (𝑥 + 2, 𝑦 − 8, 𝑧 − 5)
𝑷
𝑬𝒒𝒖𝒂𝒕𝒊𝒐𝒏 𝑰𝑰𝑰
Substitute Equation II and III in Equation I to get the Vector equation in point form:
(−4,5, −2) ∙ (𝑥 + 2, 𝑦 − 8, 𝑧 − 5) = 0
To convert to the Cartesian form, expand the LHS using the dot product definition:
−4(𝑥 + 2) + 5(𝑦 − 8) − 2(𝑧 − 5) = 0
General Form
To convert to general form, use the distributive property on the LHS:
(−4𝑥 − 8) + (5𝑦 − 40) + (−2𝑧 + 10) = 0
Simplify to get the Cartesian equation in general form:
−4𝑥 + 5𝑦 − 2𝑧 = 38
Factor to get the vector version:
(−4,5, −2) ∙ (𝑥, 𝑦, 𝑧) = 38
Normal Form
The magnitude of the normal vector is:
√(−4)2 + 52 + (−22 ) = √45 = 3√5
To convert to normal form, divide both sides of the Cartesian equation from above by the magnitude of the
normal vector:
4 5 2 38
− 𝑥+ 𝑦− 𝑧=
3√5 3√5 3√5 3√5
Factor to get the vector form:
4 5 2 38
(− , ,− ) ∙ (𝑥, 𝑦, 𝑧) =
3√5 3√5 3√5 3√5
Intercept Form
To convert to intercept form, divide both sides of the Cartesian equation in general form by 38:
4 5 2
− 𝑥+ 𝑦− 𝑧=1
38 38 38
Rearrange in intercept form:
𝑥 𝑦 𝑧
+ 𝑦+− =1
19 38 −19
−
2 5
Factor to get the vector version:
1 1 1
( , , ) ∙ (𝑥, 𝑦, 𝑧) = 1
19 38 −19
− 2
5
Example 2.155
The plane 𝑃 passes through the point (𝑎, 𝑏, 𝑐) and is parallel to the plane 𝒓
⃗⃗ ∙ (1,1,1) = 2. Write the Vector and
Cartesian equations of 𝑃 in point form, general form, normal form and intercept form. (CBSE 2014, Adapted)
⃗⃗ ∙ (1,1,1) = 2 is parallel to plane 𝑃. Since the two planes are parallel, the direction ratios of
We have that plane 𝒓
their normal vectors are equivalent. Hence, the direction ratios of the normal vector to plane 𝑃 are:
⃗⃗ = (1,1,1)
𝒏
Point Form
The Vector equation of a plane in point form is:
P a g e 165 | 168
Get all the files at: https://bit.ly/azizhandouts
Aziz Manva (azizmanva@gmail.com)
⃗ ∙ ⃗⃗⃗⃗⃗⃗⃗⃗⃗
⃗⏟
𝒏 𝑷𝟎 𝑷 = 0
𝑬𝒒𝒖𝒂𝒕𝒊𝒐𝒏 𝑰
⃗⃗ = (1,1,1) and use 𝑃0 (𝑥0 , 𝑦0 , 𝑧0 ) = (𝑎, 𝑏, 𝑐) and 𝑃(𝑥, 𝑦, 𝑧) to find:
Note that ⏟
𝒏
𝑬𝒒𝒖𝒂𝒕𝒊𝒐𝒏 𝑰𝑰
⃗⃗⃗⃗⃗⃗⃗⃗⃗
𝑷𝟎 𝑷 = (𝑥 − 𝑎, 𝑦 − 𝑏, 𝑧 − 𝑐)
⏟
𝑬𝒒𝒖𝒂𝒕𝒊𝒐𝒏 𝑰𝑰𝑰
Substitute Equation II and III in Equation I to get the Vector equation in point form:
(1,1,1) ∙ (𝑥 − 𝑎, 𝑦 − 𝑏, 𝑧 − 𝑐) = 0
To convert to Cartesian form, expand the LHS using the dot product definition:
(𝑥 − 𝑎) + (𝑦 − 𝑏) + (𝑧 − 𝑐) = 0
General Form
To convert to general form, move the constants to the RHS:
𝑥+𝑦+𝑧 =𝑎+𝑏+𝑐
Factor to get the vector version:
(1,1,1) ∙ (𝑥, 𝑦, 𝑧) = 𝑎 + 𝑏 + 𝑐
Normal Form
The magnitude of the normal vector is:
√12 + 12 + 12 = √3
To convert to normal form, divide both sides of the Cartesian equation from above by the magnitude of the
normal vector:
𝑥 𝑦 𝑧 𝑎+𝑏+𝑐
+ + =
√3 √3 √3 √3
13
14
P a g e 166 | 168
Get all the files at: https://bit.ly/azizhandouts
Aziz Manva (azizmanva@gmail.com)
Example 2.157
15
16
Example 2.159
Example 2.160
Coplanar lines are lines that are in the same plane. If two lines are not coplanar,
what kind of lines are they? Give one example of such kind of lines.
Example 2.161
P a g e 167 | 168
Get all the files at: https://bit.ly/azizhandouts
Aziz Manva (azizmanva@gmail.com)
⃗⃗⃗⃗⃗
𝒂𝟏 and ⃗⃗⃗⃗⃗
𝒂𝟐 are points on the first and second line respectively.
If the two lines are coplanar, then they lie in the same plane, and so does the vector joining them:
⃗⃗⃗⃗⃗
𝒂𝟏 − ⃗⃗⃗⃗⃗
𝒂𝟐
And finally, the dot product of a vector perpendicular to the plane with a vector in the plane must be zero:
(𝒂
⏟⃗⃗⃗⃗⃗𝟏 − ⃗⃗⃗⃗⃗)
𝒂𝟐 ∙ ⏟ ⃗⃗⃗⃗⃗𝟏 × ⃗⃗⃗⃗⃗
(𝒃 𝒃𝟐 ) = 0
𝑽𝒆𝒄𝒕𝒐𝒓 𝑽𝒆𝒄𝒕𝒐𝒓⊥
𝒊𝒏 𝒕𝒉𝒆 𝑷𝒍𝒂𝒏𝒆 𝒕𝒐 𝒕𝒉𝒆 𝒑𝒍𝒂𝒏𝒆
𝑐1 𝑐2 𝑐3
⃗⃗ ∙ (𝒂
𝒄 ⃗⃗ 𝑎
⃗⃗ × 𝒃) = | 1 𝑎2 𝑎3 |
𝑏1 𝑏2 𝑏3
Example 2.163
5−𝑥 𝑦−7 𝑧+3 𝑥−8 2𝑦−8 𝑧−5
Show that the lines −4
= 4
= −5
and 7 = 2
= 3
are coplanar.
17
Example 2.164
𝑎𝑥1 + 𝑏𝑦1 + 𝑐𝑧1 = 𝑝
⏟
𝑬𝒒𝒖𝒂𝒕𝒊𝒐𝒏 𝑰
𝑎𝑥2 + 𝑏𝑦2 + 𝑐𝑧2 = 𝑝
⏟
𝑬𝒒𝒖𝒂𝒕𝒊𝒐𝒏 𝑰𝑰
Subtract Equation II from Equation I:
𝑎(𝑥1 − 𝑥2 ) + 𝑏(𝑦1 − 𝑦2 ) + 𝑐(𝑧1 − 𝑧2 ) = 0
[𝑎, 𝑏, 𝑐] ∙ [𝑥1 − 𝑥2 , 𝑦1 − 𝑦2 , 𝑧1 − 𝑧2 ] = 0
165 Examples
P a g e 168 | 168